Sunteți pe pagina 1din 84

GAZETA SERIA

MATEMATIC
A

REVIST DE CULTUR MATEMATIC

ANUL XXVI(CV)

Nr. 4 / 2008

Margini pentru rdcinile polinoamelor


cu coecienii compleci
de Doru tefnescu

Abstract
The computation of the roots of a polynomial is one of the oldest problems in
Mathematics and it is basic for many mathematical domains. It is known through
Field Theory that the solution by radicals of polynomial equations is not generally
possible.

On the other hand the numerical solution is an valuable computational

approach.
We shall present some results on bounds for the absolute values of univariate
polynomials over the led

of complex numbers.

The knowledge of such bounds

is an important step for the localization of the roots and  implicitely  for their
aproximation with a preestablished precision.

Key words and phrases: Polynomial roots, Numerical solution.


M.S.C.: 30C15, 26C10,12D10.
Introducere
Una dintre principalele probleme care se pun n legtur cu polinoamele ntr

o variabil (nedeterminat) o constituie gsirea rdcinilor lor. Deoarece calcularea


exact a rdcinilor unui polinom cu coecieni reali sau compleci nu este posibil
dect pentru polinoame particulare este necesar s avem metode de aproximare a
acestor rdcini. n acest context determinarea intervalelor sau domeniilor n care se
gsesc rdcinile unui polinom cu coecieni reali sau compleci permite elaborarea
unor metode algoritmice de estimare a acestor rdcini. O prim etap const n
gsirea unor margini (superioare i inferioare) ale valorilor absolute ale rdcinilor.
Vom descrie cteva astfel de margini superioare i vom compara eciena rezultatelor.
Prin considerea polinomului reciproc se pot deduce ulterior i margini inferioare.
Studiile privind rezolvarea numeric a ecuaiilor algebrice au fost iniiate de
Newton, Lagrange, Cauchy i ali titani. Galois nsui a scris i el un memoriu despre
rezolvarea numeric a ecuaiilor algebrice.
Calcularea numeric a rdcinilor reale, de exemplu, are drept etap prelimi-

nar

izolarea rdcinilor,

adic calcularea unui numr nit de intervale astfel nct

287

ecare interval s conin o rdcin iar ecare rdcin s e coninut ntrun


interval.

1. Marginea lui Cauchy


Matematicianul francez

Augustin-Louis Cauchy

a publicat n anul 1822 un

criteriu simplu care permite estimarea marginilor modulelor rdcinilor polinoamelor


cu coecieni n corpul numerelor complexe.

Teorema 1.

Fie unica rdcin pozitiv a polinomului

F (X) = X n |a1 |X n1 |an | , unde a1 , . . . , an C.

Atunci toate rdcinile polinomului P (X) = X n + a1 X n1 + + an1 X + an se


gsesc n discul {|z| }.
Demonstraie. n primul rnd s observm c polinomul

F (X) = X n |a1 |X n1 |an1 |X |an |


are ntr-adevr o singur rdcin real pozitiv, avnd toi coecienii reali i o
singur schimbare de semn (regula lui Descartes a semnelor).
Considerm

z C, |z| > .

Avem


|P (z)| |z|n |a1 | |z|n1 + + |an1 | |z| + |an | = F (|z|) > 0 ,
aadar

P (z) 6= 0.
Utilizarea Metodei lui Cauchy
Procedeul lui Cauchy descris n Teorema 1 permite obinerea unor expre-

sii simple pentru marginea superioar a modulelor rdcinilor n funcie de talia


coecienilor polinomului

P.

Prezentm cteva aplicaii ale acestui rezultat la de-

terminarea unor margini ale radacinilor unui polinom cu coecienii compleci. O

A.-L. Cauchy :
Numrul 1 + M , unde M = maxni=1 |ai |, este
o margine superioar a modulelor rdcinilor polinomului P (x) = xn + a1 xn1 +
+ an C[x] .
prim aplicaie este urmtorul rezultat obinut chiar de

Corolarul 2. [Cauchy, 1822].

Demonstraie. Cu notaiile din Teorema 1 avem


F (1 + M ) = (1 + M )n |a1 |(1 + M )n1 + + |an |

(1 + M )n 1
= 1 > 0.
(1 + M )n M (1 + M )n1 + + 1 = (1 + M )n M
1+M 1
Prin urmare 1 + M > , deci 1 + M este o margine superioar a valorilor
absolute ale rdcinilor polinomului P .
2
Exemplul 1. S considerm polinomul P (X) = X 5 2X 4 2X 3 X 2 +X 2 .
Conform rezultatului precedent se obine marginea superior 1 + max{2, 1} = 3 .
1/s
este o margine superioar
Propoziia 3. Numrul M = 2 maxn
s=1 |as |
modulelor rdcinilor polinomului P .
Demonstraie. Deoarece |as | (M/2)s se obine
|ai | M ni
288

Mn
,
2i

aadar

n
X

|ai | M

ni

i=1

1
1
1
+ + n
+
2 22
2


= M

1
1 n
2


.

Cu notaiile din teorema 1 avem

F (M ) = M n
prin urmare

n
X


 n

M
1
|ai | M ni M n M n 1 n =
> 0,
2
2
i=1

este o margine superioar a modulelor rdcinilor.

Exemplul 2. Considernd tot polinomul

P (X) = X 5 2X 4 2X 3 X 2 + X 2 .
2 max{2, 1} = 4 .
Fie P (X) = X n + a1 X n1 + + an1 X + an un
polinom neconstant cu coecienii numere complexe i e 1 , . . . , n > 0 astfel nct
se obine marginea superior

Teorema 4 [Fujiwara].

1
1
1
+
+ +
1.
1
2
n

Atunci numrul

1/i

max (i |ai |)
i=1

este o margine superioar a modulelor rdcinilor polinomului P .


Demonstraie. S punem
toi

i,

deci

n
X

|ai | M ni

i=1

1/i

M = maxni=1 (i |ai |)

Avem

|ai |

Mi
i

pentru

n
n
X
X
1
Mi
M ni M n
.
i
i
i=1
i=1

Prin urmare

n
X
1
F (M ) M n M n
= Mn

i
i=1

n
X
1
1

i
i=1

!
> 0,

ceea ce demonstreaz teorema.


O alt aplicaie a Teoremei 1 este

Prropoziia 5.

Fie > 0, |a1 | . Atunci


1
n ai i1
+ max
i=2

este o margine superioar a modulelor rdcinilor.


Demonstraie. Considernd

a 1/i
i
M = + maxni=1 ,

avem

|ai | (M )i1 .
289

Prin urmare

n
X

|ai | M ni

n
X

i=1

(M )i1 M ni =

i=1

<

Mn M

M
M

Mn X
M i=1

M
M

i
<

1
M
= M n1
= Mn .
M

1
M

Prin urmare

F (M ) M n M n = 0 .
Observaie. n particular, dac

a1 = 0

atunci numrul

din Propoziia 5

poate  orice numr real pozitiv.

Exemplul 3. Considerm polinomul

P (X) = X 9 2X 8 + X 6 4X 4 + X 1 .
Avem

a1 = 2,

deci putem alege orice numr

a 1/(i1)
i
= max
max
2i9

1
,
a

a 2.

Se obine

 1/2  1/4  1/7  1/8 )


 1/4
1
4
1
1
4
,
,
,
=
.
a
a
a
a
a

Aadar alte margini superioare sunt date de

 1/4
4
M (a) = a +
a
Alegnd

a=2

pentru orice

a 2.

se obine marginea superioar

M (2) = 3.414 .
De fapt, adevrata margine superioar a modulelor rdcinilor este

2. Marginea

J.-L. Lagrange

1.997 .

R+
a enunat un alt rezultat privind marginile rdcinilor unui

polinom cu coecienii reali (v.

[4].

l reformulm pentru cazul mai general al

polinoamelor cu coecienii compleci i dm o demonstraie bazat tot pe Teorema


4 a lui

Fujiwara.

R + ]. Fie a1 , a2 , . . . , an numere complexe. Dac


R = |aj |1/j |ai |1/i = |ak |1/k pentru toi k 6= i, j , atunci numrul R + este o
margine superioar a valorilor absolute ale rdcinilor polinomului
Teorema 4 [Marginea

F (X) = X n + a1 X n1 + + an1 X + an .
Demonstraie. Dup Teorema 2 a lui

Cauchy

este sucient s artm c

unica rdcin real a polinomului

G(X) = X n |a1 |X n1 |an1 |X |an |


290

este mai mic dect

R + .

R = |aj |1/j |ai |1/i = |ak |1/k


1 , . . . , n > 0 astfel nct
Avem

k |ak | (R + )k

pentru toi

pentru toi

k 6= i, j .

Cutm acum

k = 1, 2, . . . , n .

Ar  sucient s e satisfcute inegalitile

k k (R + )k

k 6= j

pentru toi

j Rj (R + )j .
Alegem atunci


j =

R+
R

j


,

k =

R+

k
pentru

k 6= j

(1 k n),

de unde

k 
j
n
n 
X
X 1
X
1
1
R

+
=
=
+
=
k
k
j
R+
R+
k=1

k=1


1

k6=j

R+

Considerm acum

k6=j

n 

y=

R j j
n+1
R j
=
+

.
(R + )j
R (R + )j
R(R + )n
j

R
.

Se observ c

R j j
n+1

j
R (R + )
R(R + )n

y1

1
yj 1
1
+

j
y (y + 1)
y(y + 1)n

(y + 1)n + y(y j 1)(y + 1)nj 1


.
y(y + 1)n

Membrul drept al ultimei inegaliti este subunitar dac i numai dac

g(y) = y(y + 1)n (y + 1)n y(y j 1)(y + 1)nj + 1 0 .


Avem

g(y) = (y + 1)nj h(y) + 1 ,


h(y) = (y 1)(y + 1)j y(y j 1) .
y 1.

unde

Este sucient s avem

h(y) 0

pentru toi

ntradevr,

h(y)
> yj +
y1

 

 




j
j
j
1 y j1 +
1 y j2 + +
1 y+1 0
1
2
j1
291

deoarece toate parantezele de pe ultima linie sunt pozitive. De aici avem


pentru toi

h(y) 0

y 1.

Prin urmare

n
X
1
1.
k

Dup Teorema 4 numrul

k=1
superioar a modulelor rdcinilor lui

R+

este o margine

F.

3. Margini Inferioare

Cunoaterea unei margini superioare pentru modulele rdcinilor unui polinom permite calcularea imediat a unor margini inferioare. Aceasta reiese din urmtorul rezultat.

Fie P (X) = a0 X n + a1 X n1 + + an C[X] iar


P (X) = an X + an1 X n1 + + a0 polinomul s reciproc. Dac numrul K > 0
este o margine superioar a valorilor absolute ale rdcinilor polinomului P , atunci
1
este o margine inferioar a valorilor absolute ale rdcinilor polinonumrul
K
mului P .
Propoziia 7.

Demonstraie. Este sucient s observm c are loc relaia

P (X) = X P

1
X


.

2
P (X) = X 7 X 6 + 2X 4 2X 3 + X + 1. Polinomul
P (X) = X 7 + X 6 2X 4 + 2X 3 X + 1. Conform Teoremei 6 o

Exemplul 4. Fie
reciproc este

margine superioar a modulelor rdcinilor este

K =R+=

2+

2 2.449

Prin urmare o margine inferioar a modulelor rdcinilor polinomului

este

m = 0.408 .
4. Aplicaii
Gsirea unor limite pentru modulele rdcinilor polinoamelor cu coecieni
reali sau compleci nu rezolv imediat problema localizrii rdcinilor unor astfel
de polinoame. n aplicaii suntem interesai s gsim margini ct mai apropiate de
acealea adevrate. De asemenea, sunt de preferat procedeele care conduc la calcule
ce pot  duse la capt n timp real  de preferat chiar cu creionul i hrtia. Cum
calculatoarele electronice de astzi pot efectua n cteva minute  dac nu secunde
 un volum uria de calcule, considerm drept eciente i acele procedee care permit
obinerea rezultatelor doar cu ajutorul calculatoarelor.

P (X) = X 5 2X 4 + 2X 2 + X 2 C[X]. Atunci M = 2


i dup Teorema 2 obinem marginea superioar 1 + M = 3. n schimb, Propoziia
3 i Teorema 6 ne dau marginea 4.
Exemplul 5. Fie

Exemplul 6. S considerm polinomul

P (X) = X 5 + 4X 3 + 100X + 99 .
Teorema 2 a lui

Cauchy

conduce la marginea superioar

M1 = 1 + max{4, 100, 99} = 1 + 100 = 101 .


292

Utiliznd Propoziia 3 se obine

M2 = 2 max{41/2 , 1001/4 , 991/5 } = 2 3.163 = 6.326 .


Exemplul 7. S considerm polinomul

P (X) = X 11 2X 10 + X 9 2X 8 8X 4 + X 1 .
Cu notaiile din Teorema 6 avem

R=2

1.346 .

De asemenea

1 + max{|ai | ; 1 i 11} = 1 + 8 = 9
Se obin urmtoarele margini superioare:

9
4
3.346

Teorema 2
Propoziia 3
Teorema 6

Alte margini superioare pentru valorile absolute ale rdcinilor se pot obine
folosind Propoziia 5. n cazul polinomului
numr

a 2.

avem

a1 = 2,

deci putem alege orice

Se obine

a 1/(i1)
i
max
= max
2i11

1
,
a

 1/2  1/6  1/9  1/10 )


 1/6
2
8
1
1
8
,
,
,
=
.
a
a
a
a
a

Aadar alte margini superioare sunt date de

 1/6
8
M (a) = a +
a
Funcia

pentru orice

g : [0, ) R, g(x) = x + (8/x)1/6

a 2.

ind cresctoare cea mai bun

margine obinut prin Propoziia 5 este

M (2) = 3.259 .
Prin urmare, Teorema 6 i Propoziia 5 dau cele mai bune margini pentru
polinomul

P.

Prin utilizarea unui pachet de programe performant, cum este, de exemplu,

gp-pari

(v. [8]), se constat c modulul maxim al unei rdcini este

2.079 .

Concluzii
Marginile valorilor absolute ale rdcinilor polinoamelor intr-o nedeterminat
cu coecienii numere complexe pot  calculate n funcie de grad i coecieni prin
metode simple.
marginile lui

Printre cele mai eciente sunt marginea

Fujiwara.

R+

a lui

Lagrange

Cunoaterea acestor margini reprezint un pas important

pentru calcularea rdcinilor ecuaiilor algebrice.

293

Bibliograe
[1] A.-L. Cauchy,

Exercices de Mathmatiques,

t. 4, Paris (1829).

Polinoame i ecuaii algebrice,

[2] L. Panaitopol, I. C. Drghicescu,

Editura Albatros

(1980).

ber die obere Schranke des absoluten Betrages der Wurzeln einer algebraischen Gleichung, Thoku Math. J., 10, 167171 (1916).

[3] M. Fujiwara,

Trait de la rsolution des quations numriques,


uvres, t. VIII, GauthierVillars, Paris (1879).)

[4] J.-L. Lagrange,


(Reprinted in

[5] M. Mignotte, D. tefnescu,

Polynomials  An algorithmic approach,

Paris (1798).

Springer Verlag

(1999).
[6] M. Mignotte, Computer Algebra  O introducere n algebra computaional, Editura
Universitii din Bucuresti (2000).
[7] D. tefnescu,
[8]

gp-pari,

Inequalities on polynomial roots, Math. Ineqs. Appl., 5, 335347 (2002).

http://pari.math.u-bordeaux.fr/downloads.

Universitatea din Bucureti


E-mail: stef@rms.unibuc.ro
JOCUL CU NUMERE I AXIOME: SISTEME
PEANO-DEDEKIND
de

A. L. Agore i G. Militaru

Abstract
The set of non negative integers is dened by using the Peano axioms and a
detailed proof of the Hilbert recursion theorem is given, giving thus the universality
(and as a consequence the unicity) of the Peano-Dedekind systems.
We consider as new the proof of the converse of Hilbert theorem without using the
axiom of innity from Zermelo-Frankel system.

Key words: natural numbers, Peano axiom, Hilbert recursion theorem.


M.S.C.: 03E30, 03E75.
Ce este un numr natural? De ce

1 + 1 = 2?

Sunt ntrebri pe care i le pun

probabil unii copii cnd iau pentru prima dat contact cu matematica elementar.
Ceva mai trziu unii dintre acetia, atrai de matematic, probabil c se vor ntreaba
din nou, de data asta mai nuanat:
poate demostra c

1 + 1 = 2?

1+1 = 2

este o axiom? Este o teorem? Se

Vom ncerca n acest articol s elucidm cteva dintre

aceste ,,mistere ale mereu fascinantelor numere naturale.


Construcia axiomatic a numerelor naturale a fost dat pentru prima dat

Giuseppe Peano n 1889 n faimoasa lucrare Arithmetices principia, nova


methodo exposita. n aceast lucrare au fost denite ceea ce mai trziu s-au numit
sistemele Peano : ali istorici ai matematicii le denumesc sisteme Peano-Dedekind
riguros de

(aa cum le vom numi i noi n prezenta lucrare) pentru a marca i contribuia lui

Dedekind
294

la aceast construcie fundamental a matematicii.

n mod neateptat

Peano-Dedekind (aa numita ,,recursion theorem ) a fost demonstrat mult mai trziu: primii care au demonstrat-o au fost
Hilbert i Bernays n cartea lor ,,Grundlangen der Mathematik i independent de
P. Lorenzen n 1938. Este teorema 1 de mai jos, aa cum a fost demonstrat de
Hilbert i Bernays i pe care o reproducem pentru a o face cunoscut ct mai mul-

teorema fundamental a sistemelor

tor elevi i profesori ce ndrgesc matematica pentru c este una dintre cele mai
frumoase demostraii din matematica elementar.

O consecin, printre altele, a

acestei teoreme este unicitatea pn la un izomorsm a sistemelor

Peano-Dedekind

dar i existena i unicitatea adunrii i nmulirii numerelor naturale. Teorema 2 de


mai jos este o reciproc a acestei teoreme i are o demonstraie foarte elementar i
nou fr a folosi axioma innitului. Aceste dou teoreme mpreun permit cea mai
elegant i rapid deniie a numerelor naturale: ele sunt obiecte iniiale n ,,clasa

(A, a, ) formate dintr-o mulime nevid A, un element a A i


: A A (pentru detalii recomandm un recent i excelent articol [5]). n
anul 1908 Ernst Zermelo a propus primul set de axiome care fundamenteaz teoria
mulimilor: ele au fost completate ulterior de Abraham Fraenkel n 1922: sunt cele
zece celebre axiome numite axiomele Zermelo-Fraenkel care constitutie fundamentul
tuturor tripletelor

o funcie

matematicii. Pot  gsite, de exemplu n citeB, pag. 103. Una dintre aceste axiome
este aa numita axiom a innitului: din ea se construiesc uor numerele naturale
i n fapt toate axiomele

Peano

se deduc din trei din axiomele

Zermelor-Fraenkel.

n nal, vom demonstra pe scurt teorema care construiete n mod unic adunarea
numerelor naturale i vom arta c

1+1 = 2

este de fapt o teorem, rspunznd n

felul acesta ntrebrii din introducere.


n cele ce urmeaz vom lucra cu noiunea de funcie aa cum a fost denit

Kuratowski n 1914: se numete funcie


dou mulimi f A B este o submulime

de

a A !b B
Dac

(a, b) f vom nota acest


f : A B.

un triplet

(A, B, f ),

unde

sunt

cu proprietatea:

astfel nct
lucru cu

(a, b) f.

f (a) = b,

iar funcia

(A, B, f )

o vom

nota ca de obicei cu

Sisteme Peano-Dedekind
Vom introduce acum cele trei axiome care denesc sistemele Peano-Dedekind

Peano n 1889.
Se numete sistem Peano-Dedekind un triplet (N, 0, s), unde N
este o mulime nevid, 0 N i s : N N este o funcie astfel nct :
(P 1): s(x) 6= 0, x N
(P 2): s este o funcie injectiv.
(P 3): Dac P N astfel nct 0 P i pentru orice x P avem c s(x) P
atunci P = N .

aa cum au fost date de

Deniie.

(N, 0, s) este sistem Peano-Dedekind atunci folosind


P := {0} Im(s) obinem imediat c N = {0} Im(s), deci:

Observaie. Dac
oma

(P 3)

pentru

y N, y 6= 0, !x N

astfel nct

y = s(x).

Peano-Dedekind ;
recursion theorem i a fost demonstrat de D. Hilbert

Teorema urmtoare este teorema fundamental a sistemelor


ea este cunoscut sub numele de

axi-

295

P. Bernays

i independent de

P. Lorenzen

([3], [4]). Vom prezenta demonstraia

n detaliu pentru a arta c toate cele trei axiome ce denesc sistemele

Dedekind

sunt utilizate n cursul demonstraiei: n special condiia

ntr-un singur loc, extrem de bine ascuns.


aximei

(P 2)

(P 2)

Peano-

care apare

De exemplu n [4] sau n [2] utilizarea

nu este explicit evideniat.

Teorema 1. Fie (N, 0, s) un triplet Peano-Dedekind. Atunci pentru orice


triplet (A, a, ) format dintr-o mulime nevid A, un elment a A i o funcie
: A A, exist i este unic o funcie f : N A astfel
!f
nct
f (0) = a i f s = f .
N A
n
particular, orice dou sisteme Peano-Dedekind sunt
|
s |

izomorfe: i.e. dac (N, 0, s) i (N 0 , 00 , s0 ) sunt sisteme


N A
Peano-Dedekind atunci !f : N N 0 o funcie bijectiva
!f
astfel nct f (0) = 00 i f s = s0 f .

Demonstraie. Fie (A, a, ) un triplet, unde A este o mulime nevid, a un


A i : A A o functie. Fie mulimea tuturor submulimilor U ale
N A care satisfac simultan urmtoarele dou condiii:
i. (0, a) U ;
ii. dac (n, b) U atunci (s(n), (b)) U .
T
6= , deoarece N A . Fie f :=
U N A. Vom arta mai nti

element din
lui

U
c

este o funcie. Pentru aceasta avem de artat dou condiii:


a)

n N, x A astfel nct (n, x) f ;


(n, x) f i (n, x0 ) f x = x0 .

b) dac

S artm mai nti prima condiie.


Vom folosi axioma (P 3) pentru:
P := { x A astfel nct (n, x) f }. Evident 0 P cci (0, a) f . Fie
acum p P . Atunci x A astfel nct (p, x) f . Cum f obinem c
(s(p), (x)) f i deci s(p) P . Din (P 3) obinem c P = N , adic a) are loc.
0
0
S artm acum b). Fie P := {(n, x) f, (n, x ) f x = x }. Vom
arta c P veric (P 3) folosind toate cele trei condiii din axiomele Peano. S
0
0
artm c 0 P . Cum (0, a) f este sucient s artm c: (0, a ) f a = a.
0
0
0
0
Presupunem prin absurd c a 6= a i (0, a ) f . Fie atunci f := f \ {(0, a )} f .

6=

f 0 . Avem (0, a) f 0 cci (0, a) 6= (0, a0 ). Dac (n, b)


f obinem (s(n), (b)) f i deci (s(n), (b)) =
6 (0, a0 ) cci

Vom arta c

f0 f,
6=

cum

s(n) 6= 0, n N.

Deci

adic

f 0 .

Atunci

f f0 f,
6=

n P atunci s(n) P .
Presupunem prin absurd c n P astfel nct s(n)
/ P . Fie x A astfel nct
(n, x) f (existena lui x este asigurat de punctul a)). Atunci, cum f obinem
(s(n), (x)) f . Cum am presupus c s(n)
/ P obinem c z A astfel nct
(s(n), z) f i z 6= (x). Fie f 00 := f \ {s(n), z} f . Vom arta c f 00 . Mai
contradicie.

Deci

0 P.

(s(n), (b)) f 0

S artm acum c dac

6=

(0, a) f 00 cci (0, a) f i s(n) 6= 0, n N adic (0, a) 6= (s(n), z). Fie


(m, d) f 00 . Vrem s artm c (s(m), (d)) f 00 . Avem dou cazuri:
Cazul 1). Dac m = n avem (n, d) f 00 f i (n, x) f , i cum n p obinem

nti

6=

d = n, de unde obinem (s(n), (d)) = (s(n), (x)) 6= (s(n), z)


(s(n), (d)) = (s(n), (x)) f 00 .
296

cci

z 6= (x)

i deci

Cazul 2). Dac m 6= n aunci (s(m), (d)) f (f ) i (s(m), (d)) 6=


6= (s(n), z) cci s(m) 6= s(n), m 6= n (aici este singurul loc unde se folosete
00
injectivitatea lui s). Deci f
adic f f 0 f , contradicie. Deci P veric
6=

(P 3) i atunci P = N adic f este funcie; f (0) = a este doar o scriere pentru


(0, a) f .
Pentru (n, b) f avem (s(n), (b)) f adic dac b = f (n) (b) = f (s(n))
sau echivalent (f (n)) = f (s(n)), n N .
A rmas de artat unicitatea funciei f . Fie g : N A o funcie astfel nct
g(0) = a i g s = g i P := {g(n) = f (n)}. 0 P cci g(0) = a = f (0). Fie
x P . Atunci:
xP

g(s(x)) = (g(x)) == (f (x)) = f (s(x)) adic s(x) P.


Aplicm din nou

(P 3)

i obinem

P =N

A rmas de artat c orice dou sisteme

N
s |
N

!f

!g
!g

!g

| 0
s
N

Artm c

N
s |
N

Fie

N0

IdN

gf
gf

Idn

(N, 0, s) i (N 0 , 00 , s0 ) dou sisteme Peano-Dedekind.

!f : N N 0

astfel nct

f (0) = 00

f s = s0 f

!g : N 0 N

astfel nct

g(00 ) = 0

g s0 = s g .

este inversa lui


Avem:

N
|
s

este unic.
izomorfe.

Aplicnd succesiv prima parte a teoremei obinem:

adic g = f i deci f
Peano-Dedekind sunt

f:
(g f )(0) = g(f (0)) = g(00 ) = 0

()

(g f ) s = g s0 f = s (g f ).

()

Id satisface relaiile () i () i din unicitate


g f = Id. n mod analog se obine i f g = Id.

Dar i
obinem:

Vom demostra acum reciproca acestei teoreme:

Teorema 2. Fie un triplet

s:N N

(N, 0, s)

unde

este o mulime nevida,

0 N,

este o funcie cu proprietatea c:

A mulime nevida, a A i : A A o funcie !f : N A


astfel nct f (0) = a i f (s(n)) = (f (n)), n N.

( )

Atunci (N, 0, s) este sistem Peano-Dedekind.


Demonstraie. Aratm pentru nceput c (N, 0, s) veric axioma (P 3). Fie
P N astfel nct 0 N i s(x) P , x P . Fie i : P N incluziunea canonic
i(x) = x. Vom arta c i este surjectiv i deci P = N .
Considerm tripletul (A, a, ) := (P, 0, s|P ) care are
!f
sens
ntruct s(x) P , x P . Obinem c !f : N P
N P
astfel
nct f (0) = 0 i f s = s|P f adic f (s(x)) =
|
s|I |
s|I
= s(f (x)), x N .
N P
Fie funcia i f : N N . Artm c i f = IdN ; va
!f
rezulta c i este surjectiv.
297

Aplicm din nou

IdN

if
if

Idn

( )

(A, a, ) = (N, 0, s).


IdN (0) = 0 i IdN s = s IdN

pentru

Cum

avem c

IdN

este unica funcie ce nchide comutativ diagrama. Artm

i f = IdN .
(i f )(0) = i(f (0)) = i(0) = 0 i
((i f ) s)(x) = i(f (s(x))) = i(s(f (x))) = s(f (x)) i
N
N
(s (i f ))(x) = s(i(f (x))) = s(f (x)).
Deci i f = IdN adic i este surjectiv i deci P = N .
Aratm acum c s(x) 6= 0, x N . Fie x0 N astfel nct s(x0 ) = 0 i
lum n ( ) (A, a, ) := (N, x0 , ) unde : N N , (x) := 0, x N . Obinem
c !f : N N astfel nct f (0) = x0 i f s = f . Deci f (s(x0 )) = (f (x0 )).
Obinem deci c x0 = 0 adic s(0) = 0. Fie P = 0 N . Atunci 0 P i dac p P
atunci s(p) P , cci s(0) = 0 P . Din ce am artat mai sus obinem c P = N
adic (N, 0, s) = (0, 0, s) ndeplinete condiia ( ). Fie A := {a, b} cu a 6= b
(existena mulimii A este asigurat de axioma perechilor) i tripletul ({a, b}, a, )
cu : {a, b} {a, b}, (a) = (b) := b. Obinem c !g : {0} {a, b} astfel nct
g(0) = a i g(s(0)) = (g(0)). Deci g(0) = b 6= a = g(0), contradicie .
s |

|
s

if

are aceleai proprieti i deci

Avem

A rmas de artat injectivitatea funciei s. Considerm tripletul (A, 0, ) :=


= (N N, (0, 0), ), unde : N N N N , (n, m) := (m, s(m)). Din ( )
obinem c !f : N N N astfel nct f (0) = (0, 0) i f (s(n)) = (f (n)), n N .
Fie g := 1 f , h := 2 f : N N unde 1 (x, y) := x , 2 (x, y) := y ,
1 , 2 : N N N . Atunci f (n) = (g(n), h(n)) n N . Evident: g(0) =
= 1 (f (0)) = 1 (0, 0) = 0 i h(0) = 2 (f (0)) = 2 (0, 0) = 0. Cum f (s(n)) =
= (f (n)) obinem (g(s(n)), h(s(n))) = (h(n), s(h(n))), adic: g(s(n)) = h(n) i
h(s(n)) = s(h(n)).
IdN
n particular, h(0) = 0 i h(s(n)) = s(h(n)), n N . Fie

N
N
(A, a, ) := (N, 0, s). Din ( ) obinem c Idn : N N
h
este unica funcie ce nchide comutativ diagrama. Dar la fel
|
s |
s
h
face i h. Din unicitate obinem c h = IdN . Din relaia

g(s(n)) = h(n) obinem: g(s(n) = n, n N deci s este


N
N
injectiv.

Idn

Observaie. Vom arta acum cum se construiete un model de sistem PeanoDedekind folosind axioma innitului. Fie o mulime X astfel nct X i y X
avem y {y} X . Notm cu mulimea tuturor submulimilor V ale lui X care
ndeplinesc proprietile:
i.

ii. Dac
Evident
denim

y V y {y} V .
este nevid cci X .

N :=

. Avem
V
care are sens deoarece N .

s : N N, s(y) := y {y}
(N, , s) este un

S artm c

Fie

sistem

N , N

Peano-Dedekind.

n adevr:

s(y) 6= , y N cci y {y} =


6 ({} =
6 )
s injectiv este evident ntruct din y1 {y1 } = y2 y2
(am folosit axioma de separare).

298

obinem

y1 = y2

adic

obinem

Fie

P N

astfel nct

y P s(y) = y y P . Atunci P
N deci N P de unde

P = N.

Conform teoremei 1, un triplet


sm.

particip la intersecia care denete mulimea

Peano-Dedekind

este unic pn la un izomor-

Mai sus am construit efectiv un astfel de model.

Dedekind

oarecare

(N, 0, s),

a crui mulime suport

Fixm un sistem

Peano-

o vom numi mulimea nu-

merelor naturale, iar elementele sale le vom numi numere naturale. Vom nota

1 := s(0), 2 := s(1), 3 := s(2),...etc.


Printre cele mai importante consecine ale Teoremei 1 sunt teoreme care demostreaz existena i unicitatea adunrii i nmulirii numerelor naturale. Pentru
detalii complete recomandm [2].

Pentru comoditatea cititorului vom indica aici

doar cum este construit adunarea.

Teorema 3. Exist i este unic o funcie : N N N (o vom nota


(m, n) = m + n) astfel nct :
A1)

m + 0 = m,

A2)

m + s(n) = s(m + n),

m, n N . Aceast unic funcie se numete adunarea numerelor naturale.


Demonstraie. Fie m N xat. Considerm tripletul (N, m, s). Conform
Teoremei 1 aplicat tripletului (A, a, ) = (N, m, s) obinem c !fm : N N
astfel nct fm (0) = m i fm s = s fm . Denim : N N N , (m, n) :=
= fm (n) = m + n. Artm c este funcia cutat. Avem m + 0 = fm (0) = m
i m + s(n) = (m, s(n)) = fm (s(n)) = s(fm (n)) = s(m + n) deci ndeplinete
condiiile A1) i A2).
S artm acum unicitatea funciei : e funcia : N N N astfel nct
(m, 0) = m i (m, s(n)) = s((m, n)), pentru orice m, n N . Considerm
mulimea P := {n N | (m, n) = (m, n), m N }. Cum ambele funcii
i veric proprietatea A1) deducem c 0 P . Fie acum n P . Avem
c: (m, n) = (m, n), m N : deci s((m, n)) = s((m, n)), m N . Deci
(m, s(n)) = (m, s(n)), m N , adic s(n) P . Obinem c P = N , deci cele
dou functii coincid.

Observaie. Din Teorema 3 obinem:

1 + 1 = 1 + s(0) = s(1 + 0) = s(1) = 2.


Cu alte cuvinte ntrebarea din introducere capt un rspuns matematic riguros:

1 + 1 = 2 este consecina unei teoreme!

Pentru a se ajunge la ea a fost nevoie

de cteva secole de matematic, de inspiraia lui

Peano

i mna lui

Hilbert.

Bibliograe
[1] G. Bergman,

An Invitation to General Algebra and Universal Constructions,

Henry

Helson, 15 the Crescent, Berkeley CA, 94708 1998, 398 pp. 45, ISBN 0-9655211-41.
Diponibil on-line la http://math.berkeley.edu/ gbergman/245/
[2] D. Busneag, F. Boboc, D. Piciu,

Aritmetica i teoria numerelor, Editura Universitaria,

Craiova 1999. Disponibil on-line la


http://www.inf.ucv.ro/ busneag/books/aritmetica/aritmetica.pdf

299

[3] L. Henkin,

On Mathematical Induction, The Amer. Math. Monthly, 67(1960), 323-338.

[4] N. Jacobson,
[5] B. Mazur,

Basic Algebra I,

W.H. Freeman and Company, San Francsco, 1973.

When is one thing equal to some other thing ?,

preprint 5 septembrie 2006,

disponibil on-line la http://www.math.harvard.edu/ mazur/

Facultatea de Matematic i Informatic,


Universitatea din Bucuresti,
Str. Academiei 14, Romania.
E-mail: agoreloredana@yahoo.com i gmilit@al.math.unibuc.ro

Teorema de factorizare i aplicaii


de Costel Chite

Abstract
Key words:
M.S.C.: .
n acest articol vom prezenta teoreme de factorizaren cadrul mulimilor ct
i n categoria Ens i n cel al grupurilor n categoria Gr, prezentnd legtura dintre
ele.
n prima parte se studiaz teorema de factorizare n cadrul mulimilor abstracte (categoria Ens), legtura cu funciile surjective, injective, bijective i se d
un exemplu care ilustreaz faptele teoretice expuse. n partea a doua se studiaz
teorema de factorizare la grupuri (categoria Gr), care sunt n esen teoremele de
izomorsm de la grupuri, fcndu-se legtura cu faptele expuse n prima parte (de
exemplu, a se vedea legtura dintre nucleul unui morsm de grupuri i ucleul unei
funcii oarecare). n ne, partea a treia prezint ca aplicaii cteva exemple remarcabile de grupuri - factor sau de latici de subgrupuri (obinute pe baza teoremelor de
izomorsm) i se ncheie cu o scurt privire asupra grupurilor rezolubile i criteriului
de rezolubilitate al lui

Galois

pentru ecuaiile algebrice.

I. Categoria Ens. Vom reaminti cteva rezultate legate de mulimi factor.


Deniia 1. Fie f : A B o funcie. O funcie r : B A se numete

retract (sau invers la stnga) a lui f , dac r f = 1A .


Deniia 2. Fie f : A B o funcie. O funcie s : B A se numete
seciune (sau invers la dreapta) a lui f dac f s = 1B .
Propoziia 1. Fie f : A B o funcie cu A 6= . Sunt echivalente urmtoarele armaii :
a) f este injectiv;
b) f admite cel puin o retract ;
c) pentru orice mulime X i orice funcii g, h : X A avem f g = f h
implic g = h (simplicare la stnga ).
Propoziia 2. Fie f : A B o funcie. Sunt echivalente urmtoarele
armaii :
300

f este surjectiv ;
f admite cel puin o seciune ;
c) pentru orice mulime Y i orice funcii g, h : B Y avem g f = h f
implic g = h (simplicare la dreapta )
Deniia 3. Fie f = (A, B, F ) o relaie funcional, unde F A B este
gracul relaiei. Mulimea
a)

b)

Kerf = F F = {(x1 , x2 ) A A | |f (x1 ) = f (x2 )}

se numete nucleul lui f .


Observaii.
de echivalen pe A.

 1) Kerf este o relaie
1
A
= f (y) | y f (A) .
2)
Kerf
A
3) Dac pKerf : A
este surjecia canonic, atunci KerpKerf = Kerf .
Kerf
Teorema 1. (Factorizarea printr-o surjecie) Fie f : A B o funcie i
g : A C o funcie surjectiv. Exist i este unic o funcie h : C B astfel nct
f = h g dac i numai dac Kerg Kerf.
Demonstraie. ,, Fie (x1 , x2 ) Kerg . Atunci g (x1 ) = g (x2 )
h (g (x1 )) = h (g (x2 )) f (x1 ) = f (x2 ) (x1 , x2 ) Kerf .
,,  Cum g este surjectiv, rezult c exist s : C A astfel nct g s = 1C.
Denim funcia h = f s; Din g s = 1C rezult g s g = g . Fie x A, atunci
(g s g) (x) = g(x) sau
g ((s g) (x)) = g (x) (f s g) (x) = f (x)
(h g) (x) = f (x). Decif = h g .
0
0
Unicitatea lui h. Dac exist h : C B astfel nct f = h g rezult
0
h g = h g i, simplicnd, la dreapta (g este surjectiv) rezult h = h0 . 2
Observaii. 1) Dac Kerg = Kerf atunci h este injectiv.
2) Dac f este surjectiv atunci h este surjectiv.
1) Dach (c1 ) = h (c2 ), din g surjectiv, rezult c exist a1 , a2 A astfel
nct c1 = g (a1 ), c2 = g (a2 ). Deci:
sau

(h g) (a1 ) = (h g) (a2 ) f (a1 ) = f (a2 ) (a1 , a2 ) Kerf = Kerg


g (a1 ) = g (a2 ) c1 = c2 ,
deci

este injectiv.
2)

f =hg

surjectiv implic

surjectiv.

Prin dualitate se enun

Teorema 2. (Factorizarea unei funcii printr-o injecie). Fie f : B A o


funcie i g : C A o funcie injectiv.Exist i este unic o funcie h : B C
astfel nct f = g h dac i numai dac Imf Img .
Demonstraia se realizeaz ca n cazul teoremei 1 (a se vedea [4]).
Un corolar al teoremei 1 l reprezint:

Teorema 3. Dac f : A B este o funcie atunci exist o bijecie f :


A
f (A) astfel nct f = i f pKerf unde i : f (A) B , i (y) = y este
Kerf
aplicaia de incluziune.
301

Demonstraie.

Cum aplicaia

pKerf

aplicnd teorema 1, rezult c exist i este unic aplicaia


astfel nct

mImf

f = h pKerf .

Cum Imh

A
f (A), f (Kerf < x >) =
Kerf
h = i f , unde i este aplicaia de incluziune.

injectiv, rezult c funcia

h (Kerf < x >) este bijectiv


Decif = i f pKerf .
Observaie.
funciile cu domeniul

KerpKerf = Kerf ,
A
injectiv h :
B
Kerf

este surjectiv i

f :

Teorema 3 ne arat c imaginile unei mulimi

prin toate

sunt epuizate, pn la o bijecie, de mulimile ct ale lui

Exemplu. Se consider mulimile


funcia

f :AB

denit prin tabelul

A = {1, 2, 3, 4, 5, 6}, B = {a, b, c, d}


1 2 3 4 5 6
x
. Atunci:
f (x) a a a b b c

A.
i

F ={(1, a) , (2, a) , (3, a) , (4, b) , (5, b) , (6, c)} ;


F ={(a, 1) , (a, 2) , (a, 3) , (b, 4) , (b, 5) , (c, 6)} ;
Kerf ={(1, 1), (1, 2), (1, 3), (2, 1), (2, 2), (2, 3), (3, 1), (3, 2), (3, 3),
(4, 4), (4, 5) , (5, 4), (5, 5) , (6, 6)} ;
1
={{1, 2, 3} , {4, 5} , {6}} , Imf = {a, b, c} .
Kerf
ntre aceste mulimi, evident, exist o bijecie.

II. Categoria (Gr). n cadrul grupurilor ntlnim noiunea de nucleu al


Dac (G, ), (H, ) sunt grupuri i f : G H este un
ker f = {x G | f (x) = 1} este un subgrup normal al lui G,

unui morsm de grupuri.


morsm de grupuri,
numit nucleul lui
Morsmul
istntre

Kerf

f.
f ind,

n particular, funcie, se punentrebarea ce legtur ex-

de la funcii i

ker f

de la morsmele de grupuri?

Pentru a rspunde lantrebare vom analiza un morsm de grupuri


Notm

N = ker f / G; N

f : G H.
G

induce o relaie de echivalen (chiar de congruen) pe

notat prin


1
N ; (x1 ,x2 ) N G G x1 x1
= 1 f (x1 ) = f (x2 )
2 N f x1 x2
1

(x1 , x2 ) F F = Kerf unde am notat f = (G, H, F ) . Deci mulimea G subiacent grupului (G, ) se factorizeaz ca n cazul teoriei mulimilor unde funcioneaz
teoremele 1, 2, 3 prezentaten paragraful (Ens).

Fie f : G H un morsm de grupuri,


G
N / G i K = ker f , : G
, (x) = xN surjecia canonic. Dac N K
N
G
H astfelnct f = f . Mai
atunci exist un unic morsm de grupuri f :
N
mult, dac :
a) f este epimorsm dac i numai dac f este epimorsm ;
b) f este monomorsm dac i numai dac K = N ;
c) f este izomorsm dac i numai dac K = N i f este epimorsm.
Teorema 4. (Teorema factorului)

302

G
. Denim f (aN ) = f (a) . Dac aN = bN
N

1
1
atunci a
b N K , deci f a b = 1 sau f (a) = f (b). Deci f este bine denit.
Demonstraie. Fie

aN

Din

f (aN bN ) = f (abN ) = f (ab) = f (a) f (b) = f (aN ) f (bN ) ,


rezult c

f = f .
y Imf atunci exist a G astfel nct
y Imf . Reciproc, dac y Imf atunci exist

este morsm de grupuri;

a) Artm c Imf

Imf .

Fie

y = f (a) = f (aN ), deci


G
aN
astfel nct y = f (aN ) = f (a)
N

deci

y Imf ;

ker f = {aN | f (a) = 1} = {aN | a K} =


b)

este monomorsm dac i numai dac

atunci cnd

ker f = {1}

K
.
N
adic atunci i numai

K = N.

c) Se aplic a), b).

Teorema 5.

(Teorema nti de izomorsm).

G
morsm de grupuri cu K = ker f atunci
= Imf .
K

Dac f : G H este un

Demonstraie. Aplicm teorema factorului (T4) pentru

f (x) = f (x), pentru orice x G.


Lem. Fie (G, ) un grup, H G i N / G. Atunci :
a) HN = N H , deci HN G;
b) N / HN ;
c) H N / H .
Demonstraie. a) hN = N h, oricare ar  h G, n
S
S
h H ; HN =
hN =
N h = N H.

N =K

f :G

Imf ,

hH

particular pentru orice

hH

H, N G i HN = N H rezult HN G.
b) Cum gN = N g , oricare ar  g G rezult gN = N g , pentru orice g HN ,
deci N / HN .
G
G
0
c) Surjecia canonic : G
restricionat la H este : H
,
N
N
0
(h) = hN , oricare ar  h H .
ker 0 = {h H | hN = N } = {h H | h N } = H N . Subgrupul H N
este nucleul unui morsm deci este normaln H .
Teorema 6. (Teorema a doua de izomorsm) Fie (G, ) un grup , H G,
H HN
N / G atunci
.
=
H N
N
G
0
Demonstraie. Fie : G
epimorsmul canonic i = |H . Din lema
N
HN
0
0
precedent avem ker = H N i Im = {hN | h H} =
. Aplicnd prima
N
H HN
teorem de izomorsm (T5) avem
.
=
H N
N
Cum

Observaie.

Pentru a reine uor enunul teoremei se consider un romb ale crui vrfuri
se noteazntr-o ordine prin:

H N , N, HN, H .
303

Teorema 7. (Teorema

a treia de izomorsm). Dac (G, ) este un grup,


G
G N
N / G, H / G i N H atunci
.
=
H
H
N
G
G
Demonstraie. Fie aN
i denim f (aN ) = aH
. CoresponN
H
1
dena astfel denit este o funcie deoarece dac aN = bN atunci a
bN H
G
G

, f (aN ) = aH , f este un epimordeci aH = bH . Notm funcia f :


N
H
H
sm; ker f = {aN | aH = H} = {aN | a H} =
. Conform teoremeinti de
N
izomorsm rezult

N / G, H G, N 
H . Notm cu LN (G) laticea
G
G
laticea subgrupurilor lui
.
subgrupurilor lui G carel conin pe N i cu L
N
N
 
H
H1
H2
G
, (H) =
. Dac
=
, atunci pentru
Denim funcia : LN (G) L
H
N
N
N
orice h1 H1 , exist h2 H2 , astfel nct h1 N = h2 N .
1
Rezult h2 h1 N H2 decih1 H2 . Am demonstrat c H1 H2 . Analog
se aratH2 H1 . Deci H1 = H2 i este injectiv.
G
G
1
i : G
surjecia canonic.
(Q) G, N 1 (Q) i
Fie Q
N
N

1 (Q) = {aN | aN Q} = Q, deci este surjectiv.
1
Fie =
, (Q) = 1 (Q). Bijecia are proprieti interesante nsumate
Fie

(G, )

G
G N
.
=
H
H
N

un grup ,

n:

Teorema 8. (Teorema 
de coresponden).


atunci funcia : LN (G) L

G
N

Dac (G, ) este un grup, N / G

este bijectiv. Mai mult :

H2
H1

i n acest caz
N
N


H2 H1
[H2 : H1 ] =
:
;
N
N

a)

H1 H2 dac i numai dac

b)

H / G dac i numai dac

H G
/ ;
N N

H2
H1 H2
H2 N
c) H1 / H2 dac i numai dac
/
in acest caz
.
=
H1
N
N
H1
N
H1
Demonstraie. a) Dac H1 H2 , e aN
, a H1 H2 ,
N
H2
H1
H2
aN
. Invers, e aN

rezult a H2 , deci H1 H2 .
N
N
N
304

deci




H1
1
(aH1 ) = (aN )
, deci aH1 = bH1 , a
b H1 ; a1 N (bN ) =

 N 

H
H
H1
1
1
1
a bN
i (aN )
= (bN )
. Deci este bijectiv.
N
N
 N
1
H
aN a
H
H G
b) Avem (aN )
(aN )1 =
= , deci
/ .
N
N
N
N N
G
G p N
Aplicaiile G

sunt epimorsmele canonice. Avem:

H
N
N 
H
H
H
a ker(p ) (aN )
=
aN ; aN = hN, h H a H .
N
N
N
Deci H = ker(p ) / G.
c) H = H1 , G = H2 i se utilizeaz teorema a treia de izomorsm (T7).
Avem

III. Aplicaii
1) Se consider morsmul de grupuri:

f : (R, +) (C , ), f (x) = cos 2x + i sin 2x.


Deci

not

= {z C | |z| = 1} == D. Atunci,
R
aplicnd teorema nti de izomorsm, obinem c
= D. Se observ c |D| = C
Z
2) Se consider morsmul de grupuri g : (Q, +) D , g (x) = cos 2x + i
sin 2x, unde D a fost denit n exerciiul 1.
not

n
Atunci ker g = ZIm,, = {z C | n N a. i z = 1} =
= V numit grupul
ker f = {x R | f (x) = 1} = Z.

Imf

rdcinilor complexe ale unitii.


Observm c
c

Q
=V.
Z

|V | = 0 .

Atunci, aplicnd teorema nti de izomorsm obinem

det GLn (K) K .


Nucleul se noteaz cu SLn (K) = {A GL2 (K) | det A = 1} i se numete
grupul liniar special de grad n peste K .
Dar SLn (K) / GLn (K) i aplicnd teorema nti de izomorsm obinem:
3) Fie

un corp,

n N

i morsmul surjectiv

GLn (K)
SLn (K)
n particular, pentru

un corp nit cu

|SLn (K)| =

elemente obinem:

|GLn (K)|
.
q1

: G G un automorsm.
G
Considerm G G
, unde este surjecia canonic. Atunci
H
 



ker 1 = x G| 1 (x) = 1 = x G| 1 (x) H = (H) .
4) Fie

(G, )

= K .

un grup i

305

1 este epimorsm, aplicnd teoremanti de izomorsm obinem


G G
c
= .
(H)
H
5) Pentru n N, n 2, s se determine subgrupurile lui Zn .
dZ
Z
. Conform teoremei de coresponden (T8), H =
Fie H Zn
=
nZ
nZ

unde nZ dZ, deci d | n, d N . Aplicnd teorema 3 de izomorsm avem:


Z
Z
nZ
=
= Zd .
dZ
dZ
nZ


dZ
|Zn |
n


Conform teoremei lui Lagrange,
nZ = [Zn : H] = d deoarece [Zn : H] =
= |Zd | = d. Grupul H este ciclic i H =< db >, unde db = d + nZ.
6) S se determine laticea subgrupurilor grupului Z18 .
dZ
Subgrupurile H ale lui Z18 sunt de forma
unde d {1, 2, 3, 6, 9, 18}.
18Z
Cum

Obinem

6Z
18z

2Z
18Z
7) Dac

9Z
18Z

3Z
18Z

Z18

este un grup ciclic, atunci orice subgrup i orice grup factor al su

este ciclic. Se utilizeaz exerciiul 6) i stuctura subgrupurilor lui

Z i a subgrupurilor

factor ale sale.


Se tie c orice grup ciclic este izomorf e cu

Z, e cu un Zn .
A, B, C G. S se arate c:
a)
atunci A (BC) = B (A C).
A C B (A C)
b) Dac B / A atunci B C / A C i
;
=
BC
B
AC
A
c) Dac B / A i C / G atunci B C / A C i
.
=
BC
B (A C)
(G, ) un
Dac B A

8) Fie

grup i

ntr-adevr:
a) Evident, avem B (A C) A (BC). Invers,

c C . b1 a = c A C . Deci a = b b1 a B (A C).

e

a = bc

b) Aplicm teorema a doua de izomorsm considernd c:

H = A C.

Obinem

B C = (A C) B / A C

unde

b B,

K = B , G = A,

B (A C)
A C (A C) B
=
.
=
BC
B
B
c) Evident c

BC AC .

Vom arta c este normal. Fie

x = ac AC , atunci

x(BC) = acBC = acCB = aCB = aBC = BaC = BaCc = BCac = (BC)x.


306

K = BC , G = AC , H = A.
A
A (BC)
Obinem A (BC) / A i
, deci concluzia. 2
=
BC
A (BC)
Deniia 4. Un grup G se numete rezolubil dac n N i subgrupurile
1 = G0 , G1 , ..., Gn = G astfelnct Gi1 / Gi i Gi/Gi1 este grup abelian1 i n.
9) Fie (G, ) un grup, H, K G cu K / G. S se arate c:
G
a) Dac G este rezolubil, atunci H ,
sunt rezolubile.
K
G
sunt rezolubile atunci G este rezolubil.
b) Dac K ,
K
Aplicm teorema a doua de izomorsm pentru

ntr-adevr:
a)

Gi
Gi1
Gi1 H / Gi H i

1 = G0 / G1 / ... / Gn = G

exerciiul 8) punctul b).

sunt abeliene,

1 i n.

Aplicm

Gi H Gi1 (Gi H)
Gi

=
Gi1 H
Gi1
Gi1
Gi H
sunt abeliene.
Gi1 H
1 = G0 H / G1 H / ... / Gn H = H , deci H

ultimul grup ind abelian, deci i factorii


Considerm

este rezolubil.

Aplicm exerciiul 8) punctul c) i teorema a treia de izomorsm.


Obinem:
Dar:

Gi1 K / Gi K

Gi
Gi K
.
=
Gi1 K
Gi1 (Gi K)

Gi K
Gi K
K

.
=
Gi1 K
Gi1 K
K

Cum:

Gi
Gi
Gi1

=
Gi1 (Gi K)
Gi1 (Gi K)
Gi1
Gi K
ultimul grup ind abelian, deci
este abelian.
Gi1 K
G0 K G1 K
Gn K
G
G
Din 1 =
/
/ ... /
= , rezult c
este rezolubil.
K
K
K
K
K
Ki
b) Dac 1 = K0 / K1 / ... / Km = K i
este abelian, 1 i m. Atunci
Ki1
Gj
Gn
G
Gj
G0 G1
K
/
/ ... /
=
i
este abelian, 1 j n.
1=
=
Gj1
K
K
K
K
Gj1
K
Considernd 1 = K0 /K1 /.../Km = G0 /G1 /.../Gn = G, rezult G rezolubil.
variste Galois a dat o caracterizare a ecuaiilor rezolvabile prin radicali.
Teorema 9. Dac K este un corp comutativ, f K[X], grad(f ) 1, atunci
ecuaia f (x) = 0 este rezolvabil prin radicali dac i numai dac grupul Galois al
lui f (peste K ) este rezolubil.
307

A se vedea demonstraia de exemplu n lucrarea [4].

Bibliograe
[1] C. Nstsescu,

Introducere n teoria mulimilor,

Editura Didactic i Pedagogic, Bu-

cureti, 1974.
[2] C. Nstsescu,

Inele, module, categorii,

[3] Dorin Popescu, Constantin Vraciu,

Editura Academiei, Bucureti, 1976.

Elemente de teoria grupurilor nite,

Editura ti-

inic i Enciclopedic, Bucureti, 1986.


[4] Ioan Purdea, Ioana Pop,
[5] Joseph Rotman,

Algebr,

Editura GIL, Zalu, 2003.

Advanced Modern Algebra,

Prentice Hall, 2003.

Profesor drd.,
Colegiul Naional T. Vianu,
Bucureti

Izometrii liniare n

Rn

Cn ?

de Vasile Pop

Abstract
Key words:
M.S.C.: .
Punctele de plecare ale acestui articol l reprezint dou probleme deosebite
de algebr liniar, date la Olimpiada Naional de Matematic, Piteti 2007 i
Olimpiada Internaional de Matematic a Studenilor din Sud-Estul Europei, Cipru
2007. Cele dou probleme dau rspuns unei probleme teoretice importante: determinarea sau caracterizarea izometriilor n diferite spaii metrice. Scopul articolului
este de a ncadra aceste probleme n contextul teoretic corespunztor i a prezenta
rezultatele care rezolv problema izometriilor n spaii vectoriale normate de dimensiuni nite.

1. Introducere
Din punct de vedere teoretic, cunotinele minime necesare nelegerii lucrrii
se reduc la deniiile noiunilor de metric, spaiu metric, spaiu vectorial normat i
de izometrii n spaii metrice. Avnd ca model geometric planul euclidian deniiile
generale extind n mod natural noiunile de distan, norma unui vector din plan,
funcie care invariaz distanele, pentru care recomandm paragraful de ,,Grupuri
de transformri geometrice din [4].

Deniia 1.1. O funcie d : X X [0, ) se numete metric (distan)


pe mulimea X , dac sunt vericate axiomele :
(D1 ) : d(x, y) = d(y, x), x, y X ;
(D2 ) : d(x, y) + d(y, z) d(x, z), x, y, z X ;
(D3 ) : d(x, y) = 0 x = y .

308

Perechea (X, d) se numete spaiu metric.


Exemplul 1. a) Structura canonic de spaiu metric a axei reale este dat
de metrica:

d(x, y) = |x y|, x, y R.

b) Structura canonic de spaiu metric a planului euclidian

R2

este dat de

metrica:

d((x1 , y1 ), (x2 , y2 )) =
Fie

(X, +)

p
(x1 x2 )2 + (y1 y2 )2 ,

(x1 , y1 ), (x2 , y2 ) R2 .

K = R sau K = C, unde operaia


(a, x) = a x, a K , x X .

un spaiu vectorial peste corpul

de nmulire a vectorilor cu scalari este notat

Deniia 1.2. O funcie k k : X [0, ) se numete norm pe X dac


veric axiomele :
(N1 ) ka xk = |a| kxk, a K, x X
(N2 ) kx + yk kxk + kyk, x, y X
(N3 ) kxk = 0 x = 0.
Perechea (X, k k) se numete spaiu vectorial normat.
Exemplul 2. a) Norma canonic (euclidian) n Cn (Rn ) este denit prin:
p
k(z1 , z2 , . . . , zn )k = |z1 |2 + |z2 |2 + + |zn |2 ,

k k2 .
Cn (Rn ) sunt

care se noteaz cu
b) Pe

frecvent utilizate normele:

k(z1 , z2 , . . . , zn )k1 =

n
X

|zk |,

k(z1 , z2 , . . . , zn )k = max |zk |.


k=1,n

k=1
c) n general pentru orice

p [1, )

funcia

k kp : Cn (Rn ) [0, ),

denit

prin:

k(z1 , z2 , . . . , zn )kp =

n
X

!1/p
|zk |p

k=1
este o norm. n cazurile particulare

p = 2, p = 1

se obin normele de la

a) i b).
Este un simplu exerciiu demonstraia urmtorului rezultat:

Teorem. Dac (X, k k) este un spaiu vectorial normat atunci funcia


d : X X [0, ), d(x, y) = kx yk, x, y X este o metric pe X , numit
metrica indus de norm.
Observaie. Orice spaiu vectorial normat este un spaiu metric. n particular normele din Exemplul 2 induc metricile pe

d1 (x, y) =

Pn

k=1

Cn (Rn ):

|xk yk |,

d (x, y) = maxk=1,n |xk yk |,


v
uX
u n
d2 (x, y) = t
|xk yk |2 ,
k=1

Pn
1/p
dp (x, y) = ( k=1 |xk yk |p ) ,
309

pentru orice

x = (x1 , . . . , xn ), y = (y1 , . . . , yn ), x, y Cn (Rn ).

Deniia 1.3. Dac (X, k k) este un spaiu vectorial normat i f : X X


este o funcie, spunem c f invariaz norma dac kxk = kf (x)k, x X .
Este uor de demonstrat urmtorul rezultat:

Teorem. Dac (X, k k) este un spaiu vectorial normat i T : X X


este o aplicaie liniar care invariaz norma atunci T este o izometrie n raport cu
metrica indus i reciproc.
Demonstraie. ,,

Avem:

kx yk = kT (x y)k = kT (x) T (y)k d(x, y) = d(T (x), T (y)), x, y X.


,,

Avem:

d(x, y) = d(T (x), T (y)) d(x, 0) = d(T (x), T (0)) d(x, 0) = d(T (x), 0)
kxk = kT (x)k, x X.
2. Legtura ntre izometrii liniare i matrice
Ne propunem s determinm aplicaiile liniare

care

d . Dou argumente simple reduc


problema la determinarea unor matrice din Mn (C) sau Mn (R).
n
n
n
n
1) Orice aplicaie liniar T : C (R ) C (R ) este unic determinat de o
n
n
matrice MT Mn (C) sau Mn (R) i de alegerea unei baze n C sau R .

sunt izometrii n raport cu metricile

d1 , d2

T : Cn (Rn ) Cn (Rn )

2) Deniia izometriilor liniare nu depinde de alegerea unei baze.


n concluzie o izometrie liniar este denit de o matrice

Mn (R)

A Mn (C)

sau

care veric relaia:

x1
kA Xk = kXk, QX = . . . Cn
xn

sau

Rn .

Deniia 2.1. Spunem c matricea A Mn (C) sau Mn (R) este o matrice


de izometrie n raport cu norma k k dac :

kA Xk = kXk, QX Cn sau Rn .
Teorema 2.2. Mulimea matricelor de izometrie din Mn (C) sau Mn (R)
formeaz un subgrup multiplicativ n GLn (C) sau GLn (R).
Demonstraie.

Artm c mulimea izometriilor este inclus n GLn (C)

sau GLn (R). Dac prin absurd

A este matrice de izometrie i A este neinversabil,


det A = 0 i exist X Cn sau Rn , X 6= 0 astfel ca A X = 0 i atunci
kA Xk = 0 6= kXk (contradicie). Avem:
atunci

kXk = kA (A1 X)k = kA1 Xk, QX Cn (Rn )


deci i

A1

310

este matrice de izometrie.

Dac

A, B

sunt matrici de izometrie, atunci:

kA B Xk = kA (B X)k = kB Xk = kXk, QX Cn

sau

3. Structura grupului izometriilor liniare n raport cu


Teorema 3.1.

Rn .

k k1

Matricea A Mn (C) veric relaia:


kA Xk1 = kXk1 , QX Cn ,

dac i numai dac pe ecare linie i pe ecare coloan a matricei A exist un singur
element nenul, de modul 1.
Demonstraie. Fie

1
0
0
1

E1 =
. . . , E2 = . . .
0
0
coloanele matricei

In .

Din

kAEj k = kEj k
n
X

, . . . , En = 0 ,

...
1

rezult:

|aij | = 1,

j = 1, n.

(1)

i=1
Dac lum

X = E1 E2
n
X

rezult

|ai1 + ai2 | =

2=

|ai1 ai2 | = 2.

(2)

i=1

i=1
Avem:

n
X

n
X

|ai1 + ai2 |

i=1

n
X

(|ai1 | + |ai2 |) = 2

i=1

deci:

|ai1 + ai2 | = |ai1 | + |ai2 |


2=

n
X

|ai1 ai2 |

n
X

(3)

(|ai1 | + | ai2 |) = 2

i=1

i=1
deci:

|ai1 ai2 | = |ai1 | + | ai2 |

(4)

Din (3) i din (4):

ai1 = 0

sau

ai2 = 0.

(5)

Din (1) rezult c pe ecare coloan avem un element nenul. Din (5) rezult
c dac pe coloana 1 avem un singur element nenul i dac

ai1 6= 0,

atunci:

ai2 = ai3 = = ain = 0.

(6)

Rezult c pe ecare linie i pe ecare coloan avem un singur element nenul.

311

Din (6) i (1) pe ecare linie i pe ecare coloan avem un singur element

|aij | = 1.
Teorema 3.2. Matricea A Mn (R) veric relaia:

nenul i modulul lui este

kA Xk1 = kXk1 , QX M(R),

dac i numai dac pe ecare linie i pe ecare coloan a matricei A exist un singur
element nenul, egal cu 1 sau 1.
Demonstraie. Demonstraia este aceeai cu cea din Teorema 3.2, cu con-

aij {1, 1}.


Teorema 3.3. Grupul izometriilor liniare ale spaiului Rn n raport cu norma
k k1 este nit, ordinul su este n! 2n .

tinuarea

|aij | = 1

implic

Demonstraie. Conform teoremei 3.2, matricile izometriilor au cte un element egal cu 1 sau

pe ecare linie i ecare coloan.

O astfel de matrice se

obine printr-o permutare arbitrar a coloanelor matricii unitate


nmulirea ei cu 1 sau

Observaia 3.4.

2n

In ,

n!

moduri i

moduri.

Problema dat la Olimpiada Naional de Matematic,

Piteti 2007 are urmtorul enun:

Dac A Mn (R) este o matrice pentru care kA Xk1 = kXk1 , X Mn (R)


atunci exist m N astfel ca Am = In .
n
Deoarece, conform Teoremei 3.3, grupul matricelor A are ordinul nit 2 n!
n
rezult c putem lua m = 2 n!.
4. Structura grupului izometriilor n raport cu norma euclidian

Grupul izometriilor liniare ale spaiului euclidian Cn n raport


cu norma euclidian este grupul matricilor unitare:
Teorema 4.1.

On (C)

= {A Mn (C) | A A = A A = In },

unde A = (A)t este adjuncta (hermitian) a matricei A.


Demonstraie. Din kA Ej k2 = kEj k2 rezult:
n
X

|aij |2 = 1

(1)

i=1
Din

kA (Ej + Ek )k2 = kEj + Ek k2

n
X

|aij + aik |2 = 2

i=1

n
X

|aij |2 +

i=1

rezult:

n
X

|aik |2 + 2

i=1
n
X

n
X

aij aik = 2

i=1

aij aik = 0.

(2)

i=1

A A = In , deci A = A1 i atunci A A = In .
Teorema 4.2. Grupul izometriilor liniare ale spaiului euclidian Rn n raport
cu norma euclidian este grupul:
Din (1) i (2) rezult

On (R)

312

= {A Mn (R) | A At = At A = In }.

Demonstraie. Este identic cu demonstraia teoremei 4.1.


Observaia 4.3. Grupurile On (R) i On (C) sunt innite.
n particular, grupul rotaiilor:


R=
este inclus n


R =

sin
cos

cos
sin


|R

O2 (R).

5. Structura izometriilor liniare n raport cu norma


Teorema 5.1.

k k

Matricea A Mn (C) veric relaia


kA Xk = kXk , QX Cn ,

dac i numai dac pe ecare linie i coloan exist un singur element nenul, de
modul 1.
Demonstraie. Din kA Ej k = kEj k rezult max |aij | = 1, j = 1, n i
i=1,n
atunci matricea nu conine elemente de modul mai mare ca 1, deci pe ecare linie
modulul maxim este cel mult 1 i pe ecare coloan avem un element de modul 1.
Dac pe coloana

elementul nenul este

aij

X=
(transpusa conjugat a liniei

n
X

i)

i din

|aij |2 +

Li

n
X

lum:

rezult:

|aik |2 1

k=1

|aik |2 1 1 +

k6=j
deci

|aij | = 1
t

kA Xk = kXk

aik aik 1

i=1

cu

|aik |2 1,

k6=j

aik = 0, k 6= j .
n concluzie, pe ecare linie exist cel mult un element nenul (de modul 1).

Deoarece, conform Teoremei 2.2, matricea

este inversabil, rezult c pe ecare

linie exist un astfel de element.

Teorema 5.2.

Matricea A Mn (R) veric relaia


kA Xk = kXk , QX Rn ,

dac i numai dac pe ecare linie i pe ecare coloan exist un singur element
nenul, egal cu 1 sau 1.
Demonstraie. Se face la fel ca la teorema 5.1, |aij | = 1 aij {1, 1}.
Teorema 5.3. Grupul izometriilor liniare ale spaiului Rn n raport cu norma
k k este nit. Ordinul su este n! 2n .
O consecin a Teoremei 5.3 este problema dat la Olimpiada Internaional
Studeneasc din Cipru 2007.

Corolar 5.4.

Dac matricea A Mn (R) veric relaia


kA Xk = kXk , QX Rn
313

atunci exist m N astfel ca Am = In .


Demonstraie. Se poate lua m = n! 2n .
Observaia 5.5.
raport cu norma

k k ,

k k1

sunt nite i au

Grupul izometriilor liniare ale spaiului vectorial

Rn

coincide cu grupul izometriilor liniare n raport cu norma

n! 2n

elemente.

Bibliograe
[1]

Gazeta

Matematic (Suplimentul Olimpiadei Naionale de Matematic), Piteti,

2006.
[2]

Concursul Internaional Studenesc SEEMOUS 2007, Agras, Cipru.

[3] V. Pop,

Algebr liniar,

[4] V. Pop (colectiv),

Editura Mediamira, 2003 (pp. 90-104, 116-118).

Matematica pentru grupele de performan,

Editura Dacia Edu-

caional, 2004 (pp. 69-79).


[5] R. A. Horn, Ch. R. Johnson,

Analiz matricial,

Editura Theta, 2001 (pp. 204-213).

Universitatea Tehnic Cluj-Napoca,


Str. C. Daicoviciu 15, 400020, Cluj-Napoca, Romania,
E-mail: vasile.pop@math.utcluj.ro

EXAMENE I CONCURSURI
Olimpiada Internaional Sudeneasc SEEMOUS  2008
de Vasile Pop i Dorian Pop

n perioada 5-10 martie 2008 la Atena s-a desfurat a doua ediie a Olimpiadei
de Matematic a Studenilor din Sud-Estul Europei SEEMOUS2008, organizat de
Societatea de Matematic din Grecia i Societatea de Matematic din Sud-Estul Europei. La aceast ediie a Olimpiadei au participat 94 de studeni de la 21 de Universiti din Bulgaria, Cipru, Columbia, Grecia, Israel, Romnia, Rusia i Ucraina. Din
partea Romniei au participat studeni de la Universitatea Politehnic din Bucureti,
Universitatea Alexandru Ioan Cuza din Iai, Universitatea Tehnic din Cluj, Universitatea Tehnic Gh. Asachi din Iai i Academia Tehnic Militar din Bucureti.
Studenii romni au obinut 2 medalii de aur, 5 medalii de argint i 7 medalii de
bronz.

Locul nti, cu cel mai mare punctaj din concurs, a fost obinut de ctre

studentul

Cristian Tlu

de la Universitatea Politehnic din Bucureti. Concursul

a constat dintr-o singur prob cuprinznd un set de patru probleme.

Problema 1. Fie f : [1, ) (0, ) o funcie continu.


S presupunem c pentru orice a > 0 ecuaia f (x) = ax are cel puin o soluie
n intervalul [1, ).
a) Demonstrai c pentru orice a > 0, ecuaia f (x) = ax are o innitate de
soluii.
314

enun.

b)

Dai un exemplu de funcie continu strict monoton cu proprietatea din

Vladimir Todorov, Bulgaria


Soluia 1. a) S presupunem c se pot gsi constantele

f (x) 6= ax pentru orice x [b, ). Cum f este continu,


semn constant pe [b, ). Avem dou cazuri:
1) f (x) > ax pentru orice x [b, ). Denim:
ca

a>0

rezult c

b 1 astfel
f (x) ax are

f (x)
> 0.
x[1,b] x

c = min
Atunci pentru orice

x [1, )

avem:

f (x) min{a, c}x,


contradicie.
2)

f (x) < ax

pentru orice

x [b, ).

c = max
x[1,b]
Atunci pentru orice

x [1, )

Denim:

f (x)
< .
x

avem:

f (x) max{a, c}x,


contradicie.
b) Rspunsul este armativ. Alegem un ir 1 = x1 < < xk < astfel ca
(yk )k1 , yk = 2k cos k xk s e de asemenea strict cresctor. Denim f (xk ) =
= yk , k 1 i prelungim f la o funcie liniar pe poriuni f (x) = ak x + bk pentru
x [xk , xk+1 ]. Am obinut astfel o funcie continu f cu proprietatea:

irul

lim

f (x2n )
=
x2n

lim

f (x2n1 )
= 0.
x2n1

Prin urmare funcia continu

g(x) =

f (x)
,
x

ia orice valoare strict pozitiv pe intervalul

x 1,
[1, ).

Soluia 2. b) Exemplul se bazeaz pe imaginea intuitiv a gracului unei


funcii care intersecteaz orice dreapt

D : y = ax

cu

a > 0,

construind o linie

poligonal.
Considerm dreptele
puncte

An (xn , yn ) Dn

1
x
n

Dn0 : y = nx, n 2 i pe ele




1
A0n (x0n , yn0 ) Dn0 , precum i A 1,
, astfel
2
Dn : y =

irurile de
ca irurile

coordonatelor s verice relaiile:

1 < xn < x0n < xn+1 ,

1
< yn < yn0 < yn+1 ,
2

n 2.
315

Condiiile impuse conduc la:

1
1
1
< xn < xn < x0n < nx0n <
xn+1 < xn+1 .
2
n
n+1
Putem lua de exemplu
Denim funcia

xn = (n!)2

x0n = xn + 1.

avnd ca grac linia poligonal

A, . . . , An , A0n , . . . .

Din

condiiile impuse rezult c funcia este strict cresctoare.


Ecuaia

x0n .

f (x) =

1
x
n

are soluie n

Rezult c pentru orice


a


1
,n
n

xn ,

iar ecuaia

ecuaia

f (x) = nx

f (x) = ax

are soluie n

are soluie (se aplic

f (x)
0
Darboux funciei continue g(x) =
pe intervalul [xn , xn ]). Deoarece
x

1
, n = (0, ), ecuaia f (x) = ax are soluie pentru orice a (0, ).
n
O soluie asemntoare a dat n concurs studentul Cristian Tlu.

teorema lui


S

n2

Problema 2. Fie P1 , P2 , . . . , Pk , . . . un ir de poligoane convexe astfel ca


pentru orice k 1 vrfurile poligonului Pk+1 s e mijloacele laturilor poligonului
Pk . S se arate c exist un singur punct situat n interiorul tuturor poligoanelor.

Nazar Agakhanov, Rusia

Soluii. Problema a fost propus de unul dintre cei mai prolici propuntori
de probleme de cel mai nalt nivel. Ea este, n egal msur, legat de geometrie,
analiz matematic i algebr vectorial:
1.

aspectul geometric este evident din enunul problemei (intersecia unor

guri geometrice).
2. partea de analiz matematic se ncadreaz n teoremele de contracie sau
de iruri descendente de mulimi nchise.
,,ntr-un spaiu metric complet un ir descendent de mulimi nchise, cu irul
diametrelor tinznd la zero, au un unic punct comun.
3. aspectul algebric, cel mai puin evident, este coninut n majoritatea soluiilor.

Soluia 1.

Vom lucra n planul complex (se poate lucra i vectorial n

R2 ). Alegem originea planului n centrul de greutate al poligonului P1 i notm


cu a1 , a2 , . . . , an axele vrfurilor poligonului P1 (a1 + a2 + + an = 0). Notm
apoi succesiv cu ak,1 , ak,2 , . . . , ak,n axele vrfurilor poligonului Pk , k 1. Din
relaiile:

ak+1,1 =

ak,1 + ak,2
ak,2 + ak,3
ak,n + ak,1
, ak+1,2 =
, . . . , ak+1,n =
2
2
2

()

rezult c toate poligoanele au centrul de greutate n origine, iar pentru poligonul

316

Pn

obinem axele vrfurilor

an,1 =
an,2 =

1
2n1
1
2n1

n2
1
2
a1 + Cn1
a2 + Cn1
a3 + + Cn1
an1 + an

n2
1
2
a2 + Cn1
a3 + Cn1
a4 + + Cn1
an + a1

............................................................

1
n2
1
2
a1 + Cn1
a2 + + Cn1
an2 + an1 .
an,n = n1 an + Cn1
2
Folosind n toate relaiile condiia

|an,1 |
|an,2 |

1
2n1
1
2n1

a1 + a2 + + an = 0,

avem:





n2
1
2
Cn1
1 |a2 | + Cn1
1 |a3 | + + Cn1
1 |an1 |




n2
1
2
Cn1
1 |a3 | + Cn1
1 |a4 | + + Cn1
1 |an |

.....................................................................




1
n2
1
2
|an,n | n1 Cn1
1 |a1 | + Cn1
1 |a2 | + + Cn1
1 |an2 | .
2
Adunnd aceste relaii obinem:

S(Pn )
unde am notat cu


n1
(2

n)S(P
)
=
1
1
n1

S(Pk ) suma
Pk , k 1 .

n 
2n1

S(P1 ),

distanelor de la origine (centrul de greutate) la

vrfurile poligonului

S se comport ca o contracie (din n 1 n n 1),


q = 1 n1 < 1 i atunci S(Pm(n1)+1 ) q m S(P1 ), n care dac
2
limit cu m obinem:
Funcia

cu raia
trecem la

lim S(Pm(n1)+1 ) = 0,

m
deci:

lim ak,1 = lim ak,2 = = lim ak,n = 0.

k
Aceasta deoarece irul

S(Pk )

este descresctor, cci:

S(Pk+1 ) = |ak+1,1 | + |ak+1,2 | + + |ak+1,n |









ak,1 + ak,2 ak,2 + ak,3
+
+ + ak,n + ak,1
=





2
2
2
1
((|ak,1 | + |ak,2 |) + (|ak,2 | + |ak,3 |) + + (|ak,n | + |ak,1 |))
2
= |ak,1 | + |ak,2 | + + |ak,n | = S(Pk ).

Punctul comun tuturor poligoanelor este atunci punctul la care converg vrfurile poligoanelor (centrul de greutate comun tuturor).

Remarc.1 ) Aceast problem este de fapt din folclor. Ne referim la (Schoenberg,

I.J.,

Priveliti Matematice,

Editura Tehnic, 1989). Fiind dat un poligon (convex)

1 ) Remarca i Soluia 2 sunt introduse de referent.

317

n plan, poligonul 0 format de mijloacele laturilor lui este cunoscut sub numele
0
(m)
de poligon Kasner asociat lui i va  notat prin = K, deci
= Km . I.J.
Schoenberg sugereaz folosirea metodei seriilor Fourier nite ctre rezultatul enunat,
drept care vom face mai nti o scurt prezentare teoretic a acestui subiect, dup

Schoenberg.2 )

S considerm, pentru
rdcinile complexe de ordin
complexe

0 k n 1, numerele complexe zk , i k = e2ik/n ,


n ale unitii. Problema este de a determina numerele

(coecienii Fourier) astfel nct

zk =

n1
X

j kj .

j=0
(Deoarece determinantul sistemului este

Vandermonde,

sistemul are soluie unic,

ns putem obine o precizie mai mare din relaii de ortogonalitate.)


Dar

k = k .

Atunci, pentru

0 ` n 1,

zk `k =

n1
X

vom avea:

j k(j`) ,

j=0
deci:

n1
n1 n1
n1
n1
1 X X k(j`)
1 XX
1X
j k(j`) =
zk `k =
j

= ` ,
n
n
n j=0
j=0
k=0

k=0

k=0

aadar coecienii Fourier exist n mod unic.


Finalmente vom deduce identitatea lui Parseval

n1
X

|` |2 = n

`=0

n1
X
`=0

! n1
n1
n1
X
1 X X
` ` =
zk `k
zj `j =
n
j=0
`=0

k=0

n1 n1 n1
n1 n1
n1
n1
X
X
1 XXX
1 XX
zk zj `jk =
zk zj
`(jk) =
|zk |2 .
n
n
j=0
j=0
`=0 k=0

k=0

`=0

k=0

Soluia 2. n mod evident problema revine la a  date numerele complexe zk


indexate n

Zn

(axele vrfurilor poligonului iniial), a deni

a analiza comportamentul iteratelor

f (zk ).

f (zk ) =

zk + zk+1
,
2

Pentru uurina calculelor, s alegem

originea n centroidul sistemului de puncte (care nici mcar nu trebuie a  n poziie

2 ) Acelai autor folosete aceast metod pentru a demonstra proprietatea izoperimetric:


Teorem (J. Steiner). Dintre toate poligoanele cu n laturi n plan, de acelai perimetru L,
cel de arie maxim

A = L2 /(4n tan

)
n

este poligonul regulat.

Mai mult, enun i demonstreaz:

Teorem (L. Fejes Tth). Pentru m , o imagine an convenabil a lui Km converge

ctre un poligon regulat sau stelat regulat.

318

n1
X

convex), adic

zk = 0.

Atunci, folosind coecienii Fourier:

k=0

f (zk ) =

n1
 1 n1
X

zk + zk+1
1X  j
j
=
j k + k+1
=
j 1 + j kj .
2
2 j=0
2 j=0

Aceasta arat c, coecienii Fourier pentru

j =
unde

= ei/n ,

f (zk )

sunt:

1 + j
j
j = j j cos ,
2
n

de modul unitate. S observm c

0 =

n1
1X
zk 0k = 0,
n

de unde

k=0

0 = 0.

Atunci, pentru

0j n1

|j |2 = |j |2 cos2

|j |2 cos2 ,
n
n

deci, utiliznd identitatea lui Parseval:

n1
X

|f (zj )|2 = n

j=0

n1
X

|j |2 n cos2

j=0

n1
n1
X
X
|j |2 = cos2
|zj |2 .
n j=0
n j=0

Prin iterare se obine:

n1
X

|f m (zj )|2 cos2m

j=0

n1
X
|zj |2 0
n j=0

cnd

m .

Prin urmare iteratele se contract ctre centroidul sistemului de puncte.


Pentru a reveni totui cu picioarele pe pmnt, i a nu crede c avem realmente
nevoie de astfel de metode avansate, iat o soluie alternativ, total elementar, dat
de

Marius Tiba, elev pe atunci n clasa


ABC (cu notaiile obinuite):

a IX-a. Folosind formula medianei ntr-un

triunghi

m2a =

b2 + c2
a2

2
4

obinem prin nsumare:

n
X

d2k+1,i

i=1

X
1X 2
1
=
d2k,i
`k,i
d2k,i
4
4n
i=1
i=1
i=1

n
X

!2
`k,i

i=1

Pk , iar dk,i sunt distanele de la centroid la


G, ar exista un alt punct X comun interioarelor
tuturor poligoanelor, perimetrele lor ar  toate mai mari dect GX , deci prin iterarea
unde

`k,i

n
X

sunt lungimile laturilor lui

vrfuri. Dac, n afar de centroidul


relaiei de mai sus:

n
X
i=1

d2k+1,i <

n
X
i=1

d21,i

k
GX 2 ,
4n
319

absurd pentru

sucient de mare.

Soluia 3. Din relaiile

(),

1
0
0

A= .
2 ..

0
1

notnd cu

1
1
0

0
1
1

0
0
0

.
.
.

.
.
.

..

.
.
.

0
0

0
0

1
0

matricea:

0
0
0

. Mn (R)
.
.
1
1

i cu:

ak,1
ak,2

Xk = . ,
..
ak,n
obinem relaia de recuren

Xk+1 = A Xk ,
din care

Xk+1 = Ak X1 , Qk 1.
Matricea

este o matrice circular


A=C


1 1
, , 0, . . . , 0
2 2

i = f (i ), i = 0, n 1,

cu valorile proprii

f (x) =
i

0 , 1 , . . . , n1

unde:

1 1
+ x
2 2

n ale unitii. Avem:




1 + i
< 1, i = 1, n 1.
|i | =
2

sunt rdcinile de ordin

0 =

1 1
+ = 1Qi
2 2

Orice matrice circular este diagonalizabil, iar matricea de pasaj este matricea Vandermonde:

P = V (0 , 1 , . . . , n1 ),
deci:

D = P 1 A P

A = P D P 1 ,

sau

unde:

D=

320

1
0
0

0
1
0

0
0
2

0
0
0

.
.
.

.
.
.

.
.
.

..

.
.
.

n1

D =

1
0
0

0
k1
0

0
0
k2

0
0
0

.
.
.

.
.
.

.
.
.

..

.
.
.

kn1

Ak = P Dk P 1 obinem:

1 0 0 0
0 0 0 0

lim Ak = P 0 0 0 0 P 1 .
.. .. .. . .
k
.
.
. . .
.
.
0 0 0 0

Trecnd la limit n relaia

Deoarece

1
1
V (0 , 1
1 , . . . , n1 )
n

P 1 =
obinem:

1 1
1 1
1

lim Ak = . .
k
n .. ..
1 1

..

1
1

.
.
.
1

i deci

1 1

1 1 1
lim Xk = . .
k
n .. ..
1 1
unde

..



b1
a1
1
a 2 b2
1


. . = .
. .
..
.
.
bn
an
1

a1 + a2 + + an
b1 = b2 = = bn =
n

(centrul de greutate al poligonului

iniial).

(k0 )k1 , (k1 )k1 , . . . , (kn1 )k1 rezult convergena irului de matrice (A )k1 , i implicit a irului vrfurilor (Xk )k1
ale poligoanelor. Din relaia Xk+1 = A Xk obinem X0 = A X0 , cu X0 = lim Xk ,
Observaie. Din convergena irurilor
k

deci

(A 1In ) X0 = 0.
Rezult c vectorul limit
valorii proprii

0 = 1.

X0

este vector propriu pentru

A,

corespunztor

Prin calcul direct

1
1

X0 = .
..
1
este un vector propriu cu toate componentele egale. Acest raionament simplic
Soluia 3.
Soluia 3 a fost dat (incomplet) doar de doi studeni din concurs, i ace-

tia din Romnia (Octavian

Ciprian Opria

Ganea

de la Universitatea Politehnic din Bucureti i

de la Universitatea Tehnic din Cluj).

Problema 3.

S se determine toate funciile surjective:


f : Mn (R) {0, 1, . . . , n}
321

cu proprietatea f (X Y ) min{f (X), f (Y )}, pentru orice X, Y Mn (R).

Vasile Pop, Romnia

Soluie. Problema conine o caracterizare a rangului unei matrice printr-o


inecuaie funcional.
Funcia

X GLn (R)

f (X) = rang(X) veric


Y Mn (R) atunci:

condiia. Vom arta c ea este unic. Dac

f (X Y ) f (Y )
f (X Y ) = f (Y )
X GLn (R).
deci

f (Y ) = f (X 1 (X Y )) f (X Y ),

f (Y X) = f (Y ),

i analog

pentru oriceY

Mn (R)

Y Mn (R) de rang
 k poate adus prin transformri
Ik 0
, deci exist matricele X
elementare pe linii i coloane la matricea Jk =
0 0
i Z din GLn (R) astfel ca Y = X Jk Z . Din cele de mai sus rezult f (Y ) = f (Jk ).
Este sucient s denim funcia f pe matricele Jk , k = 0, n. Din Jk Jk+1 = Jk
rezult f (Jk ) f (Jk+1 ) i folosind surjectivitatea funciei f rezult f (J0 ) = 0,
f (J1 ) = 1, . . . , f (Jn ) = n. Deci f (Y ) = rang(Y ), pentru orice Y Mn (R).
Se tie c orice matrice

Problema 4.

cu proprietatea:

Fie n un ntreg strict pozitiv i f : [0, 1] R o funcie continu


Z1

xk f (x)dx = 1

pentru orice k {0, 1, . . . , n 1}. S se arate c :


Z1

f (x)2 dx n2 .

Mircea Dan Rus, Romnia


Soluie. Domeniul general al problemei este cel al spaiilor vectoriale dotate
cu produs scalar

(V, h, i)

vectorii liniar independeni

(spaii de tip prehilbertian sau euclidian).

v1 , v2 , . . . , vn

se cere s se determine vectorul

Fiind dai

de norm

minim care veric condiiile

hv, v1 i = 1 ,
unde

1 , 2 , . . . , n

hv, v2 i = 2 ,

...,

hv, vn i = n ,

sunt scalari xai. Cu un raionament general se poate ajunge

la concluzia c vectorul care realizeaz minimul este unic i se a n spaiul generat


de vectorii

v1 , v2 , . . . , vn

(Lema 1, cu demonstraie la sfritul soluiei problemei),

spaiu care n cazul nostru este cel al polinoamelor de grad cel mult

n 1.

Problema s-a dovedit a  cea mai dicil din concurs i dicultatea const n
calculul efectiv al normei minime.

1)

1 ) Totui faptul c valoarea normei minime este atins nu se cerea n problem. (N.A.)

322

Exist

1 ) polinomul unic

Z1

p(x) = a1 + a2 x + + an xn1

xk p(x)dx = 1,

care satisface relaiile:

0 k n 1.

(1)

0
Prin urmare pentru orice

Z1

k {0, 1, . . . , n 1}

avem:

xk (f (x) p(x))dx = 0,

0
i de aici

Z1
p(x)(f (x) p(x))dx = 0.
0
Avem atunci

Z1

Z1

(f (x) p(x)) dx =
0

f (x)(f (x) p(x))dx =


0

Z1
=

f (x)2 dx

0
de unde rezult:

Z1

n1
X

Z
ak+1

xk f (x)dx,

k=0

f (x)2 dx a1 + a2 + + an .

0
Artm c

a1 + a2 + + an = n2 .

Relaiile (1) pot  scrise sub forma

a2
an
a1
+
+ +
= 1,
k+1 k+2
k+n

0 k n 1.

Rezult c funcia raional:

r(x) =
are rdcinile

a2
an
a1
+
+ +
1
x+1 x+2
x+n

0, 1, . . . , n 1.

Avem:

r(x) =

q(x) (x + 1)(x + 2) (x + n)
,
(x + 1)(x + 2) (x + n)

q este un polinom de gradul n 1. Coecientul lui xn1 n q este egal cu


a1 + a2 + + an . Numrtorul lui r are rdcinile 0, 1, . . . , n 1, de unde rezult
unde

c:

q(x) = (x + 1)(x + 2) (x + n) x(x 1) (x (n 1)).


1 ) Vezi Lema 1, i nalul soluiei. Relum totui argumentarea minimalitii. (N.A.)

323

Din ultima expresie a lui

rezult c coecientul lui

xn1

este:

n(n + 1) n(n 1)
+
= n2 .
2
2
Obinem:

a1 + a2 + + an = n2
i mai mult, egalitate dac i numai dac

f = p.

S observm c scrierea n fracii simple a funciei raionale:

x(x 1) (x (n 1))
(x + 1)(x + 2) (x + n)

furnizeaz chiar coecienii polinomului

p,

deci aceasta este o alt metod de a-i

proba existena.

Demonstraia Lemei 1. S notm cu Vk subspaiul generat n V de vectorii


liniar independeni (deci nenuli)

v1 , v2 , . . . , vk ,

pentru

1 k n.

n = 1 rezultatul
v = (1 /||v1 ||2 )v1 . Fie u Vn1 dat de ipoteza de inducie

pentru n 1, i w Vn , ortogonal lui Vn1 . Avem hw, vn i =


6 0, cci altfel w ar 
n hu, vn i
,
ortogonal spaiului Vn n care se a, dar el este nenul. Fie atunci =
hw, vn i
i v = u + w , deci v Vn . Dar atunci hv, vk i = hu, vk i + hw, vk i = k , pentru
1 k n 1, i hv, vn i = hu, vn i + hw, vn i = n .
0
Unicitatea provine din faptul c pentru un alt v Vn , i cu proprietile
0
0
cerute, am avea v v ortogonal spaiului Vn n care se a, deci nul, deci v = v .
n ce privete minimalitatea, e f V oarecare cu proprietile cerute, deci
f v este ortogonal spaiului Vn , n care se a vectorul v . Atunci:
Pentru probarea existenei procedm prin inducie. Pentru

este trivial; lum

||f ||2 = ||v + (f v)||2 = hv + (f v), v + (f v)i =


= ||v||2 + 2hv, f vi + ||f v||2 = ||v||2 + ||f v||2 ||v||2 ,
f v = 0, deci f = v .
1, x, . . . , xn1 sunt evident
unic p(x) de grad cel mult n 1, cu

cu egalitate numai pentru

n cazul de fa, monoamele


deci exist un polinom
adic:

Z1

hp(x), x i =

xk p(x)dx = 1,

liniar independente,
proprietile cerute,

0 k n 1,

i de norm

||p(x)||2 =

Z1

p(x)2 dx minim printre toate funciile continue cu propri-

0
etatea de mai sus.

Univ. Tehnic din Cluj-Napoca


Str. C. Daicoviciu 15,
400020, Cluj-Napoca, Romnia
vasile.pop@math.utcluj.ro

324

Univ. Tehnic din Cluj-Napoca


Str. C. Daicoviciu 15,
400020, Cluj-Napoca, Romnia
popa.dorian@math.utcluj.ro

Subiectele date la Universitatea din Bucureti,


Facultatea de Matematic i Informatic
Concursul de admitere iulie 2008
Domeniul de licen  Informatic
I. Algebr. 1. Fie matricea:


A=

1
0

1
2


M2 (R).

A2 = 3A 2I2 , unde:


1 0
I2 =
.
0 1

a) S se arate c

A este inversabil i s se calculeze A1 .


n

c) S se calculeze A , n N .
2
d) S se determine matricile X M2 (R) astfel nct X = A.
b) S se arate c

2. Pentru orice

nZ

considerm funcia

fn : (2, ) (2, ),
n

fn (x) = 2 + (x 2)2 .
S se arate c:
a)

fm fn = fm+n pentru rice m, n Z.


G = {fn | n Z} mpreun cu operaia de compunere a funciilor

b) Mulimea

este grup comutativ.


c) Grupul

(G, )

este izomorf cu grupul aditiv

II. Analiz. 1. Se consider funcia


a) S se calculeze

(Z, +)

al numerelor ntregi.

f : [1, ) R, f (x) =

f 0.

b) S se ae maximul funciei

ln x
.
x

f.

(xn )n3 , xn = n1/n .



x,
x [1, 0]
2. Se consider fucia g : [1, 1] R, g(x) =
sin x, x (0, 1].
a) S se studieze continuitatea funciei g .
b) S se studieze derivabilitatea funciei g .
c) S se arate c funcia g admite primitive, s se ae o primitiv a
Z1
calculeze
g(x)dx.
c) S se studieze monotonia irului

ei i s se

III. Geometrie. 1. Fie


cercuri

C(O1 , R1 )

C(O2 , R2 )

C(O, R)

cercului xat. S se arate c triunghiul

2. n planul

xOy ,

un cerc xat de centru

i raz

i dou

tangente exterior ntre ele i ambele tangente interior

OO1 O2

se consider ecuaia

are perimetru constant.

x2 + y 2 6x + 4y 12 = 0.
325

a) S se arate c aceast ecuaie rerezint un cerc cruia s i se determine


raza i coordonatele centrului.
b) S se arate c dreapta de ecuaie

12x 5y + 19 = 0

este tangent la cerc

i s se determine coordonatele punctului de tangen.


c) S se scrie ecuaiile laturilor ptratului circumscris cercului, n care una
dntre laturi este pe dreapta de la punctul b).

IV. Informatic. 1. Se numete subsecven a unui vector


ntregi un vector cu cel puin un element i cel mult

V.

poziii consecutive n vectorul

i vectorul

cu

n elemente

elemente care se gsesc pe

S se scrie un program n

citete de la tastatur numrul natural


aeaz subsecvena lui

avnd

Pascal/C/C ++ care
n elemente ntregi i

avnd suma elementelor maxim.

2. Se citesc de la tastatur numu arul natural

a1 , a2 ,

...,

an .

S se scrie un program n

n i irul numerelor naturale


Pascal/C/C ++ care aeaz indicii i i j

care nceplinesc simultan condiiile:


a)
b)
c)

1 i < j n;
ai > ak i aj > ak , pentru orice k , i + 1 k j 1;
diferena j 1 este maxim .

3. S se calculeze complexitatea timp a programelor propuse pentru problemele de mai sus. n cazul n care niciuna dintre soluiile propuse nu are cmplexitate
liniar , s se scrie un program

Pascal/C/C ++ de complexitate timp liniar pentru

una din cele dou prob leme de mai sus, la alegere.

Precizri. Pentru toate problemele de mai sus se presupune c datele sunt


valabile i

3 n 1000.

Se vor descrie informal detaliile implementrii oricrui

program: semnicaia variabilelor; a structurilor de date, a blocurilor de program,


a condiiilor.

Timp de lucru: 3 ore.


Concursul Studenesc Traian Lalescu  Tradiii i
Moderniti
de Andrei Halanay

Iniiat n anii '70 din secolul trecut, concursul studenesc de matematic


,,Traian Lalescu devenise un moment important n viaa universitar (de rezultatele obinute ajungea s depind chiar numrul de locuri repartizate universitarilor).

ntrerupt o lung perioad dup 1990, concursul a fost reluat la nivelul

Universitii Politehnice din Bucureti (U.P.B.), iar doi ani mai trziu faza local a
fost urmat de una interuniversitar cu participarea la nceput a UPB i a Academiei
Militare (A.T.M.), acestora alturndu-li-se, n anii rmtori, Universitatea Tehnic
de Construcii (U.T.C.). Beneciind de sprijinul nanciar al Ministerului Educaiei
i Cercetrii, n anul 2008 a fost organizat, din nou, o etap naional. Au participat studeni de la 12 universiti: U.P.B., Universtatea din Bucureti, A.T.M.,
U.T.C., Universitatea Babe-Bolyai din Cluj, Universitatea de Vest din Timioara,

326

Universitatea Al. I. Cuza din Iai, Universitatea Tehnic Gh. Asachi din Iai, Universitatea Constantin Brncui din Tg. Jiu, Academia Nava Mircea cel Btrn din
Constana, Universitatea Maritim din Constana.
Spre deosebire de etapele anterioare ale concursului naional, n spiritul nevoii
de modernizare a nvmntului superior romnesc, n acest an a fost introdus n
cadrul concursului i activitatea de cercetare tiinic studeneasc.

Structurat

pe 3 seciuni (I: Analiz, Algebr, Geometrie; II: Ecuaii difereniale i Matematici


aplicate i III: Informatic) sesiunea a cuprins peste 30 de comunicri tiinice
studenec sti. Dintre acestea s-au remarcat cele ale studenilor

Alexandru Tomescu

de la Facultatea de Matematic, Universitatea din Bucureti: ,,Sortarea bitonic cu


modele naturale de calcul, coonductor tiinic conf. dr.

R. Ceterchi, Cezar Lupu

de la Facultatea de Matematic, Universitatea din Bucureti: ,,Remarci asupra unei


inegaliti elementare echivalent cu ipoteza lui Riemann, conductor tiinic prof.
dr.

Liviu Ornea, Alin Galatan

de la Facultatea de Matematic, Universitatea din

Bucureti: ,,Fractaliconductor tiinic prof. dr.

Radu Gologan, Claudia Zaharia,

Facultatea de Matematic, Univ. de Vest, Timioara: ,,Asupra teoremelor lui Aoki


i Rassias de stabilitate a ecuaiior funcionale, conductor tiinic prof. dr.

Radu, Alexandru Szoke

V.

de la Facultatea de Matematic, Universitatea din Bucureti:

,,Aspecte ale implementrii reelelor de procesoare biologice, conductor tiinic


prof. dr.

V. Mitrana.

Concursul a beneciat de sponsorizri din partea Inspectoratului colar al


Municipiului Bucureti i din partea Societii de tiine Matematice din Romnia.
Fundaia ,,Traian Lalescu, prin participarea direct a doamnei

Smaranda Lalescu,

nepoata marelui matematician, a avut o contribuie important la reuita manifestrii prin acordarea unui premiu de excelen (ctigat de

Alexandru Tomescu ) i

acordarea de diplome celorlali premiani, nsoite de medalia comemorativ ,,Traian


Lalescu  125 de ani de la natere i de volumul ,,Traian Lalescu  un nume peste
ani.
Succesul acestei manifestri naionale nu ar  fost posibil fr participarea
entuziast i dezinteresat a numeroase cadre didactice. De menionat aportul dom-

O. Stnil, membru al juriului la seciunea I i preedintele juriului


S. Dsclescu, prof. dr. V. Blan, conf. dr. C. Gherghe,
membrii n juriul seciunii I, prof. dr. I. Roca, prof. dr. V. Rasvan, conf. dr.
R. Constantin, membrii n juriul seciunii II, prof. dr. A. Atanasiu, conf. dr. V.
Rasvan, conf. dr. M. Olteanu, conf. dr. R. tefan, membrii n juriul seciunii III,
prof. dr.Radu Gologan, prof. dr. V. Prepeli, lect. dr. A. Toma, conf. dr. V.
igoiu implicai n detaliile organizatorice.
nilor prof. dr.

de contestaii, prof. dr.

Trebuie menionat, de asemenea, rolul U.P.B., al doamnei rector prof. dr. ing.

Ecaterina Andronescu,

fr sprijinul creia acest concurs nu ar  putut  organizat.

Festivitatea de acordare a premiilor, beneciind de prezena doamnei rector

E. Andronescu,

V. Brnznescu, director al Institutului de MatemaC. Udrite, decan al Facultii de tiine


Aplicate din U.P.B., a domnului D. tefan, decan al Faultii de Matematic din
Universitatea din Bucureti, a domnului I. Ciuc, director n Ministerul Educaiei
i Cercetrii, a domnului D. tefnescu, vicepreedinte al S.S.M.R, s-a constituit
a domnului

tic al Academiei Romne, a domnului

ntr-o veritabil pledoarie pentru excelena n nvmntul superior romnesc, cu

327

precdere n predarea i nvarea matematicii.


Pentru anii urmtori se va urmrii perfecionarea modului de organizatre,
mrirea numrului universitilor participante i creterea calitii i a numrului
comunicrilor tiinice prezentate.

Concursul Traian Lalescu  Faza naional


Mai 2008
Sesiunea de comunicri tiinice ale studenilor
Seciunea 1: Analiz, Algebr, geometrie

O. Snil FSA, UPB


S. Dsclescu FMI, UB
Conf. dr. C. Gherghe FMI, UB
Prof. dr. V. Blan FSA, UPB.

Juriu: Prof. dr.

Prof. dr.

Premiul I

Cezar Lupu,

anul IV, Facultatea de Matematic i Informatic, ,,Remarci

asupra unei inegaliti elementare echivalent cu ipoteza lui Riemann


Coordonator: Prof. dr.

Liviu Ornea

Premiul II

Cezar Lupu, Cosmin Pohoa,

anul IV, Facultatea de Matematic i Infor-

matic, ,,O ranare a inegalitii HadwigerFisher


Coordonator: Prof. dr.

Liviu Ornea

Premiul III

Marius Bucur,

Facultatea de Automatic i Calculatoare, ,,Teoria grupurilor

n analiza algoritmilor de criptare


Coordonator: Conf. dr.

Radu Constantin

Seciunea 2: Ecuaii i Matematici Aplicate

I. Roca FMI, UB
V. Rasvan Facultatrea de Automatic., Univ. din Craiova
dr. R. Constantin Facultatea de tiine aplicate, U.P.B.

Juriu: Prof. dr.

Prof. dr.
Conf.

Premiul I

Alin Galatan,

anul II, Facultatea de Matematic, Univ. Bucureti, ,,Fractali


Coordonator: Prof. dr.

Premiul II

328

Radu Gologan

Claudia Zaharia,

anul IV, Facultatea de Matematic, Univ. de Vest, Timi-

oara, ,,Asupra teoremelor lui Aoki i Rassias de stabilitate a ecuaiior funcionale

V. Radu

Coordonator: Prof. dr.

Premiul III

Simona Roxana Costache, Elena Biri,

anul III, Facultatea de tiine Apli-

cate, UPB, ,,Metoda Monte Carlo i aplicaii

V. Trcolea

Coordonator: Prof. dr.

Alexandra Podiuc,

anul III, FILS;

George Olaru,

anul II, Facultatea de Auto-

matic, UPB: ,,2D continuous-Discrete Laplace Transfomrations and Applications


Coordonator: Prof. dr. V. Prepeli ; Asist. dr. M. Prvan
Rzvan Ionescu, anul II, FILS; Andrei Chiril, anul II, FILS, UPB: ,,Stability
of linear sstem  Math. speialized software

V. Prepeli

Coordonator: Prof. dr.

Seciunea 3: Informatic

A. Atanasiu FMI, UB
V. Rasvan Facultatea de Automatic., U. P. B.
dr. M. Olteanu Facultatea de tiine aplicate, U.P.B.

Juriu: Prof. dr.

Conf. dr.
Conf.

Premiul I i Marele Premiu al Fundaiei ,,Traian Lalescu

Aexandru Tomescu,

anul IV, Facultatea de Matematic i Informatic, Univ.

Bucureti, ,,Sortarea Bitonic cu modele naturale de calcul

R. Ceterchi

Coordonator: Conf. dr.

Premiul II

Alexandru Szoke,

anul IV, Facultatea de Matematic, , ,,Aspecte ale imple-

mentrii reelelor de procesoare biologice


Coordonator: Prof. dr.

V. Mitrana

Premiul III

Daniel Cernat,

anul II, Master Catedra Matematici III, UPB, ,,Tehnici de

watermarking n procesarea imaginilor digitale


Coordonator: Prof. dr.

Alexandra Rdoi,

V. Blan

anul II, Facultatea ETTI, UPB: ,,Analiza Wavelet.

Apli-

caii: Recunoaterea formelor


Coordonatori: Prof. dr.

V. Lzrescu ;

Prof. dr.

O. Stnil

Meniune

Liviu Drgan, Cezara Florescu,

anul I, Facultatea ETTI, UPB: ,,Aplicaii ale

geometriei fractale n domeniul analizei informaiei video


Coordonator: Conf. dr.

O. andru
329

Concurs de rezolvat probleme


Seciunea A: Matematic
Premiul I:

Turea Lucian

 Univ. Bucureti;

Bogoel Beniamin

 Univ. de

Vest Timioara;

Vlad Emanuel  Univ. Bucureti; Galatan Alin  Univ. BuPopa Tiberiu  Univ. Babe-Bolyai Cluj;
Premiul III: Gavrus Cristian  Univ. Bucureti; Stratulat Ioan Tudor 
Univ. Tehnic Gh. Asachi, Iai; Papar Nicolae  Univ. Babe-Bolyai Cluj; Vlculescu Adrian Claudiu  Univ. Babe-Bolyai Cluj;
Meniuni: Bucur Marius  Univ. Politehnica din Bucureti; Farca Csaba 
Premiul II:

cureti;

Univ. Babe-Bolyai Cluj.

Seciunea B: Prol electric, Anul I


Premiul I: Opria Ciprian  Univ. Tehnic Cluj;
Premiul II: Florescu Dorian  Univ. Tehnic Gh. Asachi, Iai.
Premiul III: Nagy Aliz-Eva  Univ. Tehnic Cluj;
Meniuni: Ferchiu tefan  Univ. Politehnica din Bucureti; Apostol
 Univ. Politehnica din Bucureti;

Ciobotaru-Hriscu Iulian

tefan

 Acad. Tehnic Militar

din Bucureti.

Seciunea B: Prol electric, Anul II

Bogdea Lavinia  Univ. C. Brncui din Tg. Jiu;; Boia Rodica 


Carp Andrei  Univ. Tehnic Gh. Asachi, Iai;
Premiul II: Alecu Adrian  Univ. Tehnic Gh. Asachi, Iai;Podiuc AlexanPremiul I:

Univ. Politehnica din Bucureti;

dra

 Univ. Politehnica din Bucureti;

Premiul III: Popescu Ionu  Univ. C. Brncui din Tg. Jiu;


Meniuni: Miu Tudor  Univ. Politehnica din Bucureti; Stniloiu

 Univ. C. Brncui din Tg. Jiu;

Daca Adina

Roxana

 Univ. Politehnica din Bucureti;

Seciunea C: Prol mecanic, Anul I


Premiul I: Savastre
Premiul II: Acsinia

Andrei

Simona  Univ. Tehnic de Construcii, Bucureti;


Elena  Univ. Tehnic de Construcii, Bucureti; Crje

 Univ. Tehnic de Construcii, Bucureti;

Premiul III: Cojocaru Ruxandra  Univ. Politehnica din Bucureti;


Meniuni: Spiridon Vasile Acad. Naval Mircea cel Btrn, Constana.
Seciunea C: Prol mecanic, Anul II
Premiul I: Frumosu Flavia  Univ. Politehnica din Bucureti;
Premiul II: Popescu Alexandra  Univ. Politehnica din Bucureti;
Premiul III: Turi-Damian Sorin  Univ. Politehnica din Bucureti;
Meniuni: Cazan Costic  Univ. Tehnic Gh. Asachi, Iai; Crasmaru

 Univ. Tehnic Gh. Asachi, Iai.

330

Ionu

Concursul de matematic ,,Traian Lalescu,


Faza naional 2008, seciunea Matematic (A)
1. Fie matricea
dac tr(A

) = 0,

A Mn (C). S se arate c A este nilpotent


k N ? ; (tr(A) este urma matricei A).

dac i numai

oricare ar 

2. Fie

E o mulime nevid a intervalului (0, )


x
E , oricare ar  x E .
2p
x2 + y 2 E , oricare ar  x, y E .
(ii)

care indeplinete condiiile:

(i)

Se cere:

E 6= (0, )

(a) S se dea un exemplu de multime

care indeplinete condiiile

(i) si (ii).
(b) S se arate c

3. Fie
e

U R

f : U 7 R

E = [0, ); (E
C1



f

(P ) 1
x

S se arate c dac

O,

E ).

o submulime deschis care conine discul unitate nchis

o funcie de clas

trul de greutate n

este nchiderea topologic a lui

cu proprietatea:



f

(P ) 1,
y

{M1 , M2 , ..., Mn }

P D.

este o mulime de puncte din

P D este


n


1 XX


f (Mk ) 2.
f (P )


n

atunci pentru orice punct

cu cen-

adevarat inegalitatea:

k=1

4.

mulimea plan format din punctele interioare i laturile unui


ABCD de laturi AB = a i BC = b. Se denete funcia f : 7 R

Fie

dreptunghi
prin:

f (P ) = P A + P B + P C + P D.
S se calculeze mulimea valorilor funciei

f.

331

NOTE MATEMATICE
Legturi neateptate
de Marian Tetiva

Abstract
Key words:
M.S.C.: .
1. Fie

a, b, c

numere reale i s considerm polinomul:

f = X 2 (a2 + b2 + c2 )X + 2abc,
despre care armm c are toate rdcinile reale.

Dar un polinom de gradul al

treilea cu toate rdcinile reale are discriminantul nenegativ, deci (discriminantul


unui polinom de forma

X 3 + pX + q

este

4 a2 + b2 + c2

4p3 27q 2 )
3

obinem:

27(2abc)2 0,

care se transform imediat n:

a2 + b2 + c2

3

27(2abc)2 .

Cu alte cuvinte, am gsit exact inegalitatea mediilor (pentru numerele nenegative

a2 , b2 , c2 )!
2. Polinomul

a, b, c

de mai sus ne mai rezerv o surpriz. Anume s presupunem

sunt astfel nct:

a2 + b2 + c2 + 2abc = 1.
f (1) = 0, deci
x2 , avem atunci:

Aceast egalitate conduce la


Pentru celelalte dou, e ele

x1

x1 + x2 = 1 mi
deci inegalitatea

(x1 + x2 ) 4x1 x2

o rdcin a lui

este

1.

x1 x2 = 2abc,

(care exprim tot un discriminant

0,
x2 )

anume

discriminantul polinomului de gradul al doilea care are rdcinile

x1

1 8abc.

care ndeplinesc

Aadar, am artat c pentru orice numere reale

a, b

devine

A
a2 + b2 + c2 + 2abc = 1 are loc inegalitatea 8abc 1. Pentru a = sin ,
2
B
b = sin , c = sin C2, unde ABC este un triunghi oarecare (se tie c sinusurile
2
condiia

jumtilor unghiurilor veric aceast relaie) regsim inegalitatea:

sin
332

A
B
1
sin sin C2 ,
2
2
8

care, cum bine se cunoate, este echivalent cu inegalitatea lui

Euler, R 2r!

3. Raionamentele de mai sus se bazeaz esenial pe faptul c

are rdcini

reale, fapt pe care nu vrem s-l justicm utiliznd discriminantul, pentru a putea
apoi deduce inegalitatea mediilor. Atunci cum artm c

are rdcini reale? Ei

bine, s considerm matricea cu elementele numere reale:

0
A= a
b

a b
0 c .
c 0

Fiind o matrice real i simetric, polinomul ei caracteristic are toate rdcinile


reale. Dar, fr nicio problem, se calculeaz polinomul caracteristic al acestei matrici, se gsete, desigur, tocmai

i astfel se ncheie raionamentul nostru.

Ar  interesant de construit o matrice al crei polinom caracteristic s aib discriminantul pozitiv, iar inegalitatea pentru acest discriminant s ne dea inegalitatea
mediilor (sau poate o alt inegalitate cunoscut). Din pcate ncercrile noastre n
acest sens nu au dat niciun fel de roade.

Profesor,
Colegiul Naional Gheorghe Roca Codreanu
din Brlad
PROBLEME PROPUSE

n, n + 1 se numete adaptat,
i
h
i

(n + 1) 3 n 3 = 1.

269. O pereche de numere naturale consecutive


dac:

Care este probabilitatea ca dou numere consecutive alese la ntmplare, s


e adaptate.

Radu Gologan
270. Fie

a, b, c

numere pozitive al cror produs este egal cu 1; mai pre-

supunem c:

c = min{a, b, c}.
S se arate c:

a3 + b3 + c3 3(ab + ac + bc) c(a b)2 + c(a c)(b c).

Marian Tetiva
271. S se arate c pentru orice

GR

are loc inegalitatea:

X
cos n
7 4

.
(n + 1)4
720
n=0

Rbert Szsz
272. Fie

xi R+ , i = 1, n,

astfel nct:

333

0 x1 x2 . . . xn i x1 + x2 + . . . + xn = a.


S se determine mulimea max xi | i = 1, n i s se precizeze valorile pentru
care aceste maxime sunt atinse.

Dorin Mrghidanu

273. Dac:


e(x) =

a R+ ,

1
x

1+

x
x R

s se calculeze:

lim

lim

 x 2
n

n
X

!!!
e(x + ka) ne(x)

k=1

Dumitru Btineu-Giurgiu
274. Fie

dou numere strict pozitive, iar

Considerm irul de polinoame

(Fn )n>k

un numr natural dat.

denit prin:

Fn (X) = X n Rk X nk

j X nj .

1jn
j6=k
1. S se arate c ecare polinom
2. S se arate c irul

(n )n>k

Fn

are o unic rdcin real pozitiv

este monoton i convergent.

n .

Doru tefnescu

SOLUIILE PROBLEMELOR PROPUSE


Dup intrarea la tipar a numrului 3/2008 al revistei, am mai primit soluiile corecte la
problemele 243, 245, 246 i 247 de la domnul Gheorghe B. G. Niculescu  profesor la Colegiul
de Pot i Telecomunicaii Gheorghe Airinei din Bucureti. Facem prezenta meniune pentru ca
domnia sa s poat  inclus pe lista rezolvitorilor de probleme din acest an.
De asemenea, domnul ing. dr. Dorel Bian de la Romtelecom S.A., Bucureti, ne-a
comunicat o scurt i elegant soluie pentru pct. b) al problemei 242 din nr. 2/2008 al revistei,
soluie pe care o reproducem mai jos.
La soluia oferit de dl. prof.
Tetiva n G. M. A. nr. 2/2008, pag. 149, n locul
 Marian


iegalitii lui Blundon p 2R + 3 3 4 r, ridicat la ptrat, se poate utiliza o inegalitate mai


puternic, inegalitatea lui Gerretsen p 4R2 + 4Rr + 3r 2 .


 i2
2R + 3 3 4 r , n care
inegalitatea din dreapta este echivalent cu inegalitatea lui Euler, R 2r 0.
Se demonstreaz c inegalitatea p 4R2 + 4Rr + 3r 2

Problema 242 b) se rezolv scriind dubla inegalitate:

p2 4R2 + 4Rr + 3r2

2(4R + r)2 (2R r)


11R 4r

i demonstrnd inegalitatea din dreapta. Avem:


2(4R + r)2 (2R r) 4R2 + 4Rr + 3r2 (11R 4r)

64R3 12Rr2 2r3 44R3 + 28R2 r + 17Rr2 12r3 (R 2r) 20R2 + 12Rr 5r2 0.

334

n baza inegalitii lui Euler R r 0, n membrul stng din ultima inegalitate avem un
produs de doi factori, unul pozitiv, R 2r , iar cellalt strict pozitiv,

20R2 + 12Rr 5r2 = 20R2 + 12r(R 2r) + 19r2 > 0,


ceea ce ncheie demonstraia.
248. Fie C k (R) spaiul vectorial al funciilor reale de variabil real, difereniabile de k

ori
i

(unde, C (R)  spaiul funciilor indenit derivabile, iar C 0 (R)  spaiul funciilor continue )

L, D : C 2 (R) C o (R)

operatorii difereniali denii prin

L(y) = y 00 + qy 0 + ry,
kerL.

D(y) = y 0 ,

q, r R.

a) S se arate c kerL C (R) i c restricia lui D la kerL este un endomorsm al lui


b) Fie {y1 , y2 } o baz n kerL; pentru orice y kerL, exist a, b R unici, astfel nct

y = ay1 + by2
endomorsmul

i deci, n felul acesta, se denete un izomorsm : kerL R2 . S se arate c


D induce un endomorsm f LR (R2 ) care face urmtoarea diagram comutativ :

kerL

R2

kerL

R2

S se determine o condiie necesar i sucient pentru ca f s e un automorsm.


c) Dac Af este matricea lui f n raport cu baza canonic, iar p este polinomul caracteristic
al ecuaiei L(y) = 0, s se calculeze p(Af ).
d) Alegnd n kerL o baz convenabil, s se scrie forma general a lui Af . Cazuri particulare :
(i) q = 0, r = 1;
(ii) q = r = 1;
(iii) q = 2, r = 1.

Dan Radu
Soluia autorului. Dau a y ker L, atunci y 00 = (qy 0 + ry). Cum y C 2 (R), urmeaz

c (qy 0 + ry) C 1 (R), deci y 00 C 1 (R) i deci y C 3 (R). Raionnd recursiv, deducem c
y C (R). Pe de alt prte, dac y ker L, atunci:

y 00 + qy 0 + ry = v.
Conform celor stabilite anterior, putem deriva inegalitatea de mai sus i obinem:

y 000 + qy 00 + ry 0 = v,
y0

ceea ce ne arat c + D(y) ker L, adic a faptul c restricia lui D la ker L este un endomorsm
al acesteia.
b) S observm mai nti, c izomorsmul este denit prin egalitatea (y) = (a, b).
Probarea acestui fapt este imediat. Deoarece este un izomorsm, rezult c exist 1 : R2
ker L i deci aplicaia f cerut n enun va  f : D 1 . Evident, f este un endomorsm
al lui R2 i face comutativ diagrama dat. Pe de alt parte, s observm c ntre matricile
corespunztoare (scrise respectiv n baza {y1 , y2 } din ker L i baza canonic din R2 ) exist relaia:

Af = A AD A1 .
1
A

Dar A = A1 =
= I (n raport cu bazele considerate) i deci Af = AD . Prin
urmare, o condiie necesar i sucient pentru ca f s e automorsm este ca D s e automorsm.
Avnd n vedere c ker L este nit dimensional, acesta este echivalent cu faptul c ker D = {0}.
Cum ns ker D = R = Sp(1), rezult c f este automorsm dac i numai dac 1
/ ker L.
c) Din cele stabilite la punctul b), rezult c p(Af ) = p(AD ) i deci:

p(Af ) = A2D + qAD + rI = AD2 +qD+ridker L .


Pe de alt parte, pentru i {1, 2}, avem:


D2 + qD + idker L (yi ) = D2 (yi ) + qD (yi ) + ryi = yi00 + qyi0 + ryi = v,
aa nct p(Af ) = 0 (matricea nul).
d) S considerm ecuaia caracteristic p() = 0 asociat ecuaiei difereniale L(y) = 0 i
e 1 , 2 rdcinile sale. Deosebim urmtoarele trei cazuri:
I. 1 , 2 R, 1 6= 2 . Atunci putem lua y1 = e 1 t, y2 = e2 t i deci:

335

D(y1 ) = 1 y1 , D(y2 ) = 2 y2 ,
ceea ce conduce la matricea:

1
0

Af = AD =

0
2

n cazul particular (i), 1 = 1, 2 = 1 i deci:

1
0

Af =

0
1


.

II. 1 , 2 C \ R, 1 = 2 = + i . Atunci putem alege y1 = et cos t, y2 = et sin t i

deci:

D(y1 ) = y1 y2 ,

D(y2 ) = y1 + y2 ,

ceea ce ne conduce la matricea:

Af = AD =

1
2


=

Re1
Im1

Im1
Re!


.

3
i deci:
2


1
1
3

Af =
.
3 1
2

n cazul particular (ii), = , =

III. 1 = 2 = R. Atunci putem lua y1 = et , y2 = tet i deci:

D(y1 ) = y1 ,

D(y2 ) = y1 + y2 ,

ceea ce ne conduce la matricea:


Af = AD =


.

n cazul particular (iii), = 1 i deci:

Af =

atunci :

1
2

1
0

1
1


.

249. Dac a i b sunt numere reale pozitive astfel nct a + b = an + bn , n N, n 2,

an+1 + bn+1 2.
Soluia autorului. Scriem inegalitatea cerut sub forma omogen:

an + bn
a+b

n+1

an+1 + bn+1
2

Vasile Crtoaje

n1
.

Fr a pierde din generalitate, vom considera a b = 1. Inegalitatea devine astfel:

an + 1
a+1

sau

(n + 1)

n+1

an+1 + 1
2

n1

an+1 + 1
an + 1
(n + 1) ln
.
a+1
2

Problema revine la a arta c funcia:

f (x) = (n + 1) ln

xn + 1
xn+1 + 1
(n 1)
n+1
2

satisface condiia f (x) 0 pentru x 1. Deoarece f (1) = 0, este sucient s artm c f 0 (x) 0
pentru x 1. Avem:

1
nxn1
1
(n 1)xn
f 0 (x) = n

=
n+1
x + 1 x + 1 xn+1 + 1
336




xn1
1
xn
xn1

(n1)

=
xn + 1
x+1
xn+1 + 1
xn + 1


xn2 + . . . + x + 1
(n 1)xn1

=
x+1
xn+1 + 1
 n



x1
= n
(x 1) xn1 1 + x xn1 1 xn2 1 + . . .
(x + 1) (x + 1) (xn+1 + 1)


. . . + xn2 x2 1 (x 1) 0.
Inegalitatea din enun devine egalitate dac i numai dac a = b = 1.
=

x1
xn + 1

Soluie dat de Marian Tetiva, profesor la Colegiul Naional Gheorghe Roca-Codreanu


din Brlad. De fapt n poate  orice numr mai mare ca 1 i enunul rmne valabil. Pentru acest
lucru, s demonstrm nti c media generalizat a dou numere pozitive este logaritmic concav
(0, ); mai precis este valabil urmtoarea:
Lem. Fie a, b dou numere pozitive i funcia f dat prin :


f (x) =
Atunci

a2 + b2
2

1

este logaritmic concav, adic

,
g

x (0, ).

denit prin :

1
ax + bx
,
g(x) = ln f (x) = ln
x
2
este concav pe

x > 0.

(0, ).

Demonstraie. Calcule simple arat c derivata a doua a funciei g este:

g 00 (x) =

x2 h00 (x) 2xh0 (x) + 2x(x)


,
x3

x > 0,

unde h este funcia denit prin

h(x) = ln

ax + bx
,
2

x > 0.

La rndul ei, funcia de la numrtorul lui g 00 (care se poate deni i n 0) are derivata
x2 h(3) (x) 0, pentru orice x 0. Rezult c aceast funcie descrete pe [0.), deci este cel mult
egal cu valoarea sa n orine, care este 2h(0) = 0. Prin urmare g 00 (x) < 0, pentru orice x (0, ),
ceea ce trebuia demnstrat.
Nu am justicat faptul c h are derivata a treia negativ pe (0, ), dar vedem ndat cum
rezult tot din calcule. Anume avem:

h0 (x) =
apoi

h00 (x) =
i, n ne:

h(3) (x) =

ax ln a + bx ln b
,
ax + bx
(ln a ln b)2 ax bx
(ax + bx )2

(ln a ln b)3 (x ax ) ax bx
(ax + bx )3

Cum petru x > 0 diferenele ln a ln b i b2 a2 au semen contrae, se obine concluzia


anunat despre semnul lui h pe intervalul (0, ). (De asemenea, acelai raionament arat c
funcia f este logaritmic convex pe (0, ).)
Acum putem trece la rezolvarea problemei. Inegalitatea lui Jensen pentru funcia concav
f se scrie sub forma:

apx+qy + bpx+qy
2

1
px+qy

ax + bx
2

f racpx 

ay + by
2

q

n1
1
i q =
pentru
n
n
un anume n > 1 (nu neaprat numr natural, este clar). Obinem px + qy = n i inegalitatea
devine (i dup ridicarea la puterea n(n + 1)):
 n+1
n1 

 n

a
+ bn+1
a + b n+1
a + bn n+1

,
2
2
2
pentru orice p, q, x, y > 0 cu p + q = 1. S alegem aici x = n + 1, y = 1, p =

337

evident, cu ipoteza a + b = an + bn , aceast inegalitate conduce la an+1 + bn+1 2. Egalitatea


are loc (deoarece fucia g din lem este, de fapt, strict concav pentru a 6= b) doar dac a = b = 1.
250. Fie a, b, c numere raionale astfel nct a 6= 0 i 4ac b2 este ptratul unui numr
raional diferit de zero. S se construiasc un exemplu de funcie

i) f este aditiv, adic

f :RR

cu proprietile :

f (x + y) = f (x) + f (y),
pentru orice

x, y R;

ii) f veric relaia

af (f (x)) + bf (x) + cx = 0,
pentru orice

x R.

Gabriel Dospinescu i Marian Tetiva


Soluia autorilor. S vedem mai nti cum rezolvm problema ntr-un caz particular:

anume, pentru nceput, construim o funcie aditiv h : R R, care s verice n pus i relaia:

h (h(x)) = x,

x R.

Pentru aceasta s considerm o baz Hamel, adic o baz a lui R ca spaiu vectorial peste
Q i o partiie a sa H = a B n dou mulimi de acela si cardinal; cu alte cuvinte, exist o bijecei
: A B . Mai notm X = {x | x X} pentru orice muliem X de numere reale i denim
funcia g : H (H) H (H) prin:

g(x) =

(x),
1 (x),
(x),

1
(x),

xA
xB
x A
x B

(este uor de vzut c, dac A i B ealizeaz o partiie a lui H i aceasta este o baz a lui R peste
Q, atunci A, B , A, B partiioneaz pe H (H)). Pentru x A avem atunci g(x) = (x) B
i:

g (g(x)) = g ((x)) 1 ((x)) = x,

pentru x B avem g(x) = 1 A i



g (g(x)) = g 1 (x) = 1 (x) = x,
dac x A, g(x) = (x) B , deci:

g (g(x)) = g ((x)) = 1 ( ((x))) = x


i, n ne, pentru x B , g(x) = 1 (x) A i



g (g(x)) = g 1 (x) = 1 (x) = x,
n concluzie avem:

g (g(x) = x) ,

x H (H),

g(x) = g(x),

x H (H).

relaie care implic imediat i:


S zicem c elementele bazei Hamel considerate sunt notate ei unde i parcurge o anumit
mulime I de idici (de puterea continuumului); mai precis, e ej , j I , elementele din A i ek ,
k K elementele din B (J K = I , J K = ).
Funcia h pe care o cutm noi reprezint aa numita prelungire prin liniaritate a funciei
g , adic este denit prin:

h(x) =

ai g(ei ) =

iI

aj g(ej ) +

jJ

ak g(ek ),

kK

pentru orice:

x=

X
iI

ai ei =

X
jJ

aj g(ej ) +

ak g(ek ) R

kK

(orice x R are o unic asemenea scriere, cu ai , i I numere raionale dat ind c H = A B


este baz a lui R peste Q). Mai nti remarcm c h coincide cu g nu numai pe mulimea H (ceea
ce este resc), dar i pe H : ntr-adevr, avem pentru ei H :

h(ei ) = h ((1)ei ) = g(ei ) = g(ei )

338

(datorit faptului c g , aa cum am vzut puin mai sus, este impar). Acum se veric imediat c h
este o funcie aditiv sau, echivalent, Q-liniar (acesta este rostul prelungirii prin liniaritate) i nici
nu este greu de vzut c h (h(x)) = x, pentru orice x R. ntr-adevr, pentru x ca mai sus, avem,
conform deniiei lui h i innd seama de faptul c ej A g(ej ) B i ek B g(ek ) A,
c:
X
X

h (h(x)) =

aj g (g(ej )) +

jJ

(ak )g (g(ek )) =

aj g (g(ej )) +

jJ

kK

(ak )g (g(ek )) =

aj (ej ) +

jJ

kK

ak (ek ) = x

kK

(am folosit iar g(t) = g(t) i g (g(t)) = t, pentru orice t H (H).


Acum avem o funcie h : R R aditiv i astfel nct h (h(x)) = x, pentru orice x R.
Aditivitatea implic imediat Q-liniaritatea funciei, adic mai avem h(rx) = rh(x), oricare ar 
x R i r Q.
Pentru a rezolva problema noastr, s observm nti c, n condiiile enunului, trinomul
de gradul al doilea at2 + bt + c are dou rdcini complexe conjuate p + qi i p qi, cu p, q numere
raionale, prin urmare avem:

2p =

b
a

p2 + q 2 =

c
a

(conform relaiilor lui Vite ). De aceea, relaia pe care trebuie s ondeplineasc funcia f se poate
scrie sub forma:

2
2

f (f (x)) 2pf (x) + p + q

x = 0, x R.

Denim pe f prin:

f (x) = qh(x) + px,


pentru orice x R. Datorit Q-liniaritii funciei h i proprietii sale h (h(x)) = x, x R,
avem:

f (f (x)) = qh (qh(x) + px) + p (qh(x) + px) = q 2 h (h(x)) + 2pqh(x) + p2 x =



= 2pqh(x) + p2 q 2 x, x R,

ceea ce implic imediat:


f (f (x)) 2pf (x) + p2 + q 2 x = 0, x R
i ncheie rezolvarea problemei.
251. S se arate c, ntr-un simplex, au loc inegalitile urmtoare :

n
X
n2
ri
ri
(n 1)

;
r

r
r
+
r
2
i=1 i
i=1 i
n
n
n
X
X
n + 1 X hi
hi
n2
hi

(n 1)

;
b)
h r
n 1 i=1 hi + r
(n 2)hi + r
n1
i=1
i=1 i
n
n
n
X
X
n+1 X
ri
ri
ri

(n 1)

c)
2r + r
(n

2)r

r
n

1
(n

2)r
+
r
(n

2)
i
i
i
i=1
i=1
i=1

a)

n
X

n2
.
(n 2)(n 1)

Cu hi i ri se noteaz nlimile i razele sferelor exnscrise simplexului, iar cu


sferei nscris simplexului (i = 1, n, n 3).

raza

Mihai Miculia i Marius Olteanu


Soluia autorilor. Se tie c ntr-un simplex au loc relaiile (a se vedea D. S. Mitrinovi

. a., Recent Advances in Geometric Inequalities):

n
X

nV
nV
nV

Si , aa c :

hi = Si , r = S , ri = S 2Si , unde S =
i=1
(A)
n
n
X
X

1
1
1
1

= =
,

n2
r
r
h
i
i=1
i=1 i
unde S este aria total, Si este aria feei i, V este volumul.

339

a) innd seama de inegalitatea:


n1
X
i=1

echivalent cu:

!
xi

n1
X
i=1

1
xi

!
(n 1)2

(1)

n1
X 1
n1
1

, xi > 0, i = 1, n,
n1
n 1 i=1 xi
X
xi

(10 )

i=1

avem:

X ri
(n 1)rj
1
n1
n1
n1

X
=
X r =
r = r +r ,
ri r
n 1 i6=j ri r
j
1+
(n 1)
rj
r
r
i
i
i6=j
i6=j

de aici:

n
X
i=1

n
X
ri
ri
(n 1)
.
ri r
r
+r
i
i=1

Mai departe, utiliznd inegalitatea mediilor avem:

(n 1)

n
X
i=1

ri
n2
n2
n2
n2
(n 1) n
= (n 1)
= (n 1)
=
.
n
X
X
ri + r
n+n2
2
1
ri + r
n+r
ri
r
i=1 i
i=1

b) Pentru nceput, vom demonstra urmtoarea inegalitate (ce aparine lui Andrei Chite ).
,,Dac x1 , x2 , . . . , xn > 0, xi + x2 + . . . + x= 1, n 2, atunci :

(n 1)

n
X
i=1

n
X
1
1
(n + 1)
.
1 xi
1 + xi
i=1

(2)

Avem echivalena:

n 
X
i=1

2n

n
X
i=1

1
1

1 xi
1 + xi

n 
X
i=1

1
1
+
1 + xi
1 xi

n
n
n
X
X
X
xi
1
xi
1
2
n

.
2
2
2
1 xi
1

x
1

x
1

x2i
i
i
i=1
i=1
i=1

Deoarece xi > 0 pentru orice i = 1, n i

n
X

()

xi = 1, rezult c xi (0, 1) implic x2i (0, 1)

i=1

i deci 1 x2i > 0, pentru orice i = 1, n. Presupunem c x1 x2 . . . xn , atunci

1
1
...
. Din inegalitatea lui Cebev avem, n aceste condiii c:
1 x22
1 x2n
!
! n
n
n
n
X
X
X
1
1 X
xi
1
1

x
=

,
i
2
2
1

x
n
1

x
n
1

x2i
i
i
i=1
i=1
i=1
i=1

de aici:

n
X
i=1

1 x21

X
xi
1 X
1
1
n
=
,
1 x2i
n i=1 1 + x2i
1

x2i
i=1

adic ().
Egalitatea se obine numai dac x1 = x2 = . . . = xn sau
ceea ce este echivalent cu x1 = x2 = . . . = xn =

340

1
.
n

1
1
1
=
= ... =
,
1 x21
1 x22
1 x2n

Revenind la roblema enunat, pentru prima inegalitate a punctului b) se consider n

r
, pentru i = 1, n etc.
hi
Fie f : (0, ) R , denit prin:

inegalitatea (2) c xi =

n+1
1
1

(n 1)
, n 3, n N.
n1 1+x
(n 2) + x

f (x) =
Atunci

f 00 (x) = 2

(n + 1)(n 2 + x)3 2(n 1)2 (1 + x)3


;
(n 1)(1 + x)3 (n 2 + x)3

deoarece n 3 i x (0, 1), n N, se aratu a uor c (n + 1)(n 2 + x)3 2(n 1)2 (1 + x)3 este
pozitiv; rezult cu a f 00 (x) > 0 i deci f (x) este convex. Aplicnd, atunci, inegalitatea Jensen
funciei convexe f , avem, pentru orice xi (0, 1), i = 1, n:


f (x1 ) + f (x2 ) + . . . + f (xn ) n f

x1 + x2 + . . . + xn
n


.

(3)

r
i inem seama de relaiile (A) precum i de faptul
hi
r
r
r
 
+
+ ... +
1
h1
h2
hn
f
= 0,
=f
n
n

Dac n inegalitatea (3) alegem xi =


c

rezult n nal inegalitatea cerut.


n ne, aplicnd din nou inegalitatea mediilor avem:

(n 1)

n
X
i=1

n
X
n2
hi
(n 1)
=
n
X
(n 2)hi + r
(n 2)hi + r
i=1
hi
i=1

= (n 1)

n2
n2
n2
= (n 1)
=
.
n(n 2)2 + 1
(n 1)2
2

c) n inegalitatea (2) se alege xi =

r
, i = 1, n (se ine seama de egalitile (A); se
(n 2)ri

obine astfel prima inegalitate a punctului c). Pentru cea de a doua inegalitate se aplic inegalitatea
(3) funciei convexe g : (0, 1) R, g(x) =

1
f (x), unde f este funcia considerat la punctul
n2

r
, i = 1, n i se observ c:
(n 2)ri

r
r
r
+
+ ... +
 
 
1
1
1
(n 2)r2
(n 2)rn
(n 2)r1
g
=
f
= 0.
=g
n
n
n2
n

b); se alege xi =

Rezult:




r
r
0
+ ... + g
(n 2)r1
(n 2)rn
n
n
X
n+1 X
ri
ri

(n 1)
0.
2r + r
n 1 i=1 (n 2)ri + r
(n

2)
i
i=1
g

n sfrit, a treia inegalitate rezult tot din inegalitatea mediilor:

(n 1)

n
X
i=1

= (n 1)

ri
n2
(n 1) n
=
2
X
(n 2) ri + r
(n 2)2 ri + r
ri
i=1

n2
(n 1)n2
n2
=
=
.
n(n 2)2 + (n r)
(n 2)(n 1)2
(n 2)(n 1)
341

n toate punctele problemei egalitile au loc atunci i numai atunci cnd simplexul este
este ,,echifacial (adic S1 = S2 = . . . = Sn ).
Pentru n = 3 i n = 4 se obin frumoase inegaliti n triunghi i tetraedru.
Problema constituie o generalizare a problemelor PP.5165 i PP. 5166 din ,,Octogon Mathematical Magazine, pag. 357, octombrie 2004.
Soluie dat de Nicuor Minculete de la Universitatea Cretinu a Dimitrie Cantemir din
Braov. Fie (n 1)-simplexul A1 A2 . . . An , n 3, n spaiul euclidian (n 1)dimensional, iar
cu hi i ri se noteaz nlimile i razele hipersferelor exnscrise simplexului, cu r raza hipersferei
nscris simplexului, cu V notm volumul simplexului, cu Si notm volumul (n 2)simplexul
A1 A2 . . . Ai1 Ai+1 . . . An i n ne:

S=

n
X

Si .

(1)

i=1

ntr-un (n 1)simplex se cunosc urm atoarele egaliti:

hi =

(n 1)V
,
Si

(2)

(n 1)V
,
S
(n 1)V
ri =
.
S 2Si
r=

(3)
(4)

Ca urmare, putem stabil uor urmtoarele relaii:


n
X

r
= 1,
hi

(5)

r
= 1.
(n 2)ri

(6)

i=1
n
X

i=1

Pentru nceput vom demonstra urmtoarele patru inegaliti:


i)

n
n
X
X
1
1
(n 1)
,
a
A

ai
i
i=1
i=1

(7)

unde ai = R
+ , pentru orice i = 1, n, A =

n
X

ai ;

i=1

ii)

n
X
i=1

n
1
n+1 X 1

,
1 ai
n 1 i=1 1 + ai

unde ai = R
+ , pentru orice i = 1, n,
iii)

n
X

(8)

ai = 1;

i=1
n
X

n
1
n+1 X 1
(n 1)
,
n 1 i=1 1 + ai
n

2
+ ai
i=1

unde ai = R
+ , pentru orice i = 1, n,

n
X

(9)

ai = 1;

i=1

iv) (n 1)

n
X
i=1

1
n2

,
n 2 + ai
n1

unde ai = R
+ , pentru orice i = 1, n,

n
X

(10)

ai = 1.

i=1

i) Aplicm inegalitatea lui Cauchy-Buniakovski-Schwarz, astfel:


(a1 + a2 + . . . + ai1 + ai+1 + . . . + an )

342

1
1
1
1
1
+
+ ... +
+
+ ... +
a1
a2
ai1
ai+1
an

(n 1)2 ,

deci:

1
1
1
1
1
(n 1)2
+
+ ... +
+
+ ... +

.
a1
a2
ai1
ai+1
an
A ai

(11)

Scriind i inegalitile analoage inegalitii (11), prin nsumare, obinem:

(n 1)

n
n
X
X
1
1
(n 1)2
,
a
A

ai
i
i=1
i=1

iar mprind cu n 1, deducem inegalitatea (7).


ii) Rescriem inegalitatea (8) astfel:
n
X
i=1

n
2 X 1
1

,
1 ai
n 1 i=1 1 + ai

adic:

(n 1)

n
X
i=1

deci:

n
X

i=1

X 1
ai

,
1 a2i
1 + ai
i=1

X 1
X ai
X
1
ai

+
=
.
2
1 a2i
1
+
a
1

a
1

a2i
i
i
i=1
i=1
i=1

Prin urmare inegalitatea (8) este echivalent cu inegalitatea:

n
X
i=1

X 1
ai

.
1 a2i
1 ai
i=1

(12)

Fr a micora generalitatea, presupunem c ai a2 . . . an , de unde rezult c


1
1
1

...
i, prin aplicarea inegalitii lui Cebev, deducem inegalitatea:
1 a21
1 a22
1 a2n

n
X
i=1

deoarece

n
X

ai

n
n
n
X
X
X
1
1
1

=
,
a
i
2
1 a2i
1

a
1

a2i
i
i=1
i=1
i=1

ai = 1,n consecin demonstrarea inegalitii (8) se ncheie.

i=1

iii) Prelucrm inegalitatea (9) i se obine:

(n + 1)

n
X
i=1

n
n
n
X
X
X
1
1
n2
1
(n 1)
=
+n
,
1 + ai
n

2
+
a
n

2
+
a
n

2 + ai
i
i
i=1
i=1
i=1

ceea ce este echivalent cu:

n
X
i=1

deci:

n 
X
i=1

adic:

n
n
n
X
X
X
1
1
1
n2
+

+n
,
1 + ai
1
+
a
n

2
+
a
n

2 + ai
i
i
i=1
i=1
i=1

1
1

1 + ai
n 2 + ai

n
X
i=1


n

n 
X
i=1

n2
1

n 2 + ai
1 + a1


,

n
X
n3
ai
n
,
(1 + ai )(n 2 + ai )
(1
+
a
)(n
2 + ai )
i
i=1

iar, prin mprire cu n 3, se obine inegalitatea:


n
X
i=1

n
X
1
ai
n
.
(1 + ai )(n 2 + ai )
(1
+
a
)(n
2 + ai )
i
i=1

(13)

Fr a micora generalitatea presupunem c a1 a2 . . . an , de unde rezult c:

1
1
1

...
(1 + a1 )(n 2 + a1 )
(1 + a2 )(n 2 + a2 )
(1 + an )(n 2 + an )
343

si, prin aplicarea inegalitii lui Cebev, deducem inegalitatea:

n
X
i=1

dar

n
X
i=1

n
n
X
X
ai
1

ai
,
(1 + a1 )(n 2 + ai )
(1
+
a
)(n
2 + ai )
i
i=1
i=1

ai = 1, prin urmare deducem inegalitatea (13).


iv) Utiliznd inegalitatea Cauchy-Buniakovski-Schwarz avem:
n
X

(n 2 + ai )

i=1

n
X
i=1

deci

[n(n 2) + 1]

n
X
i=1

n consecin:

(n 1)

n
X
i=1

1
n2 ,
n 2 + ai

1
n2 ,
n 2 + ai

1
n2
.

n 2 + ai
n1

Cumulnd inegalitile (8), (9) i (10) obinem irul de inegaliti:


n
X
i=1

n
n
X
1
n+1 X 1
1
n2

(n 1)

,
1 ai
n 1 i=1 1 + ai
n 2 + ai
n1
i=1

unde ai = R
+ , pentru orice i = 1, n,

n
X

(14)

ai = 1.

i=1

Revenim la inegalitile din enun.


a) Prin utilizarea egalitilor (3) i (4), obinem notaiile:

S
ri
=
,
ri r
2Si

ri
S
=
,
ri + r
2(S Si )

ceea ce nseamn c inegalitatea:


n
X
i=1

devine:

n
X
ri
ri
(n 1)
,
ri r
r
+r
i=1 i
n
X
1
1
(n 1)
,
S
S

Si
i
i=1

ceea ce este adevrat, lund ai = Si n inegalitatea (7).


Din inegalitatea Cauchy-Buniakovski-Schwarz avem:

 n
n 
X
r X
1
2
1+
r n ,
r
i
i=1
i=1 1 +
ri
dar:


n 
X
r
= n + n 2 = 2(n 1),
1+
ri
i=1

aadar:

(n 1)

n
X
i=1

ri
n2

,
ri + r
2

veea ce nseamn c inegalitile de la punctul a) au fost demonstrate.


b) Cum
inegaliti:

n
X
r
r
= 1, lum n irul de inegaliti (14), ai =
, ceea ce implic irul de
h
h
i
i
i=1

n
X
i=1

344

n
n
X
n+1 X
1
1
n2
r n1
r (n 1)
r n 1,
1
i=1 1 +
i=1 n 2 +
hi
hi
hi

adic:

n
X
i=1

n
n
X
hi
n + 1 X hi
hi
n2

(n 1)

.
hi r
n 1 i=1 hi + r
(n

2)h
+
r
n
1
i
i=1

c) Datorit faptului c

n
X
i=1

r
r
= 1, lum n irul de inegaliti (14) ai =
,
(n 2)ri
(n 2)ri

ceea ce implic irul de inegaliti:


n
X

n
n
X
1
n+1 X
1
n2
,

(n 1)

r
r
r
n

1
n
1
1
i=1 1 +
i=1 n 2 +
(n 2)ri
(n 2)ri
(n 2)ri

i=1

aadar:
n
X
i=1

n
n
X
r
r
r
n2
n+1 X
(n 1)

2r + r
(n 2)ri
n 1 i=1 (n 2)ri + r
(n

2)
(n

2)(n
1)
i
i=1

astfel, demonstraia este ncheiat.


Observaie. irurile de inegaliti de la punctele b) i c) se pot mbunti prin urmtorul

procedeu: n inegalitatea (14) punem n loc de ai pe

1 ai
1 ai
, deoarece
R+ , pentru orice
n1
n1

n
n
X
X
1 ai
= 1, din
ai = 1, deci aceasta devine:
n1
i=1
i=1

i = 1, n,

(n 1)

n
X
i=1

n
n
X
X
1
1
1
n2
(n + 1)
(n 1)2

, (15)
n 2 + ai
n ai
(n 2)(n 1) + 1 ai
n1
i=1
i=1

iar, dac aplicm din nou acest procedeu pentru inegalitatea (15), obinem un alt ir de inegaliti,
i anume:
n
X
i=1

n
X
1
1
n2 1

(n 2)(n 1) + 1 ai
n(n

1)
1 + ai
i=1

(n 1)3

n
X
i=1

Dac nlocuim ai =

(n 1)

n
X
i=1

n2
1

.
2
(n 2)(n 1) + n 2 + ai
n1

(16)

r
n inegalitatea (15), atunci deducem irul de inegaliti:
hi

n
n
X
X
hi
hi
n2
hi
(n + 1)
(n 1)2

, (17)
2 3n + 3) h r
(n 2)hi + r
nh

r
(n
n
1
i
i
i=1
i=1

iar pentru ai =

r
, gsim irul de inegaliti:
(n 2)ri
(n 1)

n
X
i=1

n
X
ri
ri
(n + 1)

(n 2)2 hi + r
n(n 2)hi r
i=1

(n 1)2

n
X
i=1

(n2

n2
ri

.
3n + 3) (n 2)ri r
n1

(18)

Prin continuarea procedeului de mai sus se pot gsi o mulime de inegaliti care pot
mbuntii irul de inegaliti din enunul problemei.
252. Dac x1 , x2 , . . . , xn > 0, unde n N, n 2, k {1, 2, . . . , n 1}, S =

n
X
i=1

xi1 xi2 . . . xik ,

s se arate c :

1i1 <i2 <...<ik n

q
q
k
xi1 xi2 . . . xik (S xi1 xi2 . . . xik ) Cn1
S.

1i1 <i2 <...<ik n

345

x1

Gh. Szllsy
Soluia autorului. Din inegalitate mediilor rezult:

X
1i1 <i2 <...<ik

v
u

n
u k
S xi1 xi2 . . . xik
t Cn xi1 xi2 . . . xi
nk
k

S
n

deci

1
2


k
+
Cn

n
Ck
(n k) n
nk
n

k
,
= Cn

q

xi1 xi2 . . . xik S xi1 xi2 . . . xik

X
1i1 <i2 <...<ik n

k
Cn

(n k)S
=
k
nCn

q
nk k
k
Cn S = Cn1
S,
n

q.e.d.
Soluie dat de Marian Tetiva, profesor la Colegiul Naional ,,Gheorghe Roca-Codreanu
din Brlad.
Inegalitatea din enun nu este nimic altceva dect inegalitatea Cauchy-Scwarz n forma:

v
um
m p
m
X
uX X
ai bi t
ai
bi
i=1

i=1

i=1

k , numerele a egale respectiv cu produsele x


pentru m = Cn
i
i1 xi2 . . . xik (dup toate alegerile
indicilor 1 i1 i2 < . . . < ik n). Avem atunci suma numerelor ai egal cu , iar:
m
X
i=1

bi =


  k
k1
k
Cn1
S = Cn1
S
S xi1 xi2 . . . xik = Cn

1i1 <i2 <...<ik n

i nu mai este nimic de demonstrat.

ISTORIA MATEMATICII
La 100 de ani de la naterea academicianului Nicolae Teodorescu n
contextul european al tiin ei
de

Eufrosina Otlcan

Academicianul Nicolae Teodorescu (1908-2008) aparine generaiei de aur a matematicii


romneti. Personalitate marcant a cercetrii de specialitate, sa implicat cu druire educaiei
tiinice a tinerei generaii, meninerii cercetrii matematice din ar n atenia lumii tiinice
internaionale i nfptuirii unor programe naionale de dezvoltare tiinic, tehnic i cultural.
1. Studii, funcii, discipline universitare predate

Nscut la Bucureti la 5/18 iulie 1908, Nicolae Teodorescu i-a fcut aici toate studiile,
lundu-i licena n matematici n anul 1929. Doctoratul n matematici i-l trece la Universitatea Sorbona din Paris la 25 aprilie 1931. Subiectul tezei sale de doctorat pornea de la noiunea de derivat areolar introdus n 1912 de ilustrul matematician romn Dimitrie Pompeiu,
continundu-i studiul teoretic, gsindu-i aplicaii interesante n Fizica matematic i indicnd legturi cu noiunea de derivat exterioar a matematicianului francez Elie Cartan. Ca o onoare i
recunoatere fcut cercetrii romneti, n comisia de doctorat din universitatea parizian este
invitat i Dimitrie Pompeiu, fapt fr precedent i care, din cte tiu, nu s-a mai repetat nici pn
astzi.
ntors la Bucureti dup susinerea doctoratului, Nicolae Teodorescu deine, pe rnd sau
uneori simultan, funcii universitare la Facultatea de tiine a Universitii, la Academia de arhitectur, la Institutul de statistic, actuariat i calcul, la Institutul Politehnic, la Institutul de

346

Construcii. Din 1953 va  ef de catedr la Facultatea de matematic a Universitii din Bucureti, iar ntre anii 1960-1972 decan al acestei faculti. i disciplinele matematice pe care le-a
predat sunt multiple: mecanic, geometrie descriptiv i stereotomie, analiz matematic, ecuaii
difereniale, calcul operaional, matematici speciale aplicate, ecuaiile zicii matematice.
Din 1948 a fost vicepreedinte al Societii de tiine Matematice din Romnia, iar din 1975
a fost preedintele ei, coordonnd toate publicaiile periodice i ne-periodice ale acestei instituii.
Din 1949, de la ninarea Institutului de matematic al Academiei Romne, Nicolae Teodorescu
a fost ef de secie pentru Ecuaii difereniale i cu derivate pariale.
Despre lucrrile de cercetare matematic

Domeniile pe care Nicolae Teodorescu le atac n lucrrile sale tiinice sunt: analiza
matematic, ecuaiile cu derivate pariale liniare de ordinul nti i de ordin superior, teoria geometric a ecuaiilor difereniale sau a celor cu derivate pariale, calculul vectorial i tensorial,
calculul numeric. Funciile monogene (a) i funciile olomorfe (), introduse de Nicolae Teodorescu
n lucrri privind noiunea de derivat areolar a lui Pompeiu i lrgirea acestei noiuni sunt nscrise n ,,Histoire generale des Sciences, publicat sub direcia lui Ren Taton (Partea a III-a, ,,La
science contemporaaine vol. I, Le XXe sicle, Presse Universitaires de France, 1964, p. 43). Nicolae Teodorescu dezvolt, sintetizeaz i gsete semnicaii zice pentru idei care aparineau unor
nume mari ale matematicii internaionale, precum J. Hadamard, H. Weyl, E. Cartan, O. Veblen.
Extinderea noiunii lui Pompeiu de derivat areolar l-a condus la conexiuni cu cercetri ale lui De
la Valle Poussin i Lebesgue, dar i cu discipline mecanice, precum elasticitatea i hidrodinamica.
Rezultatele obinute de Nicolae Teodorescu au fost folosite de matematicieni romni, numind n
primul rnd pe Gr. C. Moisil, dar i de cercettori din afara rii, printre care A. Tomolo, T.
Vignaux, J. Ridder, F. Polaczek, V. S. Feodotov, I. N. Vequa.
Lucrrile lui Nicolae Teodorescu sunt publicate n reviste de specialitate de prim clas, precum ,,C. R. Acad. Sc. Paris, ,,Rendiconti dei Lincei, ,,Annali di Matematica din Bologna, ,,Annali
de matematica pura ed applicata, ,,Journal de mathmatiques pures et appliques, ,,Commentarii
mathematici helvetici, ,,Mathematica de la Cluj i n revistele Academiei Romne. Teza sa de
doctorat, ,,La drive arolaire et ses applications la Physique mathmatique a fost publicat de
Gauthier-Villars.
Nicolae Teodorescu a fost membru corespondent al Academiei Romne din 1955 i membru
titular din anul 1963.
3. Educator al tinerei generaii

A fost mai nti activitatea de la catedra universitar, de unde multe decenii a transmis
studenilor cunotine de nalt matematic. A pus la dispoziia studenilor leciile tiprite, precum
un ,,Curs de hidrodinamic plan i aplicaii aerodinamice (1936), ,,Calcul numeric i grac (1951),
,,Calcul vectorial (1951), ,,Metoda vectorial n zica matematic n dou volume, editate de
Editura Tehnic n 1953 i 1954, distinse cu Premiul de Stat. Mai sunt publicate ,,Curs de ecuaiile
zicii matematice, 2 volume, litograat de Universitatea Bucureti (1953 - 1954), ,,Ecuaiile zicii
matematice, partea a III-a (1959), partea a IV-a (1961), publicate de Editura de Stat Didactic i
Pedagogic.
Academicianul Nicolae Teodorescu a antrenat n cercetare, dar i n redactarea cursurilor
universitare matematicieni mai tineri. n felul acesta n anii 1950-1951 au fost litograate manualele
universitare: ,,Curs de ecuaii difereniale i cu derivate pari ale, redactat de asistent I. P. Elianu
i ,,Ecuaiile zicii matematice redactat de asistent M. Mayer n 1963 i 1965, Editura de stat
Didactic i Pedagogic public n dou volume ,,Ecuaiile zicii matematice, autori N. Teodorescu
i V. Olaru.
Preocupat de modernizarea nvmntului matematic romnesc, Nicolae Teodorescu studiaz n 1963, la Londra, aspecte ale organizrii nvmntului din Anglia.
Dezvoltarea sistemului de gndire matematic, riguros, a fost o constant n activitatea lui
Nicolae Teodorescu, materializat i prin activitatea sa la Gazeta Matematic, unde i ncepuse
activitatea creatoare nc din anii copilriei. n 1980 a ninat ,,Gazeta matematic metodic i
metodologic pentru profesori i studeni, iar vechii Gazete i adaug rubrici noi (Informatic,
Concurs, Recenzii, probleme comentate). Pn n 1980 acad. Nicolae Teodorescu a coordonat
Olimpiadele Naionale de Matematic. Profesorii de matematic din nvmntul preuniversitar au

347

beneciat de organizarea consftuirilor organizate de S.S.M.R., iniiate de acad. Nicolae Teodorescu


i la care a fost prezent de multe ori, innd lecii pentru profesori i elevi.
Pentru a trezi interesul tinerilor pentru matematic, Nicolae Teodorescu a publicat n
Gazeta Matematic att articole de prezentare a unor noiuni i teorii noi, ct i pagini dedicate unor valoroase personaliti ale matematicii romneti i strine. (Ex: ,,Metoda geometric
n zica matematic n Gazeta matematic i zic seria A, vol. V, 1953, ,,Dimitrie Pompeiu n
octombrie 1954, ,,Cel de al patrulea Congres al matematicienilor romni n iulie 1956, ,,Congresul
internaional al matematicienilor de la Edinbourg, n noiembrie 1958, ,,Personalitatea lui Ianos
Bolyai, n 1960 etc.).
4. Reprezentant al matematicii romneti n lumea tiinic european

Dup susinerea doctoratului la Paris cu teza sa care a trezit interes viu i de durat n
lumea matematicienilor, Nicolae Teodorescu continu s se arme i s creeze puni de legtur
cu oameni de tiin din Europa n calitate de secretar a fost principalul organizator al celui de al
4-lea Congres al matematicienilor romni, desfurat ntre 27 mai i 4 iunie 1956 la Bucureti. La
acest congres s-au prezentat 223 comunicri i conferine de ctre 152 matematicieni romni i 74
strini. venii din 18 ri, cei mai muli din Germania i Frana. Au fost multe nume rsuntoare
n matematica mondial, ntre care J. Hadamard, care, scria Nicolae Teodorescu ([4]): ,,la vrsta
de 92 de ani ne-a fcut deosebita cinste de a veni n ar i, n acelai articol, despre congres:
,,Pregtit cu atenie i cu grij timp de 10 luni, acest Congres a nsemnat o dat memorabil
pentru tiina romneasc, bucurndu-se de o participare internaional fr precedent n ara
noastr. Aprecierile vin i din partea marelui J. Hadamard, spunnd despre Congres c ,,indic,
dup prerea mea, o dat semnicativ n evoluia cultural a lumii ntregi n aceast epoc. Salut
aceast dat`.
N. Teodorescu a participat ca delegat al rii noastre, fcnd comunicri, la multe congrese
i colocvii in strintate ([ 1], [4]). ntre acestea: Amsterdam (1954), Praga (1955), Moscova i
Viena (1956), Paris (1957), Edinbourg (1958), Roma (1960), Balaton, Bologna i Florena (1961),
Stockholm (1962). ine conferine n Frana (1931, 1957, 1963), n Italia (1936, 1964), n Belgia
(1935, 1938, 1939, 1959), Germania de Est (1955). Dintr-o relatare fcut n Gazeta matematic
[4, pag. 209-211] deducem c Nicolae Teodorescu prezenta n conferinele sale din strintate nu
doar cercetrile i descoperirile proprii, ci i pe ale colegilor si romni: ,,Prin numeroase contribuii
ale matematicienilor romni (Gr. Moisil, M. Nicolescu, Gh. Clugreanu ) i strini [ ... ] teoria
derivatei areolare i-a ctigat un loc sigur n mtematica moderna ca o important realizare a
coalei matematice romneti.
Desigur c i multitudinea lucrrilor tiinice, publicate de N. Teodorescu n prestigioase
reviste strine de specialitate, au constituit o baz pentru prezena n lume a cercetri matematice
din Romnia.
5. Implicarea lui Nicolae Teodorescu n programul de informatizare a rii

n raportul Institutului Naional de Cercetare - Dezvoltare n Informatic, ICI, [3], se arat


c n iunie 1967 a fost elaborat i adoptat primul Program de dotare a economiei naionale cu
echipamente moderne de calcul i automatizarea prelucrrii datelor i s-a declanat activitatea
organizat la nivel naional n domeniul informaticii. ,,Un stimul important pentru elaborarea
acestui program de informatizare l-a constituit propunerea naintat conducerii statului de ctre
Mihai Drgnescu i Nicolae Teodorescu .
n articolul ,,Mihai Drgnescu  cronologia activitilor n domeniul informaticii [2], citim
c n 1966 ,,elaboreaz i nainteaz Consiliului Naional pentru Cercetare tiinic [ ... ] mpreun cu Acad. Nicolae Teodorescu, matematician, o propunere privind introducerea i utilizarea
calculatoarelor electronice n economia i societatea romneasc, propunere care a contribuit la
lansarea primului program de informatizare n Romnia.
Octogenar activ, academicianul Nicolae Teodorescu ddea rspuns discursurilor de recepie
n Academia Romn academicianului Romulus Cristescu, actualul preedinte al seciei de Matematici si regretatului academician Constantin Drmb.
Academicianul Nicolae-Victor Teodorescu, sau mai simplu, profesorul Nicolae Teodorescu,
cum ia semnat crile i cum l-am cunoscut noi, cei care i-am fost studeni, a prsit viaa academic i viaa realitilor noastre la 28 februarie 2000.
348

Bibliograe

[1] G. t. Andonie, Istoria Matematicii n Romnia, volumul 2, Editura tiinic, Bucureti,


1966.
[2] M. Drgnescu, Cronologia activitilor n domeniul informaticii, Internet.
[3] Raportul Institutului Naional de Cercetare - Dezvoltare n Informatic, ICI, Internet.
[4] Gazeta matematic i Fizic, seria A, 1956.
[5] Echipa gazetamatematica.net, 2008.
Comitetul Romn pentru Istorie i Filozoa tiinei i Tehnicii,
eufrosinaotl@imar.ro

Spaiile neolonome ale lui Vrnceanu


din punctul de vederea al geometriei distanei
de

M. Buliga

Aceast aniversare a descoperirii spaiilor neolonome de ctre Gheorghe Vrnceanu [12]


(1926), [13] (1928), este o bun ocazie pentru a descrie unele apariii surprinztoare, dar naturale,
ale acestor spaii n patru domenii matematice: operatori difereniali hipoeliptici, geometria subriemannian, teoria grupurilor Carnot i teoria geometric a grupurilor discrete. Cum de apar
spaiile neolonome n astfel de domenii variate? Cheia necesar pentru a ntelege este dat de
geometria distanei.
Vrnceanu are contribuii importante n mai multe domenii ale matematicii, nu doar teoria
spaiilor neolonome. De exemplu, n lucrarea [14] (1950), urmat de o serie de alte articole, s-a
ocupat de grupuri Lie de rang zero, numite i grupuri liforme, o clas particular de grupuri
Carnot. n [15], [16] (1962-1963) i alte lucrri a studiat scufundri ale unui grup discret n grupuri
lineare. Pot doar s presupun c intuiia sa puternic l-a ghidat spre domenii care dup un timp
s-au dovedit a  legate cu spaiile neolonome.
Gh. Vrnceanu a fost condus ctre descoperirea spaiilor neolonome de anume preocupri,
din zica teoretic i din geometria diferenial, care erau de mare actualitate n epoc. Spaiile
neolonome n mecanica clasic apar n cinematica sistemelor dinamice cu legturi liniare. O surs
iniial mai puin menionat, dar n prezent reconsiderat, se gsete n lucrrile privind termodinamica ale lui Gibbs i Carathodory. Un spaiu neolonom poate  nzestrat cu o distan numit
distan Carnot-Carathodory, n acelai fel n care o varietate diferenial poate  dotat cu o distan riemanniana. Astfel, n geometria spaiilor metrice spaiile neolonome sunt cunoscute drept
spaii Carnot-Caratheodory. Aceast din urm terminologie s-a impus n subiectele matematice pe
care le voi evoca mai jos.
Geometria sub-riemannian studiaz spaiile neolonome dotate cu distane Carnot-Carathodory. Pentru a ntelege uor natura acestor distane, s ne nchipuim c suntem n cabina unui
camion cu remorc. Dorim s parcm camionul i remorca sa paralel cu trotuarul. Este evident c
dei distana pn la trotuar este, s zicem, de 2 metri, avem nevoie s parcurgem o distan mult
mai mare pentru a parca, datorit diverselor manevre necesare. Distana aceasta este o distan
Carnot-Caratheodory. ntr-adevr, sistemul mecanic format din camion i remorca sa este descris
de un spaiu neolonom, n care se ine cont de diversele legturi (sau constrngeri) la care este
supus sistemul. Ideal, un bun ofer va parca urmnd o geodezic (drum de lungime minim) n
acest spaiu neolonom, iar distana necesar pentru a parca reprezint lungimea acestei geodezice.
Imaginaia matematicianului vede aici un spaiu neolonom ale crui puncte reprezint conguraii
(poziii) posibile ale camionului-remorc, i n care distanele se msoar de-a lungul geodezicelor.
Cum arat aceste spaii ascunse ochiului? n anii '20, dup contribuiile lui Cartan, Levi-Civita
i Weyl la teoria conexiunilor, Vrnceanu d o descriere a acestor spaii n termeni de geometrie
diferenial.
Mult mai trziu, n 1967, Hrmander [7] face o contribuie fundamental n domeniul
ecuaiilor cu derivate pariale, n care studiaz operatorii hipoeliptici (acetia sunt pentru un spaiu
neolonom ceea ce un operator eliptic, de exemplu operatorul laplacian, este pentru un spaiu

349

riemannian). Aceast contribuie a lui Hrmander este o dezvoltare a muncii pentru care primete
medalia Fields n 1962.
n 1981 M. Gromov [5] demonstreaz o teorem reciproc a alternativei lui Tits. S considerm un grup (discret) G generat de un numr nit de elemente. Dac acest grup poate  scufundat
ntr-un grup de transformri liniare ale unui spaiu vectorial nit dimensional, atunci creterea sa
este polinomial sau exponenial (aceasta este alternativa lui Tits ). Creterea unui grup discret
este o estimare a funciei care asociaz unui numr natural n (sucient de mare) numrul de
elemente ale grupului ce pot  obinute ca produse de cel mult n generatori. Dei functia cretere
depinde de alegerea generatorilor grupului, comportamentul su atunci cnd n tinde la innit este
independent de alegerea generatorilor. Alternativa lui Tits ne spune c pentru subgrupuri discrete
(i nit generate) ale grupurilor liniare numrul de elemente ale grupului ce pot  scrise ca produse
de cel mult n generatori se comport (pentru n mare) ca un polinom n n sau ca o exponenial n
n. n particular, dac grupul discret G este virtual nilpotent atunci creterea sa este polinomial.
Gromov demonstreaz c dac grupul G are o cretere polinomial atunci el este virtual nilpotent
(adic modulo un grup nit el poate  scufundat ntr-un grup de matrici superior triunghiulare).
Pentru a demonstra aceasta teorem remarcabil) Gromov face apel la spaiile neolonome!
S atam grupului G o distan: dou elemente diferite x, y din G sunt la distana m (numr
natural nenul) dac putem scrie y = ux cu u element al lui G care poate  scris numai ca un produs
de cel puin m generatori ai lui G. Grupul G devine astfel un spaiu metric, cu distana notat
cu d. Gromov arat c putem privi de departe acest spaiu metric, n felul urmtor: s notm cu
B(n) mulimea elementelor lui G care pot  exprimate ca produs de cel mult n generatori. Pentru

d
. Acest spaiu metric este
n
de diametru cel mult 2 (pentru c am mprit distana d la n). Pentru n din ce n ce mai mare
B(n) are din ce n ce mai multe elemente, iar elementele sale sunt din ce n ce mai apropiate.
orice numr natural nenul n avem spaiul metric B(n) cu distana

Obinem astfel un ir de spaii metrice care tinde (n sensul introdus de Gromov ) spre un spaiu
metric care nu mai este discret, ci continuu, mai precis este un spaiu neolonom de un tip special,
numit grup Carnot (din nou o referire la terminologia mprumutat n termodinamic). Aceasta se
ntmpla n ipoteza creterii polinomiale a lui G. Gromov arat c acest spaiu asimptotic este un
spaiu Carnot-Carathodory asociat unui grup nilpotent graduat, de unde deduce c G este virtual
nilpotent.
Grupurile Carnot, numite i grupuri omogene, vezi Folland, Stein [4], sunt obiecte de interes
n domenii ale analizei matematice i ale ecuaiilor cu derivate pariale, legate de operatorii difereniali hipoeliptici ai lui Hrmander. O clas interesant a lor este format de grupurile liforme,
introduse i studiate de Vrnceanu n [14].
Aceste grupuri apar din nou n studiul spaiilor neolonome o dat cu lucrarea lui Mitchell
[10] din 1985. Acesta demonstreaz c spaiul tangent (n sens metric) la un spaiu neolonom
(regulat) este un grup Carnot, folosind un raionament asemntor cu cel precedent. n loc s
mergem spre innitul mare, vom merge spre innitezimal. S privim vecintatea unui punct x
dintr-un spaiu neolonom M din ce n ce mai de aproape. Pentru ecare numr natural nenul n
vom considera spatiul metric B(n) al punctelor y aate la distant cel mult

1
de punctul x, cu
n

distana nd. Pentru ecare n spaiul metric B(n) are diametrul cel mult 2, pentru c am nmulit
distana Carnot-Caratheodory iniial d cu n. Pentru n din ce n ce mai mare, mulimea B(n) este
din ce n ce mai mic, iar distanele dintre punctele din ce n ce mai apropiate de x devin din ce
n ce mai mari, tinznd spre o distan nit. La limit obinem spaiul tangent n punctul x la
varietatea neolonom M . Mitchell demonstreaz c acesta este un grup Carnot (adic la rndul
su un spaiu neolonom).
Geometria metric a spaiilor neolonome este studiat n continuare de Belliche [1] i
Gromov [6] (1996). Acetia furnizeaz o descriere a spaiilor neolonome intrinsec din punctul de
vedere al geometriei distanei. ntr-un astfel de spaiu noiunile intrinseci de: innitezimal, derivare,
brat tangent, sunt altele dect cele uzuale pentru o varietate diferenial. ntrm aici ntr-un
domeniu erbinte al matematicii actuale, cel al analizei matematice pe spaii metrice generale.
Spaiile neolonome furnizeaz o clas foarte interesant de exemple pe care teoria general este
aplicat i noi idei sunt testate. Domeniul este n dezvoltare, dup cum arat contribuii recente
ale unor mari matematicieni: Cheeger [3] (1999), Margulis, Mostow [8] (1995) (i rspunsul [9] la
o critic a lui Deligne ).
O parte a interesului pentru spaiile neolonome privite din punctul de vedere al geometriei
distanei vine ca urmare a articolului lui Pansu [11] (1989), n care acesta d o nou demonstraie a

350

teoremei de super-rigiditate a lui Margulis (medalie Fields n 1978), privind scufundarea grupurilor
discrete n anume grupuri continue (grupuri Lie). O scufundare quasi-izometric unui grup discret
ntr-un alt spaiu metric (de exemplu un grup continuu cu o distan invariant la stnga) este o
scufundare fr a modica distana pe grupul discret prea mult: s notm cu d distana pe grupul
discret n raport cu un sistem de generatori, cu d0 distana pe spaiul metric int i cu f funcia
care face scufundarea. Funcia f este o quasi-izometrie dac exista constante A i B pozitive astfel
nct pentru orice x, y din grupul discret avem:

0

Ad (f (x), f (y)) d(x, y) B.
Exista ntotdeuna o astfel de scufundare? Cum numarul B poate  arbitrar de mare, este
vorba de o proprietate metrica la scara mare. Margulis demonstreaz pe o cale foarte lung faptul
c doar n cazuri foarte particulare o astfel de scufundare exist, de unde numele de super-rigiditate:
chiar dac avem voie s deformm arbitrar de mult (dar n limitele impuse de existena constantelor
A, B ) grupul discret ca s l scufundam n grupul continuu, asta este posibil doar dac cele dou
grupuri sunt apropiate n anume sens.
Pansu demonstreaz c dac privim de foarte departe scufundarea f , n stilul lui Gromov,
aceasta devine o aplicaie Lipschitz (aproape ca i cum B = 0) ntre un grup Carnot (spaiu neolonom) i un spaiu riemannian. Apoi demonstreaz c o teorem clasic de analiza (teorema
lui Rademacher: orice aplicaie Lipschitz este derivabil aproape peste tot) este adevarat pentru
situaia dat, ntr-un sens generalizat, folosind o noiune de derivare intrinsec spaiilor neolonome.
n concluzie, exist mcar un punct (din spaiul metric asimptotic la grupul discret) n care scufundarea este derivabil i derivata sa este o aplicaie liniar (morsm de grupuri). Existena
acestor aplicaii liniare este o problem algebrica uor de tranat, ceea ce ne conduce la rezultatul
de rigiditate al lui Margulis !
De aici, unde s mergem mai departe n studiul spaiilor neolonome? Domeniul este vast,
posibilitile de extindere sunt mari. mi permit n continuare s sugerez o direcie personal. Spaiile neolonome ale lui Vrnceanu sunt prezente, dup cum s-a vzut, n multe subiecte matematice
legate de proprietile innitezimale i asimptotice (la innit) ale spaiilor metrice. Aici geometria
i analiza matematic se ntlnesc. Spaiile neolonome ne obinuiesc cu noi noiuni de innitezimal
i ne ndeamn s reconsiderm rezultate matematice clasice dintr-o nou perspectiv. Vrnceanu
a introdus spaiile neolonome ca o construcie din domeniul geometriei, sub domeniul geometriei
difereniale, adic folosind concepte clasice de analiz matematic drept fundament. Ori, rezultate
recente ne arat c aceste spaii sunt, din punctul de vedere intrinsec al geometriei distanei, altfel
dect orice am vzut pn acum. n loc de spaii vectoriale gsim grupuri Carnot (un fel de spaii
vectoriale necomutative), iar n loc de varieti difereniabile gsim spaii neolonome. Poate c
este momentul s descoperim spaiile neolonome ca exemple de spaii pe care triete o alt analiz
matematic dect cea uzual (cum este sugerat n [2]). Astfel, dac facem o paralel cu apariia
spaiilor ne-euclidiene acum mai mult de un secol, poate vom putea arma cndva c primele exemple de spaii cu analiz ne-euclidian (adic la orice scar altfel dect un spaiu euclidian) aparin
geometrului romn Gheorghe Vrnceanu.
Bibliograe

[1] A. Bellaiche, The tangent space in sub-Riemannian geometry, in: Sub-Riemannian Geometry,
A. Bellaiche, J.J. Risler eds., Progress in Mathematics, Birkhser, (1996), 4-78.
[2] M. Buliga, Dilatation structures 1. Fundamentals, J. of Generalized Lie Theory and Appl., 1,
2 (2007), 65-95.
[3] J. Cheeger, Dierentiability of Lipschitz functions on metric measure spaces, Geom. Funct.
Anal., 9 (1999), no. 3, 428-517.
[4] G.B. Folland, E.M. Stein, Hardy spaces on homogeneous groups, Mathematical Notes, 28,
Princeton University Press, N.J.; University of Tokyo Press, Tokyo, 1982.
[5] M. Gromov, Groups of polynomial growth and expanding maps (with an appendix by Jacques
Tits). Inst. Hautes tudes Sci. Publ. Math., 53, 1981, 53-73.
[6] M. Gromov, Carnot-Carathodory spaces seen from within, in: Sub-Riemannian Geometry,
A. Bellaiche, J.J. Risler eds., Progress in Mathematics, 144, Birkhuser, (1996), 79-323.
[7] I. Harmander, Hypoelliptic second order dierential equations, Acta Math., 119, 1967, 147-171.

351

[8] G. A. Margulis, G.D. Mostow, The dierential of a quasi-conformal mapping of a CarnotCarathodory space, Geam. Funct. Analysis, 8 (1995), 2, 402-433.
[9] [9] G. A. Margulis, G.D. Mostow, Some remarks on the denit ion of tangent cones in a
Carnot-Carathodory space, J. D'Analyse Math., 80 (2000), 299-317.
[10] J. Mitchell, On Carnot-Carathodory metrics, Journal of Di. Geometry, 21 (1985), 35-45.
[11] P. Pansu, Mtriques de Carnot-Carathodory et quasiisometries des espaces symetriques de
rang un, Annals of Math., 129 (1989), 1-60.
[12] Gh. Vrnceanu, Sur les espaces non holonomes, C. R. Acad. Sci. Paris, 183, 852 (1926).
[13] Gh. Vrnceanu, Studio geometrica dei sistemi anolonomi, Annali di Matematica Pum ed
Appl., Serie 4, VI (1928-1929).
[14] Gh. Vrnceanu, Clasicarea grupurilor lui Lie de rang zero, Stud. Cercet. Mat., 1 (1950),
40-86.
[15] Gh. Vrnceanu, Groupes discrets lineaires, Rev. Mathm. Pure et Appl., VII, 2 (1962).
[16] Gh. Vrnceanu, Gruppi discreti e connessioni ani, Seminari dell 'Istituto Nazionale di Alta
Matematica (1962-63).
Institutul de Matematic al Academiei Romne,
Bucureti, Romnia
Marius.Buliga@imar.ro

C
de V

Colegiul Naional ,,u


din Brlad

MANIFESTRI TIINIFICE
Simpozion dedicat revistei ,,Recreaii tiinice 1 )
(1883-1888)

Academia Romn  liala din Iai a gzduit, n ziua de 15 martie a.c., simpozionul ,,125 de
ani de la apariia revistei Recreaii tiintice. Festivitatea s-a desfurat sub auspiciile Academiei
Romne, Facultii de Matematic a Universitii Al. I. Cuza, Catedrei de matematic a Universitii Tehnice Gh. Asachi i ale Asociaiei ,,Recreaii Matematice.
Cuvntul de deschidere a aparinut acad. Viorel Barbu, preedintele lialei Iai a Academiei
Romne, care a subliniat faptul c revista ,,Recreaii tiinice, este prima publicaie din ar cu
acest prol adresat tineretului. Alturi de alte evenimente ieene remarcabile ale anului 1883
 apariia ediiei T. Maiorescu a Poeziilor lui Mihai Eminescu, inaugurarea statuii lui tefan cel
Mare .a., apariia Rereaiilor tiintice reprezint un moment important al culturii i spiritualitii
romneti.
Fondatorii, distini profesori ai Universitii din Iai sau ai colilor ieene, ct i colaboratorii
revistei au asigurat o nalt inut tiinic acesteia. Revista a circulat n ntregul Regat al
Romniei de atunci i se remarc prin varietatea subiectelor abordate, rigoare, frumoasa limb
romn folosit i o grac excelent pentru acele timpuri.
Acad. V. Barbu menioneaz c, n cadrul simpozionului este lansat colecia integral
a revistei ,,Recreaii tiinice, reeditat n forma originar, nemodicat. Realizarea acestui
proiect de reeditare se datorete sprijinului material entuziast al doamnei Marinela Ghigea, director
al rmei Kepler Systemes d'Information, precum i muncii depuse de dr. Dan Tiba, cercettor la
1 ) Textul este reprodus dup cel aprut n revista ,,Recreaii Matematice, an X, nr. 3, 2008,
pp. 99-108. (N. R.)

352

Institutul de matematic al Academiei Romne i de prof. dr. Temistocle Brsan de la Universitatea


Tehnic Gh. Asachi.
Cuvntrile inute de acad. Radu Miron, prof. dr. Vasile Oproiu, prof. dr. Dorin
Iean, m.c. al Academiei, prof. dr. Teodor Precupanu, n aceast ordine, sunt prezentate mai
jos. Programul simpozionului se ncheie cu proiecia unor documente privind revista ,,Recreaii
tiinice i epoca n care a aprut aceast prezentate de prof.dr. Temistocle Brsan.

Recreaii tiintice  125 ani de la apariia primului numr


Acad. prof. dr. doc. Radu Miron

La 15 ianuarie 2008 s-au mplinit 125 de ani de la publicarea numrului 1 din primul volum
al revistei ,,Recreaii tiinice. Menit a face educaie tiinic tineretului din Regatul Romniei,
revista, care a avut o existen de numai ase ani, a depit graniele n toate zonele locuite de
romni. ninat de zece oameni nvai din vechea capital a Moldovei, ea avea s imprime n
contiina locuitorilor acestui pmnt primele capitole elevate de istorie a tiinelor din ar  i
cele nti lecii pentru un nvmnt modern n domeniul tiinelor exacte. Peste veac s-a vzut
nrurirea covritoare a ideilor vehiculate n cuprinsul acestei reviste, n coala de toate gradele i
n cercetare, conducnd la integrarea noastr n rndul rilor care aveau deja tradiii seculare.
Datorit coninutului preponderent matematic se poate arma cu deplin temei c revista a
deschis prima pagin, a matematicilor romneti. Aa cum sa remarcat mai trziu, dac publicaia
s-ar  numit ,,Recreaii matematice, ea ar  constituit prima publicaie din lume n domeniu, care
se adreseaz tineretului. Este adevrat c la apte ani de la dispariia ,,Recreaiilor tiinice, n
1895 a fost ninat ,,Gazeta Matematic cu adres special pentru tineretul romn. Dar aceast
revist este considerat a doua din lume ca prol i destinaie.
Evident, apariia ,,Gazetei, aa cum sublinia Gheorghe ieica, a fost impulsionat de
,,Recreaiile tiinice.
Acum, la 125 de ani de la ninare a celebrei reviste, se cuvine s exprimm omagii profunde
memoriei fondatorilor: N. Culianu, C. Climescu, I. Melic de la Universitatea Al. I. Cuza, G. I.
Lucescu, V. Paladi, G.I. Roiu, I. D. Rallet, G. Zarifopol, I. V. Praja i I. M. Dospinescu din
nvmntul preuniversitar ieean. Prin competena, pasiunea i sacriciile personale fcute cu
generozitate ei au reuit s trezeasc interesul pentru tiin, n general i s stimuleze gustul pentru
matematici n special.
Sunt emoionante cuvintele scrise ntr-un editorial al revistei: Credem c noi am tras cea
nti brazd care conduce ctr lucrri originale. Brazda-i mic ii ngust, dar exist !
Personalitatea fondatorilor este bine cunoscut. Ei sunt prezentai de George t. Andonie
n volumul 1 din Istoria Matematicii n Romnia. Majoritatea lor sunt oameni de tiin cu studii
nalte fcute n Frana, Italia, Olanda i Germania. Un gnd de recunotin colaboratorilor, nu
mai puin celebri: M. Tzony, V. Costin, P. Tanco, C. Gogu i rezolvitorilor pasionai, elevi pe
vremea aceea, E. Pangrati i D. Pompeiu.
Nu trebuie s uitm pe oamenii de- tiin care au susinut peste timp importana revistei
i impactul ei n cultura romneasc. Citm doar civa dintre ei: Alexandru Myller, Octav Mayer,
Ilie Popa, Gheorghe Gheorghiev, Gheorghe Banta, Gheorghe ieica, G. t. Andonie, N. N.
Mihileanu etc.
Condiiile istorice n care a aprut n 1883 revista ,,Recreaii tiinice nu erau dintre
cele mai favorabile. Unirea Principatelor abia se nfptuise, Regatul Romniei era abia ntemeiat,
Rzboiul de Independen din 1877 lsase urme adnci n contiina romnilor, alfabetul chirilic
fusese nlocuit cu cel latin, limba romn literar abia i denitivase procesul de unicare, romnii
i armau n mod decisiv aspiraia spre o societate modern. n atari condiii, dei apruser cu 23
de ani nainte universitile din Iai i Bucureti, coala de toate gradele trebuia profund recldit.
Era nevoie imperioas de regndit programarea curricular, de pregtit personalul didactic, de scris
manuale bune n limba romn, de construit coli etc.
n atari condiii spirituale, materiale i sociale dure, a pune bazele unei reviste de cultur
tiinic era un act de curaj, de patriotism. El costituia o important realizare destinat poporului
nostru. Soliditatea acestui ediciu este dat de calitatea tiinic, didactic i educaional a
subiectelor publicate, de limbajul tiinic adoptat, de graca de excepie utilizat n acea vreme.
Am prezentat aceste aspecte n articolul ,,Centenarul revistei Recreaii tiintice, Probleme de
istoria i lozoa tiinei, vol. X, 1984, Filiala Iai. Valabilitatea armaiilor fcute atunci i
pstreaz temeiul i astzi. Din acest motiv reproduc o parte din text:

353

Tonul ntregii producii matematice, cuprinznd mai bine de 90% din cele 1920 de pagini
ct nsumeaz aceast revit, a fost dat n primul rnd de fondatorii ei, care, prin prestigiul
lor, au atras foarte curnd valoroi colaboratori: Miltiade Tzony  profesor de mecanic teoretic
la Universitatea din Iai, Candide (probabil Victor Costin, pe atunci student la Paris), Iacob
Solomon - inginer, Paul Tanco  profesor de matematic i zic la Gimnaziul Superior din
Nsud, Constantin Gogu  profesor de geometrie analitic la Universitatea din Bucureti, Vasile
Butureanu  profesor de mineralogie i petrograe la Facultatea de tiinte din Iai . a.
n paginile revistei sunt publicate articole, note, probleme i soluii din domenii ca: aritmetic, algebr, geometrie elementar, geometrie analitic i diferenial, calcul diferenial i
integral, mecanic, astronomie, istoria matematicii, chimie, zic, geograe, Apare prima traducere a crtii nti din celebrele ,,Elemente ale lui Euclid. Miltiade Tzony tiprete n coloanele ei
o remarcabil culegere de probleme de mecanic teoretic. Geometria proiectiv, domeniu de mare
actualitate n acea vreme, este prezent prin traducerea primelor opt paragrafe din vestita lucrare
,,Geometria de pozictie a lui Criristian von Staudt. ntile elemente din istoria matematicilor n
antichitate sunt transpuse n limba romn de Iacob Solomon.
La succesul binemeritat al Recreaiilor tiinice a contribuit i prezentarea grac excelent. Scris ntr-o limb literar elevat, revista are, cu exceptia unor termeni matematici n
formare, ceva din culoarea i prospetimea revistelor actuale.
Privit global, ca act de cultur tiinic, revista rivalizeaz cu cele mai bune publicaii
de acest gen tiprite acum pe plan monidial.

nchei aici relatarea din articolul amintit. Dar ultima fraz trebuie corectat cu ,,acum un
secol i un sfert pe plan mondial.
Subliniez faptul c n tot cuprinsul celor ase volume ale revistei impresioneaz grija pentru
rigoarea prezentrii, acuratea exprimrii in limba romn, actualizarea expunerilor, informaia
de ultim or, profunzimea raionamentelor i, nu n ultimul rnd, atenia acordat contribuiilor
personale ale tinerilor rezolvitori sau autori ale problemelor propuse spre publicare.
A fost realizat astfel n premier, o revist romneasc extrem de important pentru
invmitnt i cercetare n tiinele exacte, de acelai nivel cu reviste similare consacrate i vestite
din lume. Dup apte ani de la stingerea activitii acestei reviste, ideea de a rspndi n rndul
tineretului pasiunea pentru matematic va  preluata de ,,Gazeta Matematic.
La 125 de ani de la apariia revistei ,,Recreaii tiinice, generaiile de astzi omagiaz
acest eveniment ca semn de adnc recunotin adus naintailor notri pentru contribuia lor
inestimabil la tezaurul tiinei i culturii romneti.
i un scurt adaos: Centenarul apariiei revistei ,,Recreaii tiinice a fost organizat n 1983
de matematicieni ieeni n Seminarul Matematic ,,Alexandru Myller, iar srbtorirea celor 125 de
ani de la apariie a fost iniiat tot n cadrul acestui Seminar pregtit ind de Asociaia ,,Recreaii
matematice, Facultatea de matematic i Institutul de Matematic ,,Octav Mayer de la Filiala
din Iai a Academiei Romne. Asociaia ,,Recreaii matematice i face un titlu de onoare prin
reeditarea integral, exclusiv prin grij proprie, a coleciei revistei ,,Recreaii tiinice. Acest fapt
l datorm prof. univ. Temistocle Brsan de la Universitatea Tehnic Gheorghe Asachi din Iai,
cercettorului dr. Dan Tiba de la Institutul de Matematic al Academiei Romne din Bucureti i
doamnei Marinela Ghigea  director al rmei Kepler Systemes d'Information. i asigurm de toat
preuirea i gratitudinea noastr.

Rolul i ponderea geometriei n revista ,,Recreaii tiintice


prof. dr. Vasile Oproiu

n revista ,,Recreaii tiinice, scris i editat de un grup de oameni de tiina i cultur


inimoi (N. Culianu, C. Climescu, I. Melik, G. I. Luceacu, V. Paladi, G.I. Roiu, I. D. Rallet, G.
Zarifopol, I. V. Praja i I.M. Dospinescu ), s-au adunat i publicat diferite materiale din domeniile
matematicii, zcii, chimiei, geograei, cosmograei, topograei, mineralogiei, istoriei matematicii
etc. Revista se adresa elevilor din clasele de gimnaziu i liceu, dar i altor categorii de persoane
interesate de cunoatere: profesori, studeni, funcionari, militari etc.
Geometria, ca ramura a matematicilor are o pondere destul de nsemnata n paginile revistei.
Trebuie s menionm, de la nceput, c G.I. Roiu public ntre anii 1883-1885 prima carte a
Elementelor lui Euclid (traducere dup o ediie italian). Am regsit cu o anumit emoie i
nostalgie multe formulri pe care le ntlnisem cnd eram student i apoi le citisem n crile lui
Emov (ediia n limba romn i cea n francez) i n cartea lui I. Vaisman. Astfel (n volumul

354

II al ,,Recreaiilor), printre deniiile lui Euclid am regsit formulri precum: Punctul este aceea
ce nu are pri, adec nu are nici o mrime, Linia este lungime fr lrgime, Suprafaa plan este
aceea care este aezat egal n respectul tuturor liniilor sale drepte. Postulatele i axiomele au fost
publicate anterior, n primul volum, ntr-o ordine diferit de cea cu care suntem obinuii. Astfel,
faimosul postulat V al lui Euclid apare ca axioma XII. Autorul prezint i deniiile i axiomele, aa
cum au fost prelucrate de Legendre n Geometria sa, ediia V, Paris, 1804. Legendre nu denete
punctul i, n legatur cu deniia, formuleaz urmtoarea aseriune: Denitia unui lucru este
exprimarea raporturilor sale ctre lucruri cunoscute. Dup care, se ncumet s denesc dreapta:
Linia dreapt este drumul cel mai scurt de la un punct la altul. Mai sunt prezentate comentarii
critice ale diverilor matematicieni relativ la aceste deniii, inclusiv noi deniii: Cea mai simpl
din toate liniile este linia dreapt a cria noiune este familiar la toi i despre care ne d o
idee un r ntins.

Apoi sunt prezentate propoziiile de la I la XXII. Cum spuneam, prezentarea


,,Elementelor continu n volumul III cu propoziiile rmase i cu exerciiile la cartea I.
n revist sunt prezentate numeroase aspecte ale geometriei elementare, utile elevilor, profesorilor i altor persoane interesate: maxime i minime geometrice, media i extrema raie, calculul
lungimilor unor linii importante din triunghi, calcule pentru patrulaterul inscriptibil (n primul
volum), proprietai ale poligoanelor i dreapta lui Simson (n vol. II), calcularea volumelor piramidei trunchiate i al conului trunchiat, proprietai sintetice ale elipsei (n vol. III), teoria
transversalelor, diviziunea armonic, fasciculul armonic, poli i polare, geometria de poziie a lui
Staudt (n vol. IV); aceasta din urm se continua i n volumul V. Chestiunile de geometrie analitic
sunt considerate separat i se refer la: seciuni plane n conul drept, construcii de curbe, cu exemplicri din clasele curbelor celebre, tratarea acestora n coordonate polare, plane principale la
suprafeele de gradul al doilea.
O seciune important n revist este cea a problemelor propuse (de regul, n jur de 10
probleme la ecare numr), la care se adug, pe parcurs, cea cu rezolvrile i listele de rezolvitori.
Trebuie s menionm c, n Regatul Romniei de atunci, existau cteva zeci de gimnazii i licee
(oricum, sub 30) i c numrul celor care rezolvau probleme era destul de mic. Moda rezolvrilor
de probleme la reviste de matematic nu prinsese nc. Menionm c existau i colaborri venite
de la elevi din Transilvania, Banat i alte regiuni ale viitoarei Romnii Mari.
Ca o apreciere cu caracter general, coninutul revistei ,,Recreaii tiinice era destul de
ridicat din punct de vedere al nivelului chestiunilor de geometrie tratate. Subiectele erau interesante
i atractive pentru numeroi cititori. Cred c, n redacia revistei, erau persoane care doreau s
fac revista ct mai atractiv.
Rsfoind cele ase volume am dat i peste un articol fascinant de la secia cosmograe, scris
de G.I. Lucescu, n care se explica n ce manier au fost concepute calendarele iulian i gregorian
i c motivul pentru care s-a fcut trecerea de la unul la altul a fost legat de ideea c, n acord
cu hotrrea Conciliului de la Niceea din anul 325, punctul de plecare pentru xarea zilei de Pati
trebuia s e echinociul de primvar i acesta trebuia s e mereu la 21 martie. Dup aceea,
se ateapta prima noapte cu lun plin i Patele se xa n duminica imediat urmtoare (astfel,
n 2008, noaptea cu lun plin cade exact n 21 martie i Patele catolic este xat n 23 martie:
xarea Patelui ortodox este mult mai complicat i ine de nite date din calendarul iudaic). De
la data Conciliului de la Niceea pn n 1582 calendarul iulian rmsese n urm cu 10 zile faa
de calendarul real (anul din calendarul iulian era puin mai lung dect anul real). Acest lucru
inuena foarte mult diverse activitai practice, de exemplu, unele lucrri agricole ce se fceau n
strns legtur cu srbtorile religioase. n 1582, papa Grigore al XIII-lea a dat o bul prin care se
decidea avansarea calendarului iulian, existent, cu 10 zile i c anii multipli de sute (mai puin cei
multipli de 400) nu sunt biseci. Acest calendar mai are o mica eroare care const n rmnerea n
urm cu o zi n circa 3300 ani, eroare considerat rezonabil i care va  corectat n viitor. n ar
la noi, calendarul grigorian a fost adoptat n 1923 (s-a trecut la stilul nou!), dat cnd ntrzierea
calendarului iulian fa de cel real, sau cel grigorian, ajunsese la 13 zile.
Revenind la revista ,,Recreaii tiinice, apreciez c un cititor interesat poate s gseasc
n cuprinsul ei lucruri incitante, att n domeniul geometriei, ct i n alte domenii ale matematicilor
i ale altor tiine.

Despre problemele de mecanic


Prof. dr. Dorin Ieean, m. c. al Academiei

Pe lng alte chestiuni interesante, n revista ,,Recreaii tiinice au aprut i un numr


de probleme de mecanic raional, semnate de Miltiade Tzony. Pe vremea cnd a publicat aceste

355

probleme, M. Tzony era profesor de mecanic teoretic la Universitatea din Iai (a funcionat n
aceast calitate n perioada 23.X.1869 - 15.III.1898).
Miltiade Tzony este autorul unui curs de mecanic, prezentat n manuscris, n dou volume.
Primul volum a fost scris n 1869, iar al doilea volum n 1881. Pe prima pagin a acestui curs
autorul scrie: ,,Cursul de Mecanic rationale i aplicat; Profesat la Universitatea de Jasy; Dup
cei mai buni autori francezi: Delaunay, Sturm, Duhamel, Bellanger, Bresse, Bour, Collignon,
Mesal.

Chasles: de Miltiade Tzony, Licentiat n tiine matematice de la Sorbona din Paris,

Ingineriu al coalei de poduri din Paris, Vechiu elevu al coalei politchnice din acestu orau,
Profesor al Universitii de Jasy i a Lyceului Nou din Jasy .

Leciile de mecanic raional de la


Universitatea din Iai se fceau dup acest curs.
n perioada 1885-1888, M. Tzony public Un curs de probleme n revista ,,Recreaii tiinice. El mrturisete c acest lucru l face ,,n scopul de a uura studentilor Universitilor
noastre completa pricepere a cursului de mecanic rational i nimerita ntrebuinare a principiilor acestei nsemnate tiine 

(vol. III, pp. 77-78). S vedem care este originea problemelor i a


rezolvrilor date. M. Tzony ne spune c ,,problemele sunt lucrate dup diveri autori ntre care
gureaz n primul loc abatele Jullien, a crui carte n aceast materie a devenit clasic . Am
constatat c toate cele 98 de probleme publicate de Tzony n ,,Recreaii tiinice sunt luate din
cartea clugrului P.M. Jullien ,,Problmes de mcanique rationnelle, aprut la Paris n anul
1855. Cartea lui P.M. Jullien conine att probleme originale ct i probleme ale altor autori.
n ,,Recreaii tiinice M. Tzony prezint 36 probleme ale crui autor este P.M. Jullien, 24 de
probleme datorate lui W. Walton i 28 probleme ale altor autori (printre care Euler, Bernoulli,
Leibniz, Laplace, Gauss, Mbius ). Menionm c problemele datorate lui W. Walton se gsesc n
cartea acestuia A Collection of Problems in Illustration of the Principles of Theoretial Mechanics,
aprut la Cambridge n anul 1842.
La nceputul prezentrii problemelor, M. Tzony arm c ,,de cte ori ne va  posibil vom
indica la nea ecrei probleme autorul cruia se datorete . Problemele publicate de Tzony sunt
i rezolvate i ,,cu toate indicaiunile necesare pentru a putea  cuprinse cu uurin de tnrul
public cetitoriu cruia este n special destinat . M. Tzony susine, pe drept cuvnt, c ,,opera
abatelui Jullien este scris intr-un mod att de laconic nct cetirea ei de nceptori este foarte
laborioas i n unele puncte aproape cu totul neneleas ".

Dintre problemele publicate de Tzony


n ,,Recreaii tiinice un numr de 51 sunt rezolvate n cartea lui P. M. Jullien. Celelalte sunt
probleme pe care Jullien le-a propus spre rezolvare. O parte dintre problemele nerezolvate de
Jullien sunt ns nsoite de guri i rspunsuri n cartea lui W. Walton. Am comparat aceste
rezolvri i guri cu cele date de M. Tzony n ,,Recreaii tiinice. Se poate spune cu certitudine
c Tzony nu s-a inspirat din cartea lui W. Walton.
Fiecare capitol din culegerea de probleme este prefaat cu ,,o scurt amintire a rezultatelor
nale ale teoriei, ntovrite de cteva noiuni istorice pe care, n lipsa altor documente, le vom
mprumuta pentru cea mai mare parte din opera tiinic de care facem meniune .

Este uor de
vzut c aceste comentarii sunt traduse din cartea lui P. M. Jullien.
Menionm c rezolvrile prezentate de M. Tzony sunt clare, iar gurile sunt ngrijite i
binevenite. Un lucru remarcabil este faptul c i n cazul problemelor rezolvate de Jullien, M.
Tzony face guri suplimentare i adaug explicaii. n problemele publicate de Tzony n ,,Recreaii
tiinice sunt 88 de guri, dintre care 66 nu se a n cartea lui Jullien.
Referitor la repartiia pe ani a problemelor se constat c n anul 1885 sunt publicate 18
probleme, n anul 1886 apar 33 probleme, n anul 1887 sunt publicate 23 probleme, iar n anul 1888
apar 18 probleme. n anul 1888 revista ,,Recreaii tiinice i nceteaz apariia. Acest lucru a
curmat publicarea reasc a altor probleme.
Menionm c problemele aprute se refer doar la partea de Static a cursului predat de
Tzony la Universitate. Dintre acestea, 18 se refer la echilibrul punctului material,39 la echilibrul
corpului rigid, 6 trateaz echilibrul unui sistem de bare articulate, 27 sunt dedicate echilibrului
relor, iar 8 se refer la principiul ,,lucrului mecanic virtual .
Cursul lui M. Tzony (n manuscris) i problemele publicate de el n ,,Recreaii tiinice
au stat la baza nvmntului Mecanicii din ara noastr.
n afar de activitatea de profesor, Miltiade Tzony s-a remarcat prin munca sa depus n
vederea propirii Romniei. A fost senator, secretar de stat la Ministerul Construciilor Publice,
director al C.F.R. Printre altele, oraul Iai i datoreaz pavarea strzilor.
Despre M. Tzony se pot spune multe lucruri. Un fost elev de-al su, Petru Culianu, care a
urmat cursurile de mecanic de la Universitatea din Iai n anii 1890, l descrie pe Miltiade Tzony

356

astfel: ,,Cu mintea agera, cu gura frumoas (a fost luat ca model de pictorul Grigorescu pentru
unele guri din biserica de la mnstirea Agapia) ce corespund ntru totul nobleei caracterului
su, el a fost cu totul devotat datoriei i naltei misiuni a profesorului .

Problematica de algebr i analiz matematic n revista Recreaii


tiinice Prof. dr. Teodor Precupeanu
Pentru matematica romneasc, apariia revistei ,,Recreaii tiinice din iniiativa unei
elite de profesori ai universitii i ai liceelor din Iai, reprezint un moment important, de nceput,
pentru crearea unei atmosfere propice dezvoltrii tiinelor matematice, de atragere a tinerilor,
stimulndu-i i amplicndu-le pasiunile, cluzindu-i spre problemele moderne ale acelei perioade.
Este de remarcat faptul c iniiatorii revistei erau la curent cu multe din preocuprile
existente n matematica european, avnd o bun informare, facilitat de accesul la o serie de
reviste importante, ndeosebi din Frana, Italia i Germania. Sunt semnalate nu numai apariia unor
rezultate importante, ci i diverse evenimente ale comunitii tiinice, cum ar , spre exemplu,
apariia revistei Acta Matematica fondat de Mittag-Leer prin casa regal suedo-norvegian,
solemnitatea retragerii din nvmnt la vrsta de 70 de ani a marelui matematician EugeneCharles Catalan, apariia unor tratate importante de matematic, discuiile generate de proiectul
Turnului Eiel, ce urma s se construiasc n Paris.
Adresndu-se n primul rnd elevilor din nvmntul secundar, revista a stimulat, de
asemenea, preocuprile profesorilor pentru modernizarea nvmntului matematic, gzduind n
paginile sale dezbateri interesante cu caracter metodic asupra programelor analitice i a metodelor
prin care s e atrai elevii pentru studiul matematicii, s se asigure o ct mai bun accesibilitate,
punnd pe primul plan intuiia i dezvoltarea abilitilor de rezolvitori de probleme.
S remarcm faptul c n acea perioad, sfritul secolului al XIX-lea, unele discipline
componente ale matematicii nu erau nc bine conturate, n acord cu felul n care ele erau concepute
la marile universiti europene, inuenate de crile importante de matematic din acei ani. Abia
apruse cursul de analiz matematic al lui Sturm (1880) i cel de calcul diferenial i integral al lui
Catalan (1878), autor i al unei cri despre serii, unic la acel moment, cri ce urmau tratatelor
celebre scrise de Leibniz, Bernoulli sau Serret.
Algebra i analiza matematic este prezent n paginile revistei att prin articole cu caracter
teoretic informativ asupra unor chestiuni importante, ct i printr-o gam variat de exerciii i
probleme ce vizau i pe studenii anilor pregtitori pentru colile politehnice din ar sau din
strintate. Semnalm astfel mai mult articole scrise de C. Climescu dedicate numerelor complexe
(numite cantiti imaginare), numere care nc erau evitate de muli matematicieni. ntlnim,
de asemenea, demonstraia teoremei fundamentale a algebrei precum i unele consideraii asupra
dreptelor imaginare sau asupra unor funcii complexe, date ns sub form implicit prin relaii
polinomiale. Mai mult, n unele probleme apar integrale ale unor funcii complexe, tiindu-se a se
depi aspectele dicile de multivocitate i determinndu-se cu claritate valorile lor principale.
n cadrul algebrei sunt cuprinse i seriile numerice. dezvolt.ile tayloriene, concepute ca
sume (numerice sau polinomiale) innite. Convergena acestora neleas intuitiv se reduce de fapt
la extinderea operaiilor numerice algebrice i pentru (nu reiese dac se folosea i ), ind
cunoscute limitele fundamentale. Se foloseau ns pentru convergen criterii ne, aproape toate
cele cunoscute astzi. Autori ai articolelor respective sunt C. Climescu, I.D. Rallet i I. V. Praja.
Erau folosite n mod uzual dezvoltrile n serii de puteri ale funciilor elementare. Tot ca fcnd
parte din algebr sunt prezentate cunoscutele formule Lagrange i Newton de interpolare ntr-un
articol foarte interesant scris de Alex. Sadoveanu.
n primul volum, din 1883, problema dezvoltrii funciilor n serie este dat ca fcnd
parte din Analiza algebric iar Calculul integral era considerat separat de Analiza matematic. De
fapt, analiza matematic era conceput la acel moment numai ca teorie a derivabilitii avnd ca
principale rezultate teorema de medie a lui Lagrange i condiiile suciente de extrem la funcii de
una sau mai multe variabile. Problemele de calcul de arii sau volume erau considerate ca fcnd
parte din geometrie sau intervenind n probleme de mecanic.
Noiunea de derivat era acceptat prin interpretrile ei: geometric  de tangent  sau cele
din mecanic. Mai mult, trebuie avut n vedere c nsi noiunea de funcie, fundamental pentru
ntreaga matematic, era conceput n acele timpuri n accepia eulerian, ceea ce corespunde astzi
funciilor elementare.
Menionm c ecuaiile difereniale sunt frecvent ntlnite n partea de mecanic i de geometrie a curbelor plane (probleme concrete de aare a unor curbe dac se dau anumite proprieti

357

metrice legate de tangente) i nu sunt prezentate ca aparinnd disciplinei de astzi Ecuaii difereniale, ceea ce este normal, ntruct aceast disciplin avea s se contureze n matematic mult
mai trziu. Este ns prezentat in cadrul Analizei matematice problema schimbrii de variabil
cu suportul oferit de ecuaiile difereniale i schimbrile de coordonate (ndeosebi polare i sferice)
deosebit de importante n mecanic, geometrie i astronomie.
Problemele de algebr i analiz matematic prezente n cele ase volume ale revistei sunt
deosebit de frumoase i ilustrative, de mare diversitate, oferind o imagine exact a coninutului
disciplinelor de matematic din acea perioad. Aproape toate pot  regsite de fapt n culegerile
de probleme de astzi.
n ncheiere, subliniem nc odat rolul remarcabil avut de revista ,,Recreai i tiinice n
dezvoltarea i impulsionarea nvmntului matematic romnesc n concordan cu cel european,
ce era urmrit ndeaproape.
Nu aveau s treac muli ani dup ncetarea brusc a apariiei acestei reviste i la universitatea ieean un fost rezolvitor al ,,Recreaiilor tiinice elabora primele lucrri originale
de matematic. Este vorba de marele matematician romn Dimitrie Pompeiu, ale crui rezultate
privitoare la teorema creterilor nite, obinute n perioada cnd funciona ca profesor al Universitii din Iai, sunt citate i astzi impresionnd prin profunzimea i elegana lor. Cercetrile
sale de analiz matematic sunt de fapt primele cercetri tiinice originale de matematic la
Universitatea din Iai.
Invocnd anterior numele lui Catalan, matematician cu vaste preocupri matematice (analiz, algebr, geometrie, teoria numerelor), se cuvine a aminti numele unuia dintre primii autori
romni ai unor cercetri tiinice originale de matematic,N. t. Botez, care stabilete o frumoas
identitate legat de seria armonic, cunoscut azi ca identitatea Catalan-Botez, rolul lui Catalan
ind acela de a o meniona n unul din articolele sale cu precizarea lui Botez ca autor.
Revista ,,Recreaii tiinice constituie pasul premergtor apariieil ,,Analelor tiinice
ale Universitii Al. I. Cuza, n 1900, revist dedicat cercetrilor originale ale profesorilor Universitii ieene, dar care a publicat nc din primele numere i articole ale unor recunoscui matematicieni strini ca Lucien Godeaux, Mauro Picone, Kentaro Yano, T. J. Willmore .a.

DIN VIAA SOCIETII


coala de var de la Buteni
Anul acesta a XII-a ediie a cursurilor de var pentru perfecionarea profesorilor din nvmntul preuniversitar s-a desfurat , ca deobicei la Buteni, n perioada 28 iulie-7 august.
Cursurile , reluate de S. S> M. R. n 1997, au o tradiie mult mai veche, ele desfurndu-se,
pentru prima dat, n anul 1957 n localitatea Scele i suferind o mic ntrerupere n anii '90.
Conform tradiiei nstpnite n ultimii ani, gazda cursurilor a fost Centrul de Pregtire
pentru Personalul din Industrie care ne-a asigurat  prin persoana domnului director general Irinel
Ghi i a subordonailor si  condiii deosebite att n privina desfurrii propriu-zise a cursurilor, ct i n privina cazrii participanilor. Le adresm, pe aceast cale, mulumirile noastre.
Programul zilnic a cuprins trei module (conferine)  inclusiv smbt  pentru a acoperi
un numr de 40 de ore, n intervalul 29 iulie - 6 august, n care s-au desfurat cursurile propriuzise. La ncheierea lor, pe 6 august, participanii au susinut un colocviu, n cadrul c aruia au
fost prezentate cele mai interesante referate selectate de comisia de examinare. Subiectele referatelor au fost la alegerea cursanilor, viznd, n general, teme tiiice sau metodice, dar i unele
probleme de istoria matematicii, precum i problematica general a nvmntului preuniversitar.
Diversicarea tematicii abordate de cursani constituie o caracteristic a ultimilor ani, ea reectnd
lrgirea ariei de preocupri a profesorilor, tendin cta acestora de a se ancora mai ferm n realitate,
prelund tradiiile naintailor. Vom remarca, n mod deosebit, seriozitatea i competena cu care
cursanii au tratat tematica aleas  desigur, n mare msur clasic  foarte muli dintre ei viznd
deschideri ctre alte domenii ale nvmntului sau ale vieii de zi cu zi sau conexiuni metodice
interesante i cu un vdit caracter original. Comisia de selecie a lucrrilor a fost alctuit din
acad. Ioan Tomescu, prof. univ. dr. Dorel Duca i semnatarul acestor rnduri.
Fa de anul trecut s-a nregistrat o cretere notabil a numrului de participani (54 fa
de 37), n ciuda faptului c aceste cursuri nu au fost nc acreditate pentru ca profesorii s poat

358

benecia de prevederile legale. Sperm ca, n urma demersurilor pe care le intreprindem anul
acesta, s obinem o ocializare a lor, mcar parial.
Spre deosebire de ali ani, repartiia zonal a participanilor a fost destul de uniform ,
meninndu-se totui, pe primul loc judeele din Moldova i, dintre acestea, judeul Iai unde se
remarc, din nou, activitatea neobosit a profesorului Vasile Nechita  secretarul lialei locale. Ca
un fapt pozitiv, vom meniona prezena unui numr mai mare de bucureteni (10), fa de numrul
extrem de mic din anii precedeni.
Viul interes strnit de conferine printre participani a fost marcat de discuiile dintre
acetia i confereniari, n cadrul i n afara cursurilorprecum i de sondajul efectuat la nele
cursurilor. De altfel aceste cursuri au constituit totdeauna u teren fertil pentru schimbarea opiniilor
ntre cursani i ntre acetia i confereniari pe teme privind nvmntul matematic romnesc i
viitorul acestuia.
Tematica cursurilor a fost atent selectat, innd seama i de subiectele sugereate de cursani n sondajele efectuate n anii anteriori. n msura posibilitilor, am cutat s pstrm un
echilibru ntre subiectele cu caracter de informare tiinic i cele viznd metodica i metodologia
predrii la clas, primele avnd, evident. un caracter preponderent. Selectarea confereniarilr a fost
fcut a cu deosebit grij, ind solicitai cu precdere acei universitari, care n decursul timpului,
s-au aplecat cu interes i seriozitate asupra nvmntului preuniversitar, ind preocupai de perpetua mbuntire i diversicare a acestuia; nu n ultimul rnd, am inut seama i de simpatiile
cursanilor exprimate prin sondajele de opinie din anii precedeni. Acetia  confereniarii - au
tiut s stabileasc un mod de comunicare simplu i ecient cu profesorii cursani, fr a abuza de
informaia tiinici, ceea ce ni se pare esenial  neadoptnd o poziie ex cathedra.
Iat tematica conferinelor susinute:
prof. univ. dr Ioan Tomescu, membru corespondent al Academiei Romne (Universitatea
din Bucureti)  ,,Numere binomiale i multinomiale  (o conferin); ,,Principii de numrare (o
conferin); ,,Principiul dublei numrri (o conferin);
prof. univ. dr Constantin Popovici (Universitatea din Bucureti)  ,,Innitatea numerelor
prime (o conferin); ,,Calculabiltate (o conferin);
prof. univ. dr Doru tefnescu (Universitatea din Bucureti)  ,,Localizarea rdcinilor
polinoamelor (o conferin);
prof. univ. dr Radu Gologan (Universitatea Politehnic din Bucureti)  ,,Rezolvarea
problemelor date la O. I. M. 2008 (o conferin);
prof. univ. dr Adrian Albu (Universitatea de Vest din Timioara)  ,,Probleme de geometrie n spaiu tratate cu ajutorul cordonatelor (dou conferine); ,,Despre teorema lui Pitagora (o
conferin);
prof. univ. dr Dumitru Buneag (Universitatea din Craiova)  ,,Mulimi de numere
(dou conferine)
prof. univ. dr Constantin Niculescu (Universitatea din Craiova)  ,,Funcii convexe,
Inegalitatea lui Popoviciu (o conferin); ,,Teorema lui Rolle (o conferin);
prof. univ. dr Dorel Duca (Universitatea Babe-Bolyai din Cluj)  ,,Sistem de votare (o
conferin); ,,Probleme de competiie i conict (o conferin); ,,Rezolvarea problemelor de conict
i competiie (o conferin)
conf. univ. dr Drago Popescu (Universitatea din Bucureti)  ,,Teoremele lui MAntel i
Turan cu aplicaii n geometria combinatorial (o conferin); ,,Probleme de geometrie combinatorial (o conferin)
conf. univ. dr. Andrei Vernescu (Universitatea Valahia din Trgovite) ,,Utilizarea
corect a criteriilor de convergen (o conferin); ,,iruri de integrale (o conferin).
Ca i n anii precedeni, vom meniona prezena la cursuri a multora dintre ,,veterani, acei
profesori care constitue un adevrat nucleu al cursanilor i al cror lider necontestat este profesorul
Vasile Nechita, participant la toate cele 12 ediii. n acest sens vom meniona c anul acesta am
acordat o nou diplom de delitate(pentru participarea la apte ediii consecutive a cursurilor)
doamnei profesor Mariana Oleniuc de la coala cu clasele I-VIII din Blgeti  Pacani (jud. Iai)
O felicitm pe aceast cale, att pe domnia sa ct i pe toi ceilali ,,veterani ai cursurilor.
La nalul acestor rnduri mai trebuie s adresm mulumirile noastre sincere domnului
profesor Nicolae Angelescu  inspector general adjunct la I. . J. Prahova i preedinte al lialei
Prahova a S. S. M.R,  i domnului profesor Mirela Dobrea  directoare a grupului colar Ion
Kolinderu din Buteni  pentru sprijinul neprecupeit acordat n buna organizare i desfurare a
acestor cursuri. Acestora i nultor altora, anonimi, le exprimm gratitudinea noastr.

359

Dan Radu

Lista absolvenilor cursurilor de perfecionare pentru


profesorii de matematic, organizate de S.S.M.R.
Buteni, 29 iulie-7 august 2008
1.
2.
3.
4.
5.
6.
7.
8.
9.
10.
11.
12.

Ababei Constantin
Agapi Maria
Badea Aurelia Liliana
Bejan Cornelia Livia
Buju Aura Marinela
Burtea Anne-Marie
Busuioc Didina
Catan Maria
Chirea Elena
Cioban Liliana
Ciuc Rodica
Cojocea Manuela Simona

13. Cojocea Tatiana


14.
15.
16.
17.
18.
19.
20.
21.
22.
23.
24.
25.
26.
27.
28.
29.
30.
31.
32.
33.
34.
35.
36.
37.
38.
39.
40.
41.
42.
43.
44.
45.
46.
47.
48.
49.

Costchescu Mrioara
Crciun Liliana
Crciun Dorinel Mihai
Creu Ciprian
Dogar Anca Veronica
Duma Iuliana
Duma Vasile
Dumitrescu tefan Nicolae
Fren Angela
Gavrilu Mihai
Ghi Gabriela Iulica
Ghi Mariana Luminia
Marinescu Damian
Mihe Maria
Murar Aurelia
Mller Gina
Nstelu Maria
Nechita Vasile
Necule Elena
Negrea Ana Maria
Nica Paula
Ninulescu Elena
Ni ctic Ctlin
Nu Gabriela
Oleniuc Claudia
Oleniuc Mariana
Plici Aurelia
Popa Filofteia
Popescu Maria
Rtea Ana-Maria
Roman Neculai
Rotundu Raluca Ioana
Rusu Adriana
Seceleanu Daniela
Srghie Daniela
Stancu Ion

360

c. cu clasele I-VIII Mihai Eminescu  Roman


c. cu clasele I-VIII nr.2 George Clinescu  Oneti
Lic. Teoretic Dimitrie Bolintineanu  Bucureti
Universitatea Tehnic Gheorghe Asachi  Iai
Lic. Teoretic Petru Maior  Gherla
Gr. c.  Ora Ovidiu
Lic de Muzic Tudor Ciortea  Braov
Col. Na. Gheorghe Lazr  Bucureti
Lic. Teoretic Nicolae Blcescu  Medgidia
Col. Naional Unirea Trgu Mure
Gr. c. Ind. Anghel Saligny  Brila
Fac. de Cibernetic, Statistic i Informatic Economic
Bucureti
c. cu clasele I-VIII nr. 108 Alexandru Obregia
Bucureti
Lic. cu Program Sportiv  Roman
Col. Naional Unirea Trgu Mure
Col. Naional Mihail Sadoveanu  Pacani
Col. Tehnic Gheorghe Asachi  Iai
c. cu cl. I-VIII Sncraiu de Mure  Trgu Mure
Col. Naional Vasile Alecsandri  Galai
c. Gimnazial nr. 26 Ion Creang  Galai
Col. Naional tefan Velovan  Craiova
Col. Tehnic Feroviar  Braov
Col. Na. Roman Vod  Roman
Lic. Pedagogic Spiru Haret  Buzu
Colegiul B. P. Hadeu  Buzu
c. cu clasele I-VIII Tudor Vladimirescu  Trgovite
Col. Tehnic Danubiana  Roman
Lic. cu Prog. Sportiv Banatul  Timioara
Lic. Ec. C. C. Kiriescu  Bucureti
Col. Naional tefan cel Mare  Trgu Neam
Col. Costache Negruzzi  Iai
Gr. c. Forestier  Cmpina
Gr. c. Traian Vuia  Trgu Mure
Gr. c. Ind. Gheorghe Asachi  Bucureti
Gr. c. I. N. Roman  Constana
Col. Tehnic Dimitrie Leonida  Bucureti
Col. Naional Unirea  Trgu Mure
Gr. c. Virgil Madgearu  Iai
c. cu clasele I-VIII Blgeti  Bacani
Col. Naional Octav Onicescu  Bucureti
c. cu clasele I-VIII nr. 12  Trgovite
coala Central  Bucureti
Gimnaziul de Sta Octavian Goga  Sighioara
c. cu cl. I-VIII Vasile AlecsandriMirceti,
c. cu cl. I-VIII Ionel Teodoreanu  Iai
Gr. c. de Arte i Meserii  Oltenia
Gimnaziul Europa  Trgu Mure
Col. Na. Al. I. Cuza  Focani
Lic. Teoretic Dimitrie Bolintineanu Bucureti

50.
51.
52.
53.

Stniloiu Nicolae
tefan Margareta
ilic Daniela
Vian Ion

54. Zidu George Daniel


37. Zaharia Dan

Gr. c. Industrial  Boca


Gr. c. Forestiar  Cmpina
Gr. c. Gheorghe Asachi  Bucureti
Lic. Teoretic Tudor Arghezi  Penitenciarul de Maxim
Siguran  Craiova
Lic. Teoretic Dimitrie Cantemir  Oneti
Col. Na. Dimitrie Cantemir  Bucureti
Dan Radu

Profesorul Mihai Gavrilu la 70 de ani


S-a nscut la 10 mai 1938 n frumosul ora Piatra Neam, din judetul Neam. Datorit
evenimentelor declanate de cel de-al doilea rzboi mondial, familia a fost nevoit s se refugieze la
Rmnicu-Vlcea, pentru o perioad. Apoi s-au ntors pe meleagurile moldave, alegnd ca reedina
oraul Roman (1944). Urmeaz coala primar ,,Sf. Gheorghe din localitate, iar din clasa a V-a
i pn n clasa a X-a, cursurile liceului de biei (actualul Colegiu National ,,Roman Vod. n
1962 termina Facultatea de Matematica de la Universitatea ,,Al. I. Cuza din Iai, ind repartizat
la liceul, unde fusese elev. n perioada 1970-1975 urmeaz cursurile Facultii de Educaie Fizic
i Sport din Bucureti, la secia fr frecven. n 1974 se cstorete i are dou fete, care nu l-au
urmat n profesie: una a terminat medicina i cea mic nane-bnci. n perioada 1979-1989 este
director la Liceul Agricol din localitate, iar n 1989 revine la C. N. ,,Roman Vod. Din martie
1990 si pn n 1998 lucreaz ca inspector colar la I. . J. Neam. n anul 2000, se pensioneaz,
n ,,scripte, dar nu i n realitate aa dup cum vom vedea n cele ce urmeaz.
Rsfoind cartea ,,Pagini din istoria Liceului ,,Roman Vod (l872 -1972) din Roman, 1972,
gsim consemnate urmtoarele:
Printre premianii sau menionaii Gazetei Matematice-Seria B, pentru activitate deosebit ca rezolvitori de probleme, pentru problem propuse sau note publicate, aproape n care
an, gsim elevi ai liceului, ca de exemplu Gavrilu Mihai, clasa a X-a-1954.
Prin activitatea deosebit n cadrul Cercului de matematic, concretizat i prin participarea la etapa nal a Olimpiadei, s-au evideniat n mod deosebit elevii Ilioi Constantin i
Gavrilu Mihai.
Domnul profesor Mihai Gavrilu, are bucuria i onoarea, ca dup terminarea Facultii de
Matematic,n centrul universitar Iai, s se ntoarc s predea matematica la Liceul Roman Vod
 pe care l terminase n Seria anului 1955  n perioada 1962-1979 i 1989-2000, fapt care, n acele
timpuri, a fost un el pentru multe generaii de elevi, care, prin terminarea studiilor au devenit
profesori i care au dorit ca i ei s pun o piatr de temelie la viitorul tinerei generaii.
Domnul profesor Mihai Gavrilu, s-a ndrgostit de matematic, datorit faptului c a avut
ca profesor, pe celebrul profesor Alexandru Cojcaru, al crui exemplu l-a urmat i ca dascl n
toat cariera lui.
n ntreaga sa activitate, domnul profesor Mihai Gavrilu a condus numeroase colective de
elevi, ca diriginte , desluindu-le tainele matematici, cile de o inere a rezultatelor foarte bune
la toate tipurile de examene, dar i antrenndu-i la tot felul de ativiti extracolare, legate de
obeiectul de matematic (concursuri, rezolvri de probleme din Gazeta Matematic i alte reviste,
participarea la sesiuni de comuncri etc.
Ca director la Liceul agricol di Roman (1979-1989) i ca inspector de specialitate la I. .
J. Neam (1990-2000) a fost un real sprijin cadrelor didactice tinere n drumul lor spre obinerea
denitivatului n nvmnt i apoi pentru obinerea gradelor didactice didctice II i I. A tiut s
in legtura ntre generaii i a convins cu experiena i cu rezultate deosebite s in cursuri cu
profesorii tineri sau s-i primeasc s asiste la ore pentru o mai bun pregtire n vederea obinerii
gradelor didactice sau a perfecionrilor ce se desfurau periodic, la nivel naional. De foarte
multe ori, a antrenat profesorii n diferite activiti extracolare (concursuri, simpozioane, sesiuni
de comunicri etc).
n anul 2000 se pensioneaz i, cum s-ar  ateptat toat lumea, ar  trebuit s-l vedem
mai rar pe domnul profesor la coal i mai mult n familie. Dar n-a fost deloc s e aa! Familia,
care i-a apreciat ntotdeauna activitatea i l-a nteles n toate mprejurrile, i-a fost i de ast dat
alturi i i-a respectat hotrrea de a-i continua activitatea.

361

i cum cei mptimii,.nu preget nici timp, nici efort pentru ca visele lor s devin realitate,
iat realizrile profesorului Mihai Gavrilu, dup pensionare:
Colaboreaz cu cteva edituri din ar i duce profesorilor i elevilor ultimile noutti din
matematic;
Este cel mai vechi, dar i cel mai activ, intermediar ntre redacia revistei ,,Gazeta Matematici coal, fcnd ca elevii i profesorii de la cele mai izolate coli ale judeului (i chiar din
judeele limitrofe), s benecieze de abonamentele la aceast revist, transportul fcndu-l, de
ecare dat, pe propria lui cheltuial;
Este prezent, cu elevi sau fr elevi, la aproape toate concursurile interjudeene ce se
organizeaz n prezent n ar; de multe ori convinge doi sau trei profesori s-l nsoeasc i face ca
drumul pn la localitatea unde se organizeaz concursul s e o excursie de neuitat, pentru c nu
permite s se sar vreun obiectiv turistic sau istoric, fr a-l vizita.
n ecare vacan de primvar particip la faza national a Olimpiadei de Matematic;
Public articole sau propune probleme n diferite reviste de specialitate (Gazeta Matematic  Bucureti, Axioma  Supliment Matematic  Ploieti etc.), ndeamn i chiar insist s fac
acelai lucru i colegii lui cu experien au mai tineri;
n anul 2004 scrie, n colaborare cu Vasile Berinde i Andrei Horvat-Marc, cartea ,,Mathematics Competitions, scoas la Editura CUB PRESS 22 din Baia Mare, lucrare ce a fost
prezentat n luna iulie a aceluia an la Copenhaga, la al X-le Congres de Educaie Matematic (ce
se desfoar din doi n doi ani), unde sunt prezentate sistemele de nvmnt din cteva ri ce
sunt anunate de la ediia anterioar;
. Este omul care a inut n via Filiala Roman a S. S. M. R., a crui preedinte devine dup
anul 2000. n aceast calitate, stabilete colaborri cu celelalte liale din ar, cu C.C.D. Neam,
cu Primria Muncipiului Roman, cu liceul cu Program sportiv Roman, cu C. N. Roman-Vod din
Roman, cu Gr. colar Vasile Sav i alte coli sau licee sau din ar.
n colaborare cu membri Filialei Roman a S. S> M. R. i cu ali simpatizani au loc
urmtoaerle activiti:
Cocursul ,,Viitorii Matematicieni, pentru clasele II-VI (mai-iunie), iniiat de domnul
profesor Mihai Gavrilu, care a ajuns la ediia a VI-a, ediie ce s-a desfurat n 7 iunie 2008, la
Liceul cu program sportiv din Roman.
Domnul profesor Mihai Gavrilu a organizat Concursul interjudeean Gheorghe Vrnceanu, pentru clasele VII-XII, la Roman, n decembrie 1995 i Concursul interjudeean Memorial
Alexandru Cojocaru pentru clasele II-XII (2004 i 2006).
organizarea sesiunilor de referate i comunicri metodico-tiinice, pentru profesori, nvtori i educatori din judeul Neam, ajungndu-se la ediia a VII-a, ediie desfurat n 3
aprilie 2008 la Liceul cu Program sportiv din Roman, unde s-au conferit i numeroase diplome de
excelen, din partea S. S. M. R., dar i din partea Filialei Roman, unor personaliti care de-a
lungul anilor au sprijinit activitile legate de matematic, ale lialei, la diverse concursuri, simpozioane sau sesiuni de comunicri, personaliti, care, prin meseria lor nu aveau nici o legtur
cu matematica. La aceste sesiuni, au participat de ecare dat profesori, nvtori sau educatori
din toate judeele Moldovei, dar i din alte judee: Timi, Prahova etc.
Organizarea simpozioanelor cu temele ,,110 ani de la ninarea Gazetei Matematice
(martie 2006), ,,Matematica i religia (octombrie 2006), ,,Matematica i sportul (iunie 2007),
,,Matematica i poezia (octombrie 2007), ,,Eminescu i Matematica (15 ianuarie 2008), ,,Artele i
matematica (octombrie 2008).
Domnul profesor Mihai Gavrilu susine n ecare an comunicri la diverse sesiuni metodico-tiinice ce se organizeaz la Sinaia, Ploieti, Roman, Bucureti.
mpreun cu membri Filialei Roman a S. S. M. R., particip n ecare an cu lucrri sau
comunicri interesante la Conferinele anuale ale S. S. M. R. organizate n diferite lcaliti din ar.
Este unul din cei mai vechi participani la Cursurile de var organizate de S.S.M.R., la
ncepput la Predeal i apoi la Buteni. Bineneles c nu vine niciodat singur i c n ecare ana
caut s mreasc numrul colegilr care l nsoesc.
Aceast bogat activitate l recomand pe domnul profesor Mihai Gavrilu pentru a 
desemnat un bun exemplu demn de urmat de ctre tinerii notri. Ne bucurm c am avut onoarea de
a-l cunoate ca om, profesor i coleg i i mulumim c i la aceast vrst a rmas del matematicii.
Suntem mndri c romacanul nostru a pus o piatr de baza la temelia nvmntului matematic
preuniversitar din Romnia i indiferent de epoca istoric, a luptat cu metodele lui ca elevii s e
bine pregtii i condui n drumul spre via.

362

Cred c a fcut multe sacricii n viat pentru a-i vedea realizate visele, dar satisfacia
mplinirii lor a fcut ca ele s e uitate!
Acum, la mplinirea frumoasei vrste de 70 de ani, i urm ct mai muli ani i nc pe
attea realizri.
Marioara Costchescu

REVISTA REVISTELOR
Revista de Matematic din Timioara
Domnul profesor Ion Damina Brchi ne-a expediat numerele 3 i 4 din 2008 ale revistei
timiorene.
Ca deobicei, revistele conin un numr de probleme propuse i rezolvate  foarte interesante
i variate  semnate de unii dintre cei mai prestigioi autori de probleme din ar.
n revist sunt publicate i o serie de scurte note matematice, din care vom aminti:
n nr. 3/2008: ,,O inegalitate echivalent cu inegalitatea mediilor (D. Mrghidanu, D.
t. Marinescu, V. Cornea ), ,,Generalizarea unei probleme date la O. M. J. - 2007 (N. Stanciu ).
n nr. 4/2008: ,,Generalizarea inegalitii Stevin-Bottema (M. Cucona ), ,,O inegalitate
vericat de funcii convexe (O. T. Pop ).
Dan Radu

Revista de matematic a elevilor i profesorilor


din judeul Cara-Severin
De la Reia am rimit nr. 23 i 24 (an IX-2008) ale revistei editate de liala Cara Severin
a S. S. M. R.
Vom meniona titlurile ctorva articole i note inserate n aceste numere:
n nr. 24: prima parte a articolului ,,Fractali semnat de G. Mahalu ), precum i notele
,,Mulimi legate (N. Stniloiu ) i ,,Inegalitatea lui Sylvester (L. Dragomir );
n nr. 25: ,,Puncte importante n triunghi (Marina Constantinescu i Mircea Constantinescu ), ,,Generalizarea unei probleme de concurs (N. Stniloiu ), ,,Partea ntreag a i partea
fracionar a unui numr real (L. Dragomir ).
Desigur, revistele conin i un mare numr de probleme propuse spre rezolvare elevilr de
gimnaziu i liceu.
Dan Radu

Axioma  supliment matematic


Prin bunvoina domnului profesor Gheorghe Crciun, redactorul coordonator al publicaiei, am primit numerele 26 i 27 din 2008 ale publicaiei ploietene editate sb egida ,,Fundaiei
oamenilor de tiin din Prahova.
Vom aminti titlurile a trei interesante note matematice publicate n aceste dou numere:
,,Asupra unui sosm geometric (M. Oprea  n nr. 26/2008), ,,Asupra conjecturii lui Collatz (t.
Stroe  n nr. 26/2008), ,,Asupra algoritmului lui Euclid (M. Oprea  n nr. 27/2008).
Demn de menionat este faptul c revista are tendina de a se rspndi din ce n ce mai
mult pe lan naional, fapt atestat de spaiul larg ocupat de listele de rezolvitori din diversele zone
ale rii.
Dan Radu

Sfera  revist de matematic


Din Bileti am primit numrul 12 (2/2007-2008) al revistei locale dedicate nvmntului
matematic preuniversitar. Revista pe care am mai prezentat-o n cadrul acestei rubrici, i continu
cu consecven apariia de ase ani (dou numere n ecare an colar, structurate n funcie de

363

materia ce urmeaz s e parcurs n semestrul respectiv), tinzndu-se s-i gseasc locul printre
publicaiile de prol ce apar n ara noastr.
Dintre notele matematice inserate n acest numr amintim: ,,Asupra unor probleme din
Gazeta Matematic (D. M. Btineu-Giurgiu ), ,,Ecuaii cu o innitate de soluii reale (I. Ivnescu ), ,,Probleme de geometrie plan rezolvate cu ajutorul geometriei n spaiu (M. D. Gurgui ),
,,Inegalitile lui Cebev dintr-o nou perspectiv (S. Pupan ).
Dan Radu

RECENZII
A. R. RAJWADE, A. K. BHANDARI, Surprises and Counterexemples
in Real Function Theory, Hindustan Book Agency, 2007

Analiza matematic ofer, poate mai mult dect oricare alt ramur a matematicii, game
extrem de diversicate de situaii, cu numeroase nuane posibile. Tot att de diversicate sunt i
diferitele contraexemple, la diverse capitole, ceea ce a fcut ca s se resimt necesitatea de a le
sistematiza n anumite cri destinate special acestora. Cea mai cunoscut carte de acest fel este,
probabil, cea a autorilor Gelbaum i Olmsted ,,Counterexamples in Analysis (Holden-Day, Inc.
San Francisco, London, Amsterdam, 1964), tradus i n limba romn n 1973.
Cartea pe care o prezentm acum trateaz, poate, mai puine probleme, dar acestea sunt
dintre cele mai interesante, mai grele i toate sunt aprofundate n detaliu. Conine urmtoarele 7
capitole:
1. Introduction to the real line R and some of the subsets
2. Functions: Pathological, peculiar and extraordinary
3. Famous everywhere continuous, nowhere, dierentiable functions: Van der Waerden's and
others.
4. Functions: Continuous, periodic, locally recurent and others
5. The derivative and higher derivatives
6. Sequences, Harmonic Series, Alternating Series and related results
7. The imnite exponential xx

.
x.

and related results.

Lucrarea se ntinde pe 290 de pagini, conine multe guri, dou apendixuri, o bibliograe
cu 118 titluri i un index.
Foarte detaliat redactat, cu o prezentare grac excelent, cartea ofer un material foarte
interesant, de mare completitudine i profunzime i prilejuiete o lectur pansionant. O recomandm cu mult cldur.
Andrei Vernescu
TEFAN OLTEANU, IOANA CRCIUN,
Profesorul Miron Oprea  ieri i azi,
Editura PREMIER, Ploieti, 2007

Volumul omagial ,,Profesorul Miron Oprea  ieri i azi a fost publicat sub egidaa Fundaiei
oamenilor de tiin din Prahova. El vine s ilustreze viaa i activitatea unei persomnaliti marcante prahovene, a unui om care i-a dedicat ntreaga via propririi nvu amntului matematic
i activismului social.
Iat una dintre profesiile de credin ale profesorului Miron Oprea :
,,. . . Am crezut i cred n adevrurile venice, eterne i neperisabile i, de aceea, primul meu
mesaj ctre tineret (n special, cel din coli) este: nvai matematic, aceasta v va face s gndii
corect i clar n orice situaie, . . . , v va face mai detepi dect alii i v apropie de Dumnezeu
. . . . mi iubesc neamul meu romnesc i de aceea mi cert semenii i elevii ori de cte ori fac abateri
grave de la calea adevrului . . . .
Volumul debuteaz cu o scurt biograe a profesorului, semnat de Ioana Crciun i Andrei
Olteanu. Piesa central o constituie, ns, un amplu interviu  realizat n anul 2005 de Andrei

364

 n care, n cuvinte simple, dar bine alese  profesorul i radiograaz viaa i i expune
crezurile. n ne, n ultima parte sunt consemnate o serie de cuvinte omagiale adresate lui, cu
ocazia mplinirii a 75 de ani, de ctre o serie de personaliti locale, precum i alte dou scurte
interviuri menite s creioneze mai bine personalitatea Omului i Profesorului.
Volumul conine o bogat iconograe, ceea ce face din lecturarea lui un demers plcut i
instructiv.
Crciun

Dan Radu
ION NEDELCU, ANCA TUESCU, LUCIAN TUESCU,
Probleme de matematic pentru concursuri,
Editura REPROGRAPH, Craiova, 2007

Volumul de fa conine circa 350 de proleme, aproape toate ind originale, ele purtnd
semntura autorilor ntr-o serie de reviste din ar i din strintate.
Problemele, fr a  deosebit de dicile, ntrunesc exigenele unor probleme de concurs,
scop pentru care au i fost create, soluiile acestora  prezentate pentru toate problemele  sunt de
multe ori ingenioase i incitante pentru elevii de gimnaziu sau liceu.
Cartea se adreeaz, n primul rnd, elevilor ce pregtesc diverse concursuri colare, dar i
profesorilor angrenai n pregtirea lor.
Dan Radu

POTA REDACIEI
Dan Giurgiu  coala nr. 37 din Craiova. Am primit materialul cu titlul ,,Curs opional
de matematic la clasa a VIII-a  Relaii metrice. l vom supune ateniei Colegiului Redacional.
Alina Sntmrian  Departamentuul de matematic al Universitii Tehnice din Cluj.
Articolul dumneavoastr cu titlul ,,Approximations for a generalization of Euler's constant se a
n studiul Comitetului de Redacie.
Dumitru Btineu-Giurgiu  Bucureti. Nota matematic cu titlul ,,Din nou asupra
problemei 50 din Gazeta MAtematic seria A se a n atenia Colegiului Redacional care va
decide aupra oportunitii publicrii lui.
Dorian Licoiu  coala cu clasele I-VIII din Dbuleni. Articolul pe care ni l-ai expediat
cu titlul ,,Criteriul de divizibilitate cu numere Fermat va  supus analizei Colegiului Redacional.
Marian Tetiva  Colegiul Naional ,,Gheorghe Roca-Codreanu din Brlad. Nota matematic intitulat ,,Asupra unei congruene utile n demonstraia teoremei Erds-Grinsberg-Zio se
a n atenia Colegiului redacional. De asemenea, am primit i o problem propus de dumneavoastr.
Laureniu Modan  Bucureti. Am primit cele dou probleme propuse. Le vom supune
ateniei Colegiului Redacional.
Ovidiu Pop  C. P. 514, O. P. 5, 440310 Satu Mare. Am primit problema propus de
dumneavoastr. O vom supune ateniei Colegiului Redacional.
Nicuor Minculete  Universitatea Dimitrie Cantemir din Braov. Am primit articolul
dumneavoastr cu titlul ,,The extension of Koci's inequality to the convex quadrilateral, Colegiul
Redacional urmnd s-l analizeze, fapt ce este valabil i pentru cele ase probleme propuse.
Mihail Bencze  Str. Hrmanului, nr. 6, 505600 Scele. Am primit problema propus de
dumneavoastr O vom analiza n cadrul Comitetului de Redacie.
Alexandru Szke  Facultatea de Matematic i informatic a Universitii din Bucureti.
Articolul dumneavoastr intitulat ,,Aspecte ale implementrii reelelor de pocesare evoluionist
se a n prezent la referat, urmnd ca apoi s hotrm n ce msur este publicabil sau nu.
Dan Radu

ERAT

1.

n G.M.-A nr.

4/2007, la pag.

se repet.

2.

La pag. 103 din G.M.-A nr.

341, primele cinci rnduri vor  omise, deoarece

2/2008, n scrierea relaiilor (31),

(32), (33), (35)

avem respectiv:

ha + hb + hc + hd 16r

(n loc de ,,=);

365

16R
(n loc de ,,=);
3
64
m2a + m2b + m2c + m2d
R2 (n loc de ,,=);
9
1
b21 + b22 + b23 16R2 (n loc de ,,=).
4

ma + mb + mc + md

2.

La pag. 105, tot din G.M.-A nr.

2/2008, avem ( du stabilirea inegalitii (46):

,, Similar punctului c) . . . . . . rezult:

d21 + d22 + d23 b21 + b22 + b23 4R2 ,


(47)
....................................

2
1
1
d21 + d22 + d23 (d1 + d2 + d3 )2
54 3 r3 R2 = . . . . . .
3
3
( n loc de )
n G.M.-A nr.
n G.M.-A nr.

loc de

GLn (R).

n G.M.-A nr.

3/2008, la pag. 282, rndul 19 de sus se va citi ,,23 n loc de 28.


3/2008, pe coperta IV, rndul 4 de sus se va citi ,,L / GLn (R)
3/2008,

la pag.

283, n titlul primei recenzii de carte se va citi

,,. . . Paatero n loc de ,,Paajero.

Redacia

Anun important
n urma unei edine a Comitetului de redacie provizoriu s-a hotrt ca, ncepnd
cu numrul 1 din anul 2009, formatul revistei s e modicat. Astfel, rubricile permanente ale revistei vor  urmtoarele:
1. Articole tiinice i de informare tiinic
2. Note matematicearticole metodice
3. Examene i concursuri
4. Didactica matematicii
5. n sprijinul cursurilor obionale
6. Puncte de vedere
7. Probleme propuse
8. Soluiile problemelor propuse
9. Istoria matematicii
10. Manifestri tiinice
11. Din viaa societii
12. Recenzii
Rubricile Revista revistelor i Pota redaciei vor disprea. Conrmarea primirii
materialelor expediate de autori se va face pe sit-ul societii.
Rugm, pe aceast cale, autorii s fac, pentru materialele expediate, o recomandare
de ncadrare ntr-o anumit rubric.

Redacia

TABLA DE MATERII
Vol. XXVI (CV) 2008
366

I. Articole tiinice i de informare tiinic, articole metodice

1. A. L. Agore

Jocul cu numere i axiome: Sisteme Peano-Dedekind . . . . . . . . 4

G. Militaru

2. W.G.Bosko

An exploration of Hilbert's Neutral geometry . . . . . . . . . . . . . . . 1

Bogdan Suceav
Adrian I. Vjiac

3. Costel Chite
4. Cristina-Diana

Teorema de factorizare i aplicaii . . . . . . . . . . . . . . . . . . . . . . . . . . . 4


Aplicaii ale statisticii matematice n sport . . . . . . . . . . . . . . . . . . 1 38

Costandache

5. Solomon Marcus
6. Dorin
Mrghidanu

7. Mihai Miculi

Singurtatea matematicianului . . . . . . . . . . . . . . . . . . . . . . . . . . . . . 3 165


Generalizri ale inegalitilor lui Young, Hlder,
Rogers i Minkovski . . . . . . . . . . . . . . . . . . . . . . . . . . . . . . . . . . . . . . . . 3 208
Ranri ale unor inegaliti geometrice n tetraedru . . . . . . . . . 1 29

Marius Olteanu

8. Marius Olteanu
9. Vasile Pop
10 A. Reisner

11. Doru tefnescu


12. Marian Tetiva
13. Andrei Vernescu

Noi ranri ale inegalitii lui Durrande n tetraedru . . . . . . . .


Izometrii liniare n Rn i Cn . . . . . . . . . . . . . . . . . . . . . . . . . . . . . . . .
Grupuri de matrici din Mn (R) i aplicaii la rezolvarea
ecuaiei difereniale LX 0 + M X = 0, unde M Mn (R)
i GLn (R) . . . . . . . . . . . . . . . . . . . . . . . . . . . . . . . . . . . . . . . . . . . . . . . . . .
Margini pentru rdcinile polinoamelor cu coecienii compleci . . . . . . . . . . . . . . . . . . . . . . . . . . . . . . . . . . . . . . . . . . . . . . . . . . . . . . .
O nou demonstraie a inegalitii lui Surnyi. . . . . . . . . . . . . . .
Aproximarea polinomial uniform a funciilor continue [2] . .

2 98
4

3 197
4
2 93
1 23

II. Examene i concursuri

1. Andrei Halanay
2. Eugen Pltnea
3. Vasile Pop

Concursul Studenesc Traian Lalescu  Tradiii i Modernitate 4


Examenul pentru obinerea gradului didactic II, sesiunea
august 2007, Universitatea Transilvania din Braov. . . . . . . . . . 1 47
Olimpiada Internaional Sudeneasc SEEMOUS  2008 . . . 4

Dorian Pop

4. Dan Schwarz
5.
6.

IMC 2007, Blagoevgrad, Bulgaria . . . . . . . . . . . . . . . . . . . . . . . . . . . 2 109


Subiectele date la Universitatea din Bucureti . . . . . . . . . . . . . . . 4
Concursul Traian Lalescu  Faza naional 2008,
Seciunea matematic (A) . . . . . . . . . . . . . . . . . . . . . . . . . . . . . . . . . . 4

III. Puncte de vedere

1. Laureniu Modan

Starea actual a nvmntului superior romnesc . . . . . . . . . . 2 119

IV. Sugestii pentru cursurile opionale

1. Ctlina Anca
Isofache

Modelarea din punct de vedere matematic a unor


fenomene din natur.Curs opional integrat de
biomatematic pentru clasa a XI-a . . . . . . . . . . . . . . . . . . . . . . . . . . 1 51

V. Note matematice

1. W.G. Bosko
2. Anghel Dana

Behind an elementary problem of geometry . . . . . . . . . . . . . . . . .


Imagini geometrice ale mulimii numerelor reale deduse
dintr-o problem de loc geometric . . . . . . . . . . . . . . . . . . . . . . . . . . .
3. Gabriel Dospinescu How many disjoint subsets with a given sum of elements
Marian Tetiva
can {1, 2, . . . , n} have? . . . . . . . . . . . . . . . . . . . . . . . . . . . . . . . . . . . . .
4. Nicuor Minculete O nou demonstraie a inegalitii lui Erds-Mordell i
extinderea ei . . . . . . . . . . . . . . . . . . . . . . . . . . . . . . . . . . . . . . . . . . . . . . . .
5. A. Popescu-Zorica La constante d'Euler exprime par des intgrales . . . . . . . . . . .
6. A. Reisner
Asupra comutantului unui endomorsm (unei matrice
7. M. Tetiva
Legturi neateptate . . . . . . . . . . . . . . . . . . . . . . . . . . . . . . . . . . . . . . . .
6. A. Vernescu
O demonstraie elementar a inegalitii lui Jordan, . . . . . . . . .

2 126
2 131
3 216
3 231
3 220
4
2 128

Dan Coma

367

VI. Note metodice

1. Gh. Costovici
2. Liliana Crciun
Ioan Totolici

3. Dan Pleu

Exemple de monoizi izomor . . . . . . . . . . . . . . . . . . . . . . . . . . . . . . . . 2 134


Demonstrarea unor inegaliti cu ajutorul generalizrii
inegalitii lui Cauchy la matrici . . . . . . . . . . . . . . . . . . . . . . . . . . . . 3 235
Vindem cri la kilogram! . . . . . . . . . . . . . . . . . . . . . . . . . . . . . . . . . . 3 244

VII. Istoria matematicii

1. T. Brsan, Dan Tiba Recreaii tiinice  125 de ani de la apariie. . . . . . . . . . . . . . .


2. M. Buliga
Spaii neolonome ale lui Vrnceanu din punctul de vederea
al geometriei distanei . . . . . . . . . . . . . . . . . . . . . . . . . . . . . . . . . . . . . .
3. Liviu Florescu
Prof. dr. doc. Ilie Popa. In memoriam  100 de ani
de la natere . . . . . . . . . . . . . . . . . . . . . . . . . . . . . . . . . . . . . . . . . . . . . . . .
4. Eufrosina Otlcan La 100 de ani de la naterea academicianului
Nicolae Teodorescu n contextul european al tiinei . . . . . . . . .
5. Ralf Schindler
Kurt Gdel (1906-19785), . . . . . . . . . . . . . . . . . . . . . . . . . . . . . . . . . . .
6. N. Stanciu
Despre irul lui Fibonacci . . . . . . . . . . . . . . . . . . . . . . . . . . . . . . . . . . .
7. V. ugulea
Cteva amintiri despre Societatea de tiine Matematice
din Romnia . . . . . . . . . . . . . . . . . . . . . . . . . . . . . . . . . . . . . . . . . . . . . . . .

2 150
4
2 154
4
1 72
3 265
3 278

VIII. Manifestri tiinice

1. C.P.Niculescu,

2. A. Vernescu

3. A. Vernescu,
4. A. Vernescu
5.

A IX-a Conferin Naional de Matematic i Aplicaii


(CAMA'07),Iai, 26-27 octombrie 2007 . . . . . . . . . . . . . . . . . . . . . .
............................................................
Simpozion de Funcii Complexe n Onoarea Doamnei Profesor
Cabiria Andreian-Cazacu, Facultatea de Matematic i
Informatic, Bucureti, 18 februarie 2008. . . . . . . . . . . . . . . . . . . .
Al Optulea Seminar Romno-German de Teoria
Aproximrii i Aplicaii, Sibiu, 28 mai-1 iunie 2008 . . . . . . . . . .
Semicentenarul Institutului de Calcul Numeric ,,Tiberiu
Popoviciu din Cluj-Napoca, 7-10 mai 2008 . . . . . . . . . . . . . . . . .
Simpozion dedicat revistei ,,Recreaii tiinice (1883-1888)

1 76
2 147

4 336
3 281
3 280
4

IX. Din viaa Societii

1. Dan Coma
2. M. Costchescu
3. L. Modan
4. D. Radu
5. D. Radu
6. M. Trifu
6.

Concursul Interjudeean de Matematic Danubius,


Ediia a doua, Corabia, 10 mai 2008 . . . . . . . . . . . . . . . . . . . . . . .
Profesorul Mihai Gavrilu la 70 de ani . . . . . . . . . . . . . . . . . . . . . .
Profesorul Constantin Corduneanu  O via dedicat
matematicii . . . . . . . . . . . . . . . . . . . . . . . . . . . . . . . . . . . . . . . . . . . . . . . . .
A XXXIV-a Sesiune de comunicri metodico-tiinice
a Filialelor din judeul Prahova ale S.S.M.R. . . . . . . . . . . . . . . .
coala de var de la Buteni . . . . . . . . . . . . . . . . . . . . . . . . . . . . . . . .
Conferina Naional a Societii de tiine Matematice
din Romnia . . . . . . . . . . . . . . . . . . . . . . . . . . . . . . . . . . . . . . . . . . . . . . . .
Lista absolvenilor cursurilor de perfecionare pentru
profesorii de matematic, organizate de S.S.M.R.
Buteni, 29 iulie-7 august 2008 . . . . . . . . . . . . . . . . . . . . . . . . . . . . . .

3 281
4
1 79
1 77
4
1 82

XI. Revista revistelor  rubric permanent redactat de Dan Radu

1. Recreaii matematice, nr. 2/2007 . . . . . . . . . . . . . . . . . . . . . . . . . . . . . . . . . . . . . . . . . . . . . . . . . . . 1


2. Axioma  supliment matematic, nr. 22, 24, 25/2007) . . . . . . . . . . . . . . . . . . . . . . . . . . . . . . . 1
3. Revista de matematic a elevilor i profesorilor din judeul Cara-Severin,
nr. 21 (2007) . . . . . . . . . . . . . . . . . . . . . . . . . . . . . . . . . . . . . . . . . . . . . . . . . . . . . . . . . . . . . . . . . . . . . . . 1
4. Creative Mathematics and Informatics, vol 16 (2007) . . . . . . . . . . . . . . . . . . . . . . . . . . . . . . . 1
5. Revista de matematic mehedinean, nr. 7 (2006) . . . . . . . . . . . . . . . . . . . . . . . . . . . . . . . . . 2
6. Argument, nr. 1o/2008 . . . . . . . . . . . . . . . . . . . . . . . . . . . . . . . . . . . . . . . . . . . . . . . . . . . . . . . . . . . . . 2
7. Revista de Matematic din Galai, nr. 30 (2008) . . . . . . . . . . . . . . . . . . . . . . . . . . . . . . . . . . . . 2
8. Revista de matematic a elevilor i profesorilor din judeul Cara-Severin,

368

85
85
86
86
158
158
159

9.
10.
11.
12.
13.
14.
15.
16.
17.
18.
19.
20.

nr. 22 (2007) . . . . . . . . . . . . . . . . . . . . . . . . . . . . . . . . . . . . . . . . . . . . . . . . . . . . . . . . . . . . . . . . . . . . . . . 2
Revista de Matematic Grigore Moisil, nr. 2 (2007) . . . . . . . . . . . . . . . . . . . . . . . . . . . . . . . . . 2
Carpathian Journal of Mathematics, vol. 23 (nr. 1-2/2007)6 . . . . . . . . . . . . . . . . . . . . . . . . 2
Recreaii matematice, nr. 1 2008 . . . . . . . . . . . . . . . . . . . . . . . . . . . . . . . . . . . . . . . . . . . . . . . . . . . 3
Revista de Matematic din Timioara, nr. 1/2008 . . . . . . . . . . . . . . . . . . . . . . . . . . . . . . . . . . 3
Revista de matematic a elevilor i profesorilor din judeul Cara-Severin,
nr. 19 (2007) . . . . . . . . . . . . . . . . . . . . . . . . . . . . . . . . . . . . . . . . . . . . . . . . . . . . . . . . . . . . . . . . . . . . . . . 3
Creaii matematice, seria B, nr. 2/2007 . . . . . . . . . . . . . . . . . . . . . . . . . . . . . . . . . . . . . . . . . . . . . 3
Revista de matematic i informatic, nr. 1 i 2/2008 . . . . . . . . . . . . . . . . . . . . . . . . . . . . . . . 3
Creative Mathematics and Informatics, vol. 17 (2008) . . . . . . . . . . . . . . . . . . . . . . . . . . . . . . 3
Revista de Matematic din Timioara, nr. 3, 4/2008 . . . . . . . . . . . . . . . . . . . . . . . . . . . . . . . . 4
Revista de matematic a elevilor i profesorilor din judeul Cara-Severin,
nr. 23, 24 (2008) . . . . . . . . . . . . . . . . . . . . . . . . . . . . . . . . . . . . . . . . . . . . . . . . . . . . . . . . . . . . . . . . . . . 4
Axioma  supliment matematic, nr. 26, 27 (2008) . . . . . . . . . . . . . . . . . . . . . . . . . . . . . . . . . . . 4
Sfera  revist de matematic, nr. 2 (2007-2008) . . . . . . . . . . . . . . . . . . . . . . . . . . . . . . . . . . . . 4

159
159
160
281
282
282
282
282
283

XII. Recenzii

1. M. Ivan

8. D. Radu

Geometrie pentru gimnaziu, liceu i concursuri,


Editura MEDIAMIRA, Cluj, 2007 . . . . . . . . . . . . . . . . . . . . . . . . . . . . . .
Liliana Niculescu, Metoda reducerii la absurd, Editura GIL,
Zalu, 2006 . . . . . . . . . . . . . . . . . . . . . . . . . . . . . . . . . . . . . . . . . . . . . . . . . . . . .
Bogdan Enescu, Arii, Editura GIL, Zalu, 2006 . . . . . . . . . . . . . . . . .
Nicuor Minculete, Teoreme i probleme specice de geometrie,
Editura EUROCARPATICA, Sf. Gheorghe, 2007 . . . . . . . . . . . . . . .
Dan Schwarz, Gabriel Popa, Probleme de numrare, Editura
GIL, Zalu, 2006 . . . . . . . . . . . . . . . . . . . . . . . . . . . . . . . . . . . . . . . . . . . . . . .
Virgil Nicula, Cosmin Pohoa, Diviziune armonic Editura
GIL, Zalu, 2007 . . . . . . . . . . . . . . . . . . . . . . . . . . . . . . . . . . . . . . . . . . . . . . .
Iurie Boreico, Marinel Teleuc, Invariani i jocuri Editura GIL,
Zalu, 2007 . . . . . . . . . . . . . . . . . . . . . . . . . . . . . . . . . . . . . . . . . . . . . . . . . . . . .
Bogdan Enescu, Polinoame, Editura GIL, Zalu, 2007, . . . . . . . . . .

9. D. Radu

Maria Elena Panaitopol,

2. R. Miculescu
3. R. Miculescu
4. R. Miculescu
5. R. Miculescu
6. R. Miculescu
7. R. Miculescu

10. D. Radu
11. D. Radu

12. D. Radu
13. D. Radu
14. D. Radu
15. D. Radu
16. D. Radu

17. A. Vernescu

18. A. Vernescu

Vasile Pop,

Laureniu Panaitopol

98

1
1

87
88

90

2 160
2 162
2 162
1

86

88

, Probleme de

geometrie plan, soluii trigonometrice, Editura GIL, Zalu,


2006 . . . . . . . . . . . . . . . . . . . . . . . . . . . . . . . . . . . . . . . . . . . . . . . . . . . . . . . . . . .
Ion Cucurezeanu, Ptrate i cuburi de numere ntregi, Editura
GIL, Zalu, 2007 . . . . . . . . . . . . . . . . . . . . . . . . . . . . . . . . . . . . . . . . . . . . . . .
Pantelimon George Popescu, Ioan V. Maftei, Jos Luis Diz
Barrero, Marian Dinc, Inegaliti matematice, Editura
DIDACTIC I PEDAGOGIC, Bucureti, 2007 . . . . . . . . . . . . . .
Gheorghe Crciun i colaboratorii, Duelul matematic, Editura
TIPARG, Piteti, 2007 . . . . . . . . . . . . . . . . . . . . . . . . . . . . . . . . . . . . . . . . .
Eduard Dncil, Ioan Dncil, Ghidul nvtorului, Editura
IULIAN, Bucureti, 2008 . . . . . . . . . . . . . . . . . . . . . . . . . . . . . . . . . . . . . . .
Eduard Dncil, Ioan Dncil, Matematica servete!, Editura
ERCPRES, Bucureti, 2007 . . . . . . . . . . . . . . . . . . . . . . . . . . . . . . . . . . . .
tefan Olteanu, Ioana Crciun, Profesorul Miron Oprea  ieri
i azi, Editura PREMIER, Ploieti, 2007 . . . . . . . . . . . . . . . . . . . . . . . .
Ion Nedelcu, Anca Tuescu, Lucian Tuescu, Probleme de
matematic pentru concursuri, Editura REPROGRAPH,
Craiova, 2007 . . . . . . . . . . . . . . . . . . . . . . . . . . . . . . . . . . . . . . . . . . . . . . . . . . .
Dorel I. Duca, Eugenia Duca, Exerciii i probleme de Analiz
matematic, vol. I, Editura CASA CRII DE TIIN,
Cluj, 2007 . . . . . . . . . . . . . . . . . . . . . . . . . . . . . . . . . . . . . . . . . . . . . . . . . . . . . .
Dumitru Popa, Exerciii de Analiz matematic, Biblioteca

2 160

2 161
2 162
3 284
31 285
1

75

75

87

369

19. A. Vernescu

20. A. Vernescu

Societii de tiine Matematice din Romnia, Editura MIRA,


Bucureti, 2007 . . . . . . . . . . . . . . . . . . . . . . . . . . . . . . . . . . . . . . . . . . . . . . . . 1 89
Rolf Nevanlinna, Veikko Paajero, Introduction to Complex
Analysis, Second Edition, AMS CHELSEA PUBLISHING,
Providence, Rhode Island, 2007 . . . . . . . . . . . . . . . . . . . . . . . . . . . . . . . . . 3 283
A. R. Rajvade, A. K. Bhandar, Surprises and Contraexemples
in Real Function Theory, Hindustan Book Agency, 2007 . . . . . . . . 4

XIII. Probleme propuse  rubric permanent redactat de Dan Radu


i Radu Miculescu

1.
2.
3.
4.
5.
6.
7.

D. Btineu-Giurgiu (261, 273)


Mihly Bencze (257)
Radu Gologan (269)
Dorin Mrghidanu (272)
Nicuor Minculete (256)
Laureniu Modan (258)
Marius Oltenu (263)

8.
9.
10.
11.
12.
13.
14.

Dan Radu (253, 259, 264)


George Stoica (265)
Rbert Szsz (271)
Gh. Szllsy (255, 267)
Doru tefnescu (274)
Marian Tetiva (236, 239, 245)
Daniel Vcreu (268)

XIV. Soluiile problemelor propuse  rubric permanent redactat de


Dan Radu i Radu Miculescu
233 (Constantin P. Niculescu, Andrei Vernescu), 234 (Dan Radu, Marian Tetiva,
Nicuor Minculete, Gheorghe B. G. Niculescu, Marius Olteanu), 235 (Marcel ena,
Gheorghe B. G. Niculescu, Marian Tetiva, Ioan Ghi, Marius Olteanu),
236 (Marian Tetiva), 237 (Marius Olteanu, Nicuor Minculete, Ioan Ghi) . . . . . . . .
238 (Dan Radu, Marian Tetiva, Rbert Szsz, Benedict G. Niculescu),
239 (Marian Tetiva, Rbert Szsz, Benedict G. Niculescu), 240 (Vasile Crtoaje,
Rbert Szsz, Marius Olteanu), 241 (Gh. Szllsy, Marian Tetiva, Nicuor Minculete,
Marius Olteanu, Rbert Szsz, Benedict G. Niculescu), 242 (Nicuor Minculete,
Marian Tetiva, Marius Olteanu, Rbert Szsz, Benedict G. Niculescu, Dorel Bian)
243 (Dan Radu, Marian Tetiva, Marius Olteanu, Gheorghe B. G. Niculescu),
244 (Vasile Crtoaje), 245 (Marian Tetiva, Marius Olteanu, Ilie Bulacu,
Gheorghe B. G. Niculescu), 246 (Dan Coma, Ilie Bulacu, Gheorghe B. G. Niculescu),
247 (Ovidiu Pop, Marius Olteanu, Gheorghe B. G. Niculescu) . . . . . . . . . . . . . . . . . . . . . .
248 (Dan Radu), 249 (Vasile Crtoaje, Marian Tetiva), 250 (Gabriel Dospinescu,
Marian Tetiva), 251 (Mihai Miculia, Marius Olteanu, Nicuor Minculete,),
252 (Gh. Szllsy, Marian Tetiva) . . . . . . . . . . . . . . . . . . . . . . . . . . . . . . . . . . . . . . . . . . . . . . . . . .

XV. Pota redaciei  rubric permanent redactat de Dan Radu


1 (91); 2 (163); 3 (285); 4 (5).

370

64

140

249

S-ar putea să vă placă și